Сохранен 516
https://2ch.hk/sci/res/294030.html
24 декабря Архивач восстановлен после серьёзной аварии. К сожалению, значительная часть сохранённых изображений и видео была потеряна. Подробности случившегося. Мы призываем всех неравнодушных помочь нам с восстановлением утраченного контента!

Вопросов тред №26

 Аноним 26/08/15 Срд 13:48:56 #1 №294030 
14405861360980.jpg
Вулканирует.
Задаем здесь вопросы, которые не достойны отдельного треда.
Прошлый >>287321 (OP)
Аноним 26/08/15 Срд 14:04:27 #2 №294032 
>>293997
>Кто тебе мешал не быть униженем, отпизженым и развлеченым, омеженька?
По сути единственный, кто меня унижал и пиздил - это отец. Но я вырос бунтарем, если ты не заметил.
>>294005
Эта макака отвечает вместо меня.
Аноним 26/08/15 Срд 14:39:36 #3 №294035 
>>294032
>этот промахивающийся мимо тредов омежка
А ручонки-то дрожат, хаха.
Аноним 26/08/15 Срд 14:52:20 #4 №294042 
>>294030 (OP)
Как научить Хром понимать Ок Гугл на уровне андроида?
Есть вебка, голосовой поиск в районе "ок гугл" - в поисковой строке появляется ответ по заросу "ок гугл"
Аноним 26/08/15 Срд 14:54:14 #5 №294043 
>>294042
Это очень серьёзная научная проблема. Её только Ньютон мог решить, если легенды не врут.
Аноним 26/08/15 Срд 14:55:03 #6 №294044 
>>294043
Поэтому и спрашиваю у отцов
Аноним 26/08/15 Срд 14:55:25 #7 №294046 
Если многомировая интерпретация верна, то как можно пометить эту вселенную, чтобы завтра можно было убедиться, что я всё ещё в ней?
Аноним 26/08/15 Срд 14:55:30 #8 №294047 
>>294042
Не сказал что Хром на пк, проебался
Аноним 26/08/15 Срд 14:57:23 #9 №294048 
>>294028
Фоссилизация — совокупность процессов преобразования погибших организмов в ископаемые. Она сопровождается воздействием различных факторов среды и прохождением процессов диагенеза — физических и химических преобразований, при переходе осадка в породу, в которую они включены.

После гибели организма в первую очередь происходит разрушение мягких тканей, затем — заполнение пустот скелета минеральными соединениями. Иногда пустоты скелета подвергаются пиритизации, ожелезнению, в них могут возникать друзы и включения кальцита, аметиста, флюорита, галенита и т. д. При фоссилизации скелет подвергается перекристаллизации, приводящей к устойчивым минеральным модификациям. Например, арагонитовые раковины моллюсков преобразуются в кальцитовые. Известны случаи минерализации, когда первичный химический состав скелета изменяется (псевдоморфозы). Так, известковые раковины частично или полностью замещаются водным кремнеземом и наоборот. Иногда наблюдаются фосфатизация, пиритизация и ожелезнение минеральных и органических скелетов.

Растения при фоссилизации обычно подвергаются полному разрушению, оставляя т. н. отпечатки и ядра, однако их остатки обнаруживаются в ископаемом виде начиная с докембрия. Также растительные ткани могут замещаться минеральными соединениями, чаще всего кремнеземом, карбонатом и пиритом. Подобное полное или частичное замещение стволов растений при сохранении внутренней структуры называется петрификация.

>ети кости не кости, очепятки не очепятки, там все гнеет, слава нурглу
ОПРОВЕРЖЕНИЕ ХИМИИ ЛИТОСФЕРЫ ОТ ВЕЛИКОГО УЧЕНОГО ДВАЧОВСКИ!!!! ПОКУПАЙТЕ 25 НОМЕР СЦИ!!!!!! В НЕМ ВСЕ ТАК ЖЕ КАК ВЫ ВИДЕЛИ В РЕКЛАМЕ!!!!!

перекатился.
как в настоящих йоба играх.
Аноним 26/08/15 Срд 14:58:28 #10 №294049 
>>294046
А если не верна?
И да, никак. Разве что навалить кучу посреди комнаты перед сном и утром проверять на месте говно или нет.
Аноним 26/08/15 Срд 15:01:19 #11 №294050 
>>294044
https://support.google.com/websearch/answer/2940021?hl=ru
Аноним 26/08/15 Срд 15:03:17 #12 №294051 
>>29404
>2015
>Не знать о полтергейсте-говнокраде
О чём с тобой вообще можно говорить?
Аноним 26/08/15 Срд 15:03:52 #13 №294052 
>>294049
-->>294051
Аноним 26/08/15 Срд 15:05:10 #14 №294053 
>>294051
Еще лучше. Если говно осталось, значит ты не в вселенной с привидениями говнокрадами.
Аноним 26/08/15 Срд 15:08:26 #15 №294055 
>>294053
А если вселенных без говнокрадов несколько?
Аноним 26/08/15 Срд 15:10:01 #16 №294056 
>>294055
Тогда ты точно не в своей личной вселенной с говнокрадами же. Это очевидно любому охотнику на привидений-говнокрадов.
Аноним 26/08/15 Срд 15:11:02 #17 №294057 
>>294056
Что то странное происходит в округе
Кому мы звоним? ОХОТНИКАМ ЗА ГОВНОКРАДАМИ!
Аноним 26/08/15 Срд 15:11:23 #18 №294058 
Что можете сказать про самозарождение ИИ в сети?
Аноним 26/08/15 Срд 15:11:45 #19 №294059 
>>294056
Ты даже не понимаешь сути вопроса. Ты что, школьник, школьник?
Аноним 26/08/15 Срд 15:12:10 #20 №294060 
Раз уж пошла такая телега, то я хотел бы запостировать некие данные по задокументированным цифрам населения планеты.
"Знаменитый английский историк Гиббон, как было уже указано, также интересовался вопросами населения и определил численность населения Римской империи при Антонинах в 120 млн человек. К этой цифре он пришел следующим путем: римских граждан было 20 млн., жителей провинций — вдвое больше, т е. 40 млн., всего свободные — 60 млн., рабов — столько же. Итого — 120 млн. человек. Расчет, как видим, чрезмерно смелый и мало обоснованный.
Помимо Гиббона, ряд других ученых оценивал население Римской империи, но в их оценках нехватало достаточно хорошего знания истории и правильного использования всех источников, которые остались нам от античного мира. Впервые это сделал Белох, который на основании тщательного анализа всего материала определил население Римской империи в 54 млн. человек (к моменту смерти Августа).
Более подробно результаты работы Белоха могут быть представлены в таблице на стр. 20.
Из приведенных цифр видно, что в некоторых странах античного мира уровень плотности был чрезвычайно высок. Наибольшую плотность дает Египет — 179 человек на 1 км2. Этот уровень плотности превзойден современным Египтом лишь в 2,5 раза. Высокая плотность древнего Египта объясняется чрезвычайно благоприятными условиями для земледелия вдоль берегов Нила. Менее высокая плотность, но все же довольно значительная, была в странах Малой Азии. В Европе плотность была ниже, даже в самой Италии — 24 человека на 1 км2. В Галлии плотность падала до 6,3 человека и в дунайских странах до 4,7 человека на 1 км2. Что же касается еще более отдаленных от Рима стран, то в них плотность населения была, наверно, ниже.
Следует указать, что оценка Белоха им самим была определена как минимальная, в ряде случаев действительно он дал слишком заниженные оценки, не доверяя источникам древности. В позднейших своих работах Белох несколько повысил цифры, первоначально им данные."
Аноним 26/08/15 Срд 16:33:53 #21 №294064 
Чет я бля не понял: гравитационые волны и гравитоны слабо или ни хуя не взаимодействуют с веществом, но бля как тогда мы притягтваемся к объектам? Или при притяжении мы поглашаем виртуальные гравитоны, как в слвчае с электромагнитным взаимодействием?

Аноним 26/08/15 Срд 17:14:27 #22 №294069 
Поясните за ядерную физику. Почему каждое ядро свинца снимает с гамма кванта половину его энергии? Выходит что гамма квант в таком случае никогда не исчезнет. Период полураспада тоже в эту же тему.
Аноним 27/08/15 Чтв 10:21:57 #23 №294088 
Расскажите про поляризацию волн. До сих пор не понимаю. Рассмотрим монохроматическую плоскую э.-м. волну. Если она монохроматична, если она плоская, значит она всегда поляризована? Есть выделенные плоскости, в которых колеблются напряженности электрического и магнитного полей. Пусть есть система отсчета, с которой связан ортонормированный базис, пусть один из из ортов направлен вдоль волнового вектора плоской волны. Будет ли смещаться относительно оставшихся двух ортов плоскость, в которой колеблется напряженность электрического поля? Как в данном случае будет работать поляризатор? Он эту плоскость будет поворачивать? Или когда говорят о поляризации, имеют в виду цуги волн, не монохроматичное излучение?
Аноним 27/08/15 Чтв 17:24:11 #24 №294159 
14406854513950.jpg
14406854513961.jpg
Прочитал, что хомо сапиенс почти в самом начале становления вида чуть не пришел кирдык, дословные цитаты:
1,5млн лет назад в африке умерли леса и пришла саванна. 70-75% найденых архантропов- дети и подростки, т.е. большая часть умирало, не успев оставить потомство.
Потому что нас было миллион, возможно несколько миллионов. А потом было 6-10 лет вулканической зимы и под тысячу лет локального похолодания. И нас осталось от 5 до 20 тысяч.
Как называется в научной литературе этот период и где можно прочитать моар о нем? Гугл выдает в основном всякую фигню про pleistocene extinction фауны - но меня интересует именно человеческий вид в этот период.
Аноним 27/08/15 Чтв 18:24:45 #25 №294177 
14406890852370.png
>>294088
Как-то ты сложно мыслишь, ортонормированные базисы какие-то, зачем столько наукообразного шума.

Мысленно приделай шнур к стене и тряси его - получишь поперечные волны. Можешь трясти по горизонтали, поможешь по вертикали, можешь круговыми движениями или как угодно еще.

Далее если провести вектор от "оси" до любой точки веревки (аналог вектора E), то он будет или колебаться в одной плоскости, или описывать круг, эллипс или еще как. Если же он полностью рандомный, то поляризации нет.

Поляризатор в такой модели - это щель, которая будет ослаблять волну, если она не находится полностью в плоскости этой щели. Если волна и щель перпендикулярны, вообще все в ноль потухнет.
Аноним 27/08/15 Чтв 18:39:54 #26 №294181 
Если поебаться в гондоне, это будет считается потеряй девственности?
Аноним 27/08/15 Чтв 19:39:30 #27 №294191 
14406935704840.jpg
>>294030 (OP)
>>294030 (OP)
Почему космические аппараты не плавятся от такой температуры? отмазка за плохую теплопроводность безвоздушной среды не катит, потому что аппараты находятся под непосредственным воздействием прямых солнечных лучей
Аноним 27/08/15 Чтв 19:46:51 #28 №294192 
>>294191
Там зеркала стоят, они отражают всю жару обратно.
Аноним 27/08/15 Чтв 19:54:00 #29 №294195 
>>294191
Температуру чего ты там мерять собрался, олух? Жидкого вакуума? Альбедо? Не не слышал.
Аноним 27/08/15 Чтв 20:03:01 #30 №294197 
14406949819250.jpg
>>294030 (OP)
Что это за стрелка из матана? Что она обозначает и чем отличается от стрелок с прямым острием?
Аноним 27/08/15 Чтв 20:13:17 #31 №294198 
>>294195
Температура абсолютно чёрного тела, лол.
Аноним 27/08/15 Чтв 20:15:36 #32 №294200 
14406957368440.png
>>294197
Аноним 27/08/15 Чтв 20:20:44 #33 №294202 
народ как вы относитесь к идеи о том ,что время это квантовое поле,благодаря которому мы и всё во вселенной меняется и энтропирует,а во время движения чтела со скоростью света или близкой к ней,оно замедляется потому,как частицы времени(далее хронионы),не успевают за телом и отставая от него ослабляют своё воздействие,а около сверхмассивных космических объектов вроде чёрных дыр и т.п. замедляется потому что гравитоны (частицы гравитации),взаимноотталкиваются с хронионами.
пруфоф не будет так как этот бред придумал сам,ничего похожего не видел,если есть ссылку плз.
Аноним 27/08/15 Чтв 20:27:00 #34 №294203 
>>294202
>далее хронионы
Почему тогда не было последующих упоминаний?
Аноним 27/08/15 Чтв 20:27:06 #35 №294204 
>>294197
Это из теории категорий, такими стрелками обозначают морфизмы.
Аноним 27/08/15 Чтв 20:32:07 #36 №294206 
>>294203
да хотел что то по другому написать,но как то не задалось,только сейчас заметил)
Аноним 27/08/15 Чтв 21:11:50 #37 №294213 
Почему сердце бьется?
Как?
Аноним 27/08/15 Чтв 21:19:59 #38 №294216 
>>294213
биение сердца вызвано сокращением мышечной ткани из которой оно состоит,ткань же сокращается от электрических импульсов,исходящих из мозга.
Аноним 27/08/15 Чтв 21:28:16 #39 №294217 
>>294216
Здрасьте. Сердцу не нужен мозг, чтобы биться, у cердца свой собственный ритмоводитель.
Аноним 27/08/15 Чтв 21:34:27 #40 №294219 
>>294216
Сердце получает чувствительную, симпатическую и парасимпатическую иннервацию. Симпатические волокна от правого и левого симпатических стволов, проходя в составе сердечных нервов, передают импульсы, которые ускоряют ритм сердца, расширяют просвет венечных артерий, а парасимпатические волокна проводят импульсы, которые замедляют сердечный ритм и суживают просвет венечных артерий. Чувствительные волокна от рецепторов стенок сердца и его сосудов идут в составе нервов к соответствующим центрам спинного и головного мозга. википедия пруф https://ru.wikipedia.org/wiki/%D0%A1%D0%B5%D1%80%D0%B4%D1%86%D0%B5_%D1%87%D0%B5%D0%BB%D0%BE%D0%B2%D0%B5%D0%BA%D0%B0
и ещё https://ru.wikipedia.org/wiki/%D0%98%D0%BD%D0%BD%D0%B5%D1%80%D0%B2%D0%B0%D1%86%D0%B8%D1%8F
Аноним 27/08/15 Чтв 22:32:41 #41 №294225 
>>294219
Хули все так сложно? Я это мозг, я нихуя не знаю о собственном устройстве и о том, чем я как бы управляю.
Аноним 27/08/15 Чтв 22:36:34 #42 №294227 
>>294225
мы много чего не знаем,но это прекрасно,возможность познавать что то новое о мире в котором ты существуешь есть самое крутое что вообще может быть.
Аноним 27/08/15 Чтв 22:43:16 #43 №294228 
>>294227
Нет, я не хочу ничего познавать, я хочу получать удовольствие. Мне нужно больше дофамина, серотонина и эндорфинов!
Аноним 27/08/15 Чтв 22:45:33 #44 №294230 
>>294228
химия с ботаникой в помощь))
Аноним 27/08/15 Чтв 22:46:48 #45 №294231 
>>294230
может я и ебанутый шизик ,но узнавая что то новое я прям дикий кайф ловлю.
Аноним 27/08/15 Чтв 22:50:43 #46 №294232 
>>294230
Мамка твоя тоже помочь может. Когда я буду её ебать, у меня будут выделятся эндорфины)))
Аноним 27/08/15 Чтв 22:56:26 #47 №294233 
>>294232
ахах смешно прям до слёз))))рили ка я мог забыть.
Аноним 27/08/15 Чтв 23:23:12 #48 №294237 
Какое определение можно дать мышлению с точки зрения эволюции человека? Сознанию?
Аноним 27/08/15 Чтв 23:25:50 #49 №294238 
>>294237
самосознание имышление это продукт эволюции нашего программного обеспечения,необходимая опция для нормального существования высокоразвитых представителей живого мира,наши компьютеры,на данный момент обладают очень примитивным если можно так выразиться сознанием,действие на основе простейших предустановленных опций,без собственной воли и не делая выводов из происходящего с ним,так живут насекомые,моллюски и тому подобные живые организмы,высшие же животные,да да и мы с вами,обладают кроме предустановленных простейших системных опций-инстинктов,ещё и двумя более сложными это,как раз таки возможность анализировать окружающий нас мир и делать на основе своих наблюдений выводы,из этих выводов формируется второй сложный механизм сознания это воля так как мы принимаем решения,как и что делать,на основе анализа окружающей нас среды.так что как бы многоуважаемые гуманитарии не разглагольствовали о душе,на деле она(наша душа)сформирована на основе опыта полученного в процессе жизнедеятельности и что не мало важно генетики(так как хоть и сознание всех младенцев по умолчанию одинаковое,но генетика вносит свои поправки,такие как например задатки темперамента или предрасположенность ко всевозможным психическим расстройствам)
Аноним 28/08/15 Птн 00:08:18 #50 №294243 
>>294237
Мышление - следствие образования неокортекса и второй сигнальной системы, т.к. более социальные и умелые особи были жизнеспособней
Аноним 28/08/15 Птн 00:11:40 #51 №294244 
>>294238
>так что как бы многоуважаемые гуманитарии не разглагольствовали о душе
Во-первых, ты мало что знаешь о физиологии человек, и нихуя не знаешь о мышлении. Это современное положение научного знания. Собственно, на этом можно и закончить, потому что ниче кроме мнения науки и своих домыслов у тебя нету.
Аноним 28/08/15 Птн 00:14:45 #52 №294246 
>>294244
ну многоуважаемый знаток раскажите же нам свою версию.
> ниче кроме мнения науки
лол кто как не наука дает нам понимание окружающего мира.
Аноним 28/08/15 Птн 01:28:40 #53 №294249 
14407145204860.jpg
>>294246
>дает нам понимание окружающего мира.
Аноним 28/08/15 Птн 02:11:09 #54 №294252 
Я хз, двач, просвети меня. Взял книжечку по астрофизике для МГУ, открыл - там формула на формуле и формулой погоняет. Причём формулы такие, что для далёкого от матана человека типа меня они вообще ниочём.

Собсно, имеет ли смысл ознакомление с астрофизикой без знания матана?

Или конкретно матан лично мне там не пригодится (в наше-то время, когда компьютер всё расчитает за тебя)?

Аноним 28/08/15 Птн 02:12:32 #55 №294254 
Анон, как человек становится учёным?
Есть ли официальный статус "учёного", из чего он состоит, кем выдаётся, какие критерии?
Аноним 28/08/15 Птн 02:12:57 #56 №294255 
>>294252
> для МГУ,

для первого курса мгу * фикс
Аноним 28/08/15 Птн 02:16:28 #57 №294256 
>>294252
Зачем тебе ознакамливаться с астрофизикой?
>в наше-то время, когда компьютер всё расчитает за тебя
Совет,не пытайся оправдывать свое незнание и пиздеть про то, что не знаешь.
Аноним 28/08/15 Птн 02:17:09 #58 №294257 
Поскольку тут собралось дохуя верунов в науку, то так уже и быть, в надцатый раз поясню.
>кто как не наука дает нам понимание окружающего мира
Наука - это церковь, есть науный метод, который является неким прикладным, одним из методов систематизации и организации познания.
Научный метод, в свою очередь, понимает под собой создание теорий, верификацию, эксперименты.
Когда мама-питекантроп учила детей пользоваться каменным орудием, то она делала ровно то, что делают современные родители-воспитатели по отношению к современным детям. При чем тут наука? Не ясно.
Те догмы, которые ты хаваешь, и которые ты никогда не проверял - это церковные заповеди, которые ты почерпнул из библий, может быть из слов жрецов, или при посещении храмов. При чем тут наука? Ну вот собственно это она и есть.
Помимо верифицируемых явлений, которые доступны для воспроизведения любой макаке, есть неверифицируемые (доступные только мне или недоступные в силу отсутствия технолоигй) или практически неверифицируемые вещи (то есть я не могу построить коллайдер и столкнуть в нем частицы, потому что я нищеброд). Так вот последних двух намного больше, чем первы, и абсолютное большинство знаний ты получил ненаучным путем. Но кроме этого есть знания, которые не дают даже в храме науки, потому что ученые, для того чтобы им дали деньги на коллайдеры, должны лизать жопу власть имущим, а для этого нужно игнорировать ряд фактов, в том числе тот факт, что существуют нематериальные сущности и явления, которые нельзя пощупать, но которые оказывают решающее значение на устрой цивилизации.
И теперь ты мне говоришь "что же если не наука дает нам понимание". Ты ебанутый? Для абсолютного большинства людей наука не несет никакого понимания - она несет только кучу мусора, типа понимания айтембилдов на героях дота 2 или теорем веерштрасса.
Аноним 28/08/15 Птн 02:18:14 #59 №294258 
>>294254
Выбираешь область науки, ебешься в универе пару лет, выбираешь научника, ебешься еще как минимум лет 5-6 ... вот ты уже и ученый, хуй дроченный.
Аноним 28/08/15 Птн 02:19:31 #60 №294259 
>>294257
>Для абсолютного большинства людей
И это печально.
Аноним 28/08/15 Птн 02:21:08 #61 №294260 
>>294254
Статус ученого выдается теми же, кто создал этот самый статус ученого. То есть это обычно власть имущие, и хуесосы лижущие власть имущим жопу.
Я ни в коем случае не отрицаю, что среди честных людей есть ученые, их даже стремятся наградить званиями, чтобы оправдать весь этот институт. Примерно так же нобилевские премии в основном выданы заслужено, но значительный процент премий, особенно за мир и экономику, - это тупо пропаганда и политика.
Аноним 28/08/15 Птн 02:26:27 #62 №294262 
>>294257
Две хрустальные сферы эфира этому просвещенному. Согласен по всем пунктам. Пока сам не проведешь эксперимент - уверенности быть не может. Пиздят все, особенно в наше время, когда большинство ученых либо неучи, либо продажны.
Аноним 28/08/15 Птн 02:29:05 #63 №294263 
>>294260
Сам нобель кстати был против премий за экономику, если не ошибаюсь.

>>294256
Зачем? Мне ИНТЕРЕСНО.

Но я не хочу ознакамливацца с матаном. Что делать?
Аноним 28/08/15 Птн 02:40:09 #64 №294264 
>>294263
С чем конкретно в астрофизике ты желаешь ознакомиться? Там особо и не с чем ознакамливаться, чем дальше в лес - тем больше голых теорий, а где теории по астрофизике - там оч много матана.
Телескопов космических у тебя нету, наверное даже простого нету, и матан ты не желаешь учить - что ты тогда хочешь в астрофизике познать? Карту созвездий?
Аноним 28/08/15 Птн 02:55:12 #65 №294269 
>>294263
Дальше научпопа не уйдешь.
Аноним 28/08/15 Птн 02:55:34 #66 №294270 
Почему море солёное?

Где была бы вся эта соль и в какой форме (где металлы и где хлор) если бы жидкой воды на земле никогда не было?

Есть ли на земле источники каменной соли никак не связанные с морской водой ни в один момент своей геологической истории? Как они выглядят? Как кристаллы? Жилы "руды"? Карманы песка? Сплошной солевой "камень"?
Аноним 28/08/15 Птн 03:28:55 #67 №294274 
>>294270
Соль была бы везде. Изначально она разбросана довольно хаотично, но из-за своей хорошей растворимости вымывается при контакте с водой - отсюда скапливание в морях.
Источники, не связанные с морями, есть, но они нахуй никому не нужны. Стоимость соли квалификации "чистая" - 10 амероцентов, стоимость добычи из говеных руд - 10 амеродоларов. Идея ясна?
Аноним 28/08/15 Птн 03:34:07 #68 №294277 
>>294264
спасибо за ответ. А где без конкретно матана легко?
Аноним 28/08/15 Птн 03:37:09 #69 №294278 
>>294277
Легко картошкой торговать, морковкой. В офисе сидеть и менеджером работать легко. Ну, в общем. много есть еще разных мест, где можно без матана.
Аноним 28/08/15 Птн 03:40:49 #70 №294279 
Шок, карта реликтового излучения напоминает очертания материков земли cdn.postnauka.netdna-cdn.com/img/2015/04/sazhin09.jpg
Аноним 28/08/15 Птн 03:41:20 #71 №294280 
>>294278
Я про науку, бро. Не гунди.
Аноним 28/08/15 Птн 04:16:37 #72 №294281 
>>294280
в гуманитарщине всякой, в любой другой области соснешь хуйца
Аноним 28/08/15 Птн 05:58:12 #73 №294283 
>>294281
Да ладно. В той же клеточной биологии матана реально не треба (да и в куче других областей биологии). Ну то есть его читают на первом курсе, но в реальной жизни в большинстве случаев необходимости в нём нет. Я не говорю, что он там вообще не применяется, но в огромном количестве реальных работ никуда за пределы 11го класса не выходят (ну то есть там сколько-то матстата ещё есть, конечно).
Аноним 28/08/15 Птн 06:38:53 #74 №294284 
>>294058
Что можешь сказать о зарождении дерьмодемона в канализации, в которую ты постоянно спускаешь свою фапчу?>>294283
Потому и стагнация в этой сфере. Биологи "слепы" к закономерностям сложнее нормального распределения или среднему по популяции.
Аноним 28/08/15 Птн 06:45:45 #75 №294287 
>>294284
помню нам травили байку, что какие-то биологи опубликовали статью в которой по сути переоткрыли формулу прямоугольников для интегрирования.
Аноним 28/08/15 Птн 06:58:17 #76 №294288 
>>294284
Есть деятели, которые вполне окей математику туда применяют, есть обалсти, которые без крутой математики вообще плохо живут, но она вот реально не всегда и не везде нужна. И потом, откуда вообще представление о стагнации?
Аноним 28/08/15 Птн 07:25:01 #77 №294290 
>>294260
Приведи примеры чисто политических, на твой взгляд, премий по экономике, плз. Я бы в этом ключе скорее литературу вспомнил.

>>294263
Во времена Нобеля экономики как науки ещё по большому счету просто не было.
Аноним 28/08/15 Птн 07:29:05 #78 №294292 
>>294290
Она и сейчас не наука, а модели экономики не работают.
Аноним 28/08/15 Птн 07:51:19 #79 №294293 
>>294292
Они пока что только в вакууме работают, проблемы с предсказанием действий людей, если бы люди были рациональны то всё работало бы. Но люди на ненужную ебату бабосы тратят и хуй предскажешь что вася решит переплатить в 2 раза за вещь того же качества, но от известного бренда или то что люди воообще хуету не нужную покупают.
Аноним 28/08/15 Птн 07:56:15 #80 №294294 
>>294252
>2015
>Не знать математику.
https://youtu.be/E51Kv7UB04U?t=3099
Аноним 28/08/15 Птн 07:57:15 #81 №294295 
>>294294
Сук, время не прилипло. с 51:39
Аноним 28/08/15 Птн 08:09:34 #82 №294296 
>>294293
А экономике (особенно макроэкономике, за оторую нобелевки в основном и дают) такие вещи пофиг. Совершенно неважно, на что именно люди тратят деньги, денежные потоки всё равно будут описываться примерно одинаково в рамках экономики в целом. Опять же, рациональность или нерациональность - понятие растяжимое. Тот же естественный уровень безработицы с одной стороны кажется нерациональным (людям платят зарплату выше, чем казалось бы необходимо для полного использования трудовых ресурсов), с другой - это имеет рациональное объяснение (уменьшается текучка, повышается производительность), и такая штука в моделях, кстати, вполне фигурирует - собственно, если нерациональное поведение предсказуемо, его можно учесть и оно учитывается.
Аноним 28/08/15 Птн 08:38:38 #83 №294299 
>>294284
>Что можешь сказать о зарождении дерьмодемона в канализации

Ай вонt то билив
http://www.youtube.com/watch?v=XR9vLZ8tGiM
Аноним 28/08/15 Птн 08:58:43 #84 №294302 
https://www.youtube.com/watch?v=OvmvxAcT_Yc
Fucking magnets, how do they work, анон?
Я начитался про закон кулона, виртуальные фотоны, прочую хуйню, но так до конца и не понял, как именно один кусок магнита притягивается к другому. Что за виртуальные фотоны? Как они рождаются? Реальны ли они? Какая у них длина волны?
Аноним 28/08/15 Птн 10:34:51 #85 №294308 
>>2942 вы в чём то правы,однако если бы каждый человек собравшийся заняться какой либо из областей науки брался за проверку всех теорем,ставил все эксперименты,и проверял все законы,то жизни бы не хватило на открытие чего то нового,ах да и вы говорили о том что большинство моих знаний получено не научным путём,ну тогда вспомните свои школьные годы,что не парились над доказательством теорем?,что на физике,химии не делали ни одного эксперимента?(я конечно не знаю точно,но похоже это делают все),ну а про >нематериальные сущности и явления, которые нельзя пощупать, но которые оказывают решающее значение на устрой цивилизации.
это вы про всю бозонную материю?
Аноним 28/08/15 Птн 14:14:57 #86 №294337 
>>294294
довольно претенциозное высказывание.

Только вот наш мозг хорошо реагирует на "красоту мира", о которой говорил фейнман и без знания математики.

Симметрия, золотое сечение, гармония и прочее - это мы хорошо чувствуем.

Короче, пизданул ниочём. Особенно учитывая, что конкретно ПРИРОДА не любит ни симметрии, ни размеренности, ни любого другого порядка.

Идеальный круг видел в природе? Квадрат? Пирамиду? Кто-то из птиц может пропиликать что-то уровня моцарта или хотя бы уровня средненькой попсы? Нет.

знаешь, почему? В природе это не нужно.
Аноним 28/08/15 Птн 14:23:08 #87 №294338 
>>294274
КСТАТИ!!!

ПОЧЕМУ РЕКИ НЕ КОНЧАЮТСЯ?

Они же текут круглосуточно, куча воды в день утекает. И...что? Откуда это всё берётся??
Аноним 28/08/15 Птн 14:35:01 #88 №294340 
14407617010270.jpg
>>294338
ты в началке учился???(нет я ни в коем случае не оскорбляю тебя но это как то странно)
Аноним 28/08/15 Птн 14:42:48 #89 №294341 
>>294340
Иии?

Источники:дождь, таяние снегов. Так?

Но ведь река течёт 24 часа в сутки, а дождь идёт редко. 99,999999999999999999999% капель упадют не в реку, и не на берега, откуда они могут в неё стечь.

И всё же, реки не кончаются. Почему?
Аноним 28/08/15 Птн 15:01:13 #90 №294342 
>>294341
вся вода упавшая не в реку впитывается в землю и образует подземные реки впадающие в обычные,ну и осадки выпавшие в высокогорье образуют ледники которые так же питают реку
Аноним 28/08/15 Птн 16:11:13 #91 №294350 
А что начать изучать мне, я гуманитарий, в математику не могу, хотя если начать со школьных книжек, то смогу, но нужно ли мне это, мне интересно разбираться в заболеваниях головы, эта инфа мне дается легко, чем еще я могу себя заебать?
Аноним 28/08/15 Птн 16:34:09 #92 №294354 
>>294350
>мне интересно разбираться в заболеваниях головы

немудрено. У тебя видать у самого...заболевания головы.
Аноним 28/08/15 Птн 16:36:18 #93 №294355 
>>294354
если человек не может нормально сформулировать вопрос это ещё не значит что он больной(нормально говорить не могут лишь все могут только те кто говорят)

Аноним 28/08/15 Птн 17:00:45 #94 №294359 
Пиздец вы тут все умные.
Аноним 28/08/15 Птн 17:16:16 #95 №294363 
14407713767210.jpg
Что это за травка, как называется?
Аноним 28/08/15 Птн 17:52:17 #96 №294368 
>>294363
Портулак вроде.
Аноним 28/08/15 Птн 18:02:54 #97 №294370 
>>294350
> нужно ли мне это
Если такой вопрос вообще стоит, то нет.
Аноним 28/08/15 Птн 18:37:23 #98 №294373 
>>294290
>Приведи примеры чисто политических, на твой взгляд, премий по экономике
Пол Кругман. Человек косит под дурачка, "ну я не знаю почему совок развалился", и при этом награжден такой почетной премией.
Один анон весьма точно описал суть экономики: если ты будешь изучать бозоны, то сколько их не сталкивай, как их не изучай, какие факты не узнавай - бозоны останутся бозонами, и по другому себя вести не будут. Но если ты покажешь, что графики бирж падают, то все поведение игроков рынка изменится, и старая модель уже будет неактуальна.
Модели экономики бесполезны именно потому, что есть конкретные рычаги, которые влияют на поведение участников экономики, то есть, по сути экономика = политика, и вся работа экономики подчиняется велению чьей-то левой пятки.
Экономические теории же строятся не в наблюдении за явлениями, наоборот - экономические теории создают политическое устройство общества. То есть, причина и следствие стоят наоборот. Представьте, что в зависимости от того, как вы опишете движение тела - так тело и будет двигаться. Как это можно называть наукой?
Таким образом, экономика - это искуство манипуляции, это учение об обмане, о манипуляции, и о незнании. Как только некая уловка становится известной - она перестает быть актуальной. Именно так модель работает, пока игроки не знают, что за ней стоит.
В свою очередь политики либеральных стран по всему миру из кожи вон лезут для того, чтобы эту иллюзию поддерживать, они возвышают экономистов, они дают им премии, по всем телевизорам этих экономистов цитируют.
Я дам еще такой простой пример. Вот идет вторая мировая война. Я сталин. Я грю "нам пиздец, немцы превосходят нас во всем, наши армии закрыты в котлах, наши заводы с горем пополам перекочевали на восток, но еще не заработали в полной мере. Берите винтовки, и идите подыхать, пока мы что-то придумаем". То будет? Да разбегутся все и сдадутся в плен. Поэтому, солдатам говорят, что вероломно напали фашики, что осталось еще чуть-чуть, что победа близка, что вот мы деревню какую-то освободили, что бой где-то там с сталинграде идет который месяц. И знаешь что? Войну таки выиграли.
Экономическая теория - это один в один та самая пропаганда. Ты должлен следовать рыночному эгоизму, честным отношениям, свободе рынка, мочить всех, кто препятствует этому рынку, потому что этот диктатор сделает плохо людям, не имеющим доступа к рынкам, и так далее.
Никто из мудил экономистов не мог предсказать кризиса 2008. Нахуй они тогда вообще все нужны? Никто на западе не предскажет кризиса 2018 года, потому что всем таким умникам просто будут затыкать рот.
И напоследок, я расскажу свой личный опыт разговора с экономистом: на вопрос "как может экономика предсказать кризис 2008 года" ответ был "ну он мог произойти, мог не произойти". Вот вся суть экономики. Экономика может пойти так, а может и эдак, может сюда, а может и не сюда. Предсказательная ценность практически нулевая. Нахуй тогда эти модели нужны? Зная законы ньютона я могу предсказать с высокой точностью движение тела, зная законы экономики я могу нихуя.
Аноним 28/08/15 Птн 18:44:08 #99 №294375 
>>294296
>такая штука в моделях, кстати, вполне фигурирует - собственно, если нерациональное поведение предсказуемо, его можно учесть и оно учитывается
Ты можешь что хочешь учитывать - толку от твоего учета никакого. Завтра агенты ГРУ перемочат всех участников игил, и эта организация перестанет поставлять западным странам нефть - цена на нефть возрастет, и ты сука никак этого не можешь учесть до того, как уже будет поздно. Остальное же предсказание уровня "я ткнул ножом в бочину и бросил в подворотне - терпила наверное умрет".
Отсюда понимать политику = понимать экономику. Если ты понимаешь, кто на кого завтра нападет, кто кого завтра замочит - ты можешь предсказывать экономические события. И мат модели тут лишь помогают оценивать реакцию людей, которую ты и должен знать, а потому должен быть еще и кем-то вроде психолога-социолога.
Указанная тобой модель трудоустройства-безработицы характерна лишь для стран свободного рынка труда, не более и не менее и возникла она благодаря длительной пропаганде среди людей.
Аноним 28/08/15 Птн 18:47:35 #100 №294377 
>>294308
>если бы каждый человек собравшийся заняться какой либо из областей науки брался за проверку всех теорем,ставил все эксперименты,и проверял все законы,то жизни бы не хватило на открытие чего то нового
99% людей с ложечки кормят доказательств теорем, а все эксперименты делаются по методичке. Это очень важно делать, чтобы сформировать у ребенка отвращение к самостоятельному познанию.
В том числе здесь сидит очень много мудил, про черные дыры рассуждают, про бессмертия, ну такие вот сорт оф онанизмы, при том, что овер дохуя есть вещей, которые ждут, пока вы начнете их делать и изучать. Как бы двач для того и сделан, чтобы собирать онанистов.
Аноним 28/08/15 Птн 18:56:57 #101 №294378 
>>294377
ну я не могу сказать про других но сам я частенько делал, хоть и не значительные,но всё же эксперименты,ну и в свободное время почему бы не пофилосовствовать на тему вечной жизни и не порассуждать про чёрные дыры(всяко лучше чем социоблядствовать)
Аноним 28/08/15 Птн 19:06:12 #102 №294381 
>>294378
> свободное время
24/7/365
Аноним 28/08/15 Птн 19:23:19 #103 №294386 
>>294381
ну у меня его не так много
Аноним 28/08/15 Птн 19:26:20 #104 №294387 
Почему не проводят нихуя анализов? Можно же заранее формировать списки людей предрасположенных ко всякой хуйне и не знаю, что то делать с этим в зависимости от линии партии.


>Некоторые дети явно более агрессивны, чем другие, и это часто приводит к совершению преступлений: 72% несовершеннолетних преступников (делинквентов) сидят в тюрьме за агрессивные преступления. Делинквенты поразительно часто обнаруживают психические отклонения. Попавшие в тюрьму подростки мужского пола – даже в 90% случаев. Помимо антисоциального поведения, у них часто отмечают злоупотребление наркотиками, наличие психозов и синдром дефицита внимания и гиперактивности (ADHD). Генетические факторы играют здесь важную роль, как показывают наблюдения над близнецами. Речь идет о небольших изменениях в ДНК (полиморфизм) генов, участвующих в производстве и разрушении химических нейротрансмиттеров в мозге. Небольшие генные вариации белков, разрушающие химические нейромедиаторы в мозге, могут приводить к большей агрессивности, алкоголизму и жестокому самоубийству. Сниженная активность химического нейротрансмиттера серотонина сочетается с большей агрессивностью, импульсивностью и антисоциальным поведением. У китайских мужчин в гене, участвующем в выработке серотонина, было найдено небольшое изменение, ответственное за преступления с применением насилия, антисоциальные расстройства личности, зависимость от алкоголя или других веществ. Другое изменение того же белка повышает риск пограничного расстройства личности, с проявлением импульсивности и агрессии. Генетический фон также может впоследствии вносить значительный вклад в наше агрессивное и преступное поведение.
>Во время развития плода среда также влияет на последующую агрессивность. Для мужчин, матери которых во время беременности часто недоедали Голодной зимой 1944‑1945 гг. в Нидерландах, ко времени призыва на военную службу опасность антисоциальных расстройств личности повышалась в 2,5 раза (см. Ш.З). Недостаточное питание плода в матке всё еще встречается в нашем обществе всеобщего благосостояния, но теперь только из‑за плохой работы плаценты. Курение матери во время беременности, в сочетании с генетическим фоном младенца, в девять раз повышает вероятность ADHD у ребенка. И к ADHD добавляются агрессивность и повышенная вероятность войти в столкновение с законом (см. 111.2).
Так же как степень будущей агрессивности в значительной мере устанавливается уже в период нахождения в матке, закладываются и многие другие особенности характера.


>Мы покидаем матку с различной предрасположенностью к агрессивному поведению – в зависимости от нашего пола, генетического фона, количества питания, которое мы получали через плаценту, а также количества никотина, алкоголя и медикаментов, которые употреблялись матерью в период беременности. Опасность проявления нами несдержанного, антисоциального, агрессивного или делинквентного (противоправного) поведения увеличивается в пубертатный период из‑за повышения уровня тестостерона. Существуют значительные различия в агрессивном поведении в зависимости от пола. Мужчины совершают в 5 раз больше убийств, чем женщины. Кроме того, мужчины только в 20% случаев убивают члена семьи или знакомого, тогда как женщины в 60% случаев убивают кого‑либо из тех, кто был им близок. Возрастная картина убийств, совершаемых мужчинами, стереотипна. С ростом уровня тестостерона в пубертатном периоде растет и число убийств. Пик приходится на промежуток от 20 до 24 лет, затем это число снижается до минимума в возрасте 50‑54 лет. Идентичная картина возрастного распределения числа убийств наблюдается в самых различных местах: в Чикаго, в Англии, в Уэльсе, в Канаде. Снижение уровня криминального поведения у мужчин после 20 связано не с падением уровня тестостерона, а с развитием к этому возрасту лобной доли коры больших полушарий, префронтальной коры (рис. 14), которая обуздывает наше импульсивное поведение и способствует моральной оценке наших поступков. Позднее развитие префронтальной коры означает также, что нормы уголовного права для взрослых не должны применяться раньше, чем эта структура мозга достигнет полной зрелости, что происходит в возрасте 23‑25 лет. Политики не считаются с этой моделью развития и, дабы снискать расположение напуганных избирателей, выступают даже за еще более раннее применение уголовного права для взрослых. Алкоголь тормозит функции префронтальной коры, что иногда приводит к внезапным, бессмысленным вспышкам насилия после вечернего времяпрепровождения с выпивкой. Повреждение префронтальной коры в первые годы жизни также может впоследствии приводить к совершению антисоциальных и аморальных поступков.

>Мужской гормон тестостерон стимулирует агрессивное поведение. У одних мужчин уровень тестостерона выше, чем у других, и поэтому их агрессивность гораздо более вероятна. У сидевших в тюрьме за жестокие преступления и изнасилования уровень тестостерона был выше, чем у других заключенных. Такая же связь между высоким уровнем тестостерона и большей агрессивностью была отмечена и у женщин‑заключен‑ных. Уровень тестостерона повышен у заключенных‑мужчин и у новобранцев с антисоциальным поведением.


Аноним 28/08/15 Птн 19:27:51 #105 №294388 
>>294375
Это то же самая, что говорить "физика не работает, вот врежется самолёт в башню и весь сопромат тебе ничего не скажет". Экономика не смогла предсказать кризиса 2008, но в момент его начала она смогла предсказать, как он будет развиваться и позволила не скатить его в новую великую депрессию. До кризиса 2008 года было 30 с лишним лет поразительно ровного роста развитых экономик, как раз за счёт того, что экономисты могли давить кризисы в зародыше. Проблема влияния информации на поведение рынка экономистам тоже замечательно известна, ещё с 70х годов, тот де рост инфляции на инфляционных ожиданиях - явление известное и учитываемое при построении монетарной политики. Модель безработицы, кстати, была ни разу не очевидной вещью - на ней крепко накололся Никсон в своё время, попытавшись через инфляцию снизить безработицу до нуля.
Аноним 28/08/15 Птн 19:29:03 #106 №294389 
>>294387
> С помощью томографии были также обнаружены функциональные различия в гипоталамусе, относящиеся к сексуальной ориентации. Исследователи применяли в своих исследованиях пахучие вещества, феромоны, выделяющиеся через пот и мочу. Феро
моны оказывают влияние на сексуальное поведение, хотя осознанно их запах мы и не воспринимаем. Мужской феромон стимулирует активность гипоталамуса как гетеросексуальных женщин, так и гомосексуальных мужчин, но не вызывает никакой реакции у гетеросексуальных мужчин. Очевидно, последних этот мужской запах не интересует. Позже выяснилось, что у лесбиянок феромоны вызывают другие реакции, чем у гетеросексуальных женщин. Функциональные связи между миндалевидным телом и другими отделами мозга были более обширными у гетеросексуальных женщин и гомосексуальных мужчин, чем у гетеросексуальных мужчин и гомосексуальных женщин. Наблюдения показывают, что функциональные цепи мозга у людей различной сексуальной ориентации действуют по‑разному. Функциональная томография выявила также различия в активности и других областях мозга. У гетеросексуальных мужчин и гомосексуальных женщин таламус и префронтальная кора реагировали сильнее на фотографию женского лица, тогда как у гомосексуальных мужчин и гетеросексуальных женщин эти структуры мозга сильнее реагировали на мужское лицо. Таким образом, наш мозг обладает многими структурными и функциональными различиями в том, что касается нашей сексуальной ориентации, и возникают эти различия уже во время нахождения плода в матке, во втором периоде беременности.


>В первые два месяца беременности формируется пол будущего ребенка. Затем приходит очередь сексуальной ориентации, при этом большую роль играет гормон тестостерон. Мальчики получают его в большем объеме (в определенные периоды даже в десять раз больше), чем девочки. Благодаря этому и выстраивается мозг гетеросексуального мужчины. Если же женщина беременна мальчиком, но в ее организме тестостерона выделяется меньше по тем или иным причинам, есть повод говорить о "гомосексуальном мозге". У девочек ситуация противоположная: переизбыток этого гормона становится предпосылкой проявления у нее в будущем лесбийских чувств. И наконец, менее значительный излишек или недостаток тестостерона может привести к бисексуальности. Иногда - если мать ждет второго сына - выработка тестостерона в ее организме замедляется, поскольку во время первой беременности против него выработался своего рода иммунитет. В этом случае младший брат может оказаться геем. Впрочем, тестостерон - далеко не единственный гормон, оказывающий влияние на формирование сексуальных предпочтений. Внешними же факторами здесь может стать стресс, испытанный во время вынашивания плода, или прием определенных медикаментов.


>Гормоны и другие химические вещееcтва играют важную роль в развитии нашей сексуальной ориентации. У девочек, имевших в период внутриутробного развития высокий уровень тестостерона из‑за синдрома конгенитальной гиперплазии коры надпочечников, большая вероятность би‑ и гомосексуальности. Между 1939 и 1960 гг. в Европе и в США 2 миллиона беременных женщин получали эстрогеноподобное вещество диэтилстилбестрол (DES) с целью предотвращения выкидыша. Хотя DES таким воздействием не обладал, врачи охотно его выписывали, и пациентки были довольны, что получают лечение. DES также повышает вероятность би‑ и гомосексуальности у девочек. Воздействие на плод никотина и амфетамина также повышает вероятность того, что родившаяся дочь может стать лесбиянкой.
>Вероятность гомосексуальности у мальчиков возрастает с ростом числа уже родившихся братьев. Это объясняется защитной реакцией матери во время беременности на маскулинные вещества, которые выделяет ее будущий сын. Защитная реакция усиливается с каждой следующей беременно‑(тью ребенком мужского пола. Стресс во время беременности также увеличивает вероятность появления на свет ребенка гомосексуальной ориентации, из‑за того что стрессовый гормон кортизол матери влияет на вырабатывание сексуальных гормонов плода.
Аноним 28/08/15 Птн 19:49:17 #107 №294392 
>>294337
>Симметрия, золотое сечение, гармония и прочее - это мы хорошо чувствуем.
>чувствуем
Но не осознаём( а многие вообще отрицают что красота может иметь математическое ворожение), а для человека в этом разобравшимся осознание этого даёт + к ощущениям от прикосновения ко всему этому.


Я так понял что имеется в виду то что математика даёт +1 уровень к пониманию красоты и удовольствию её лицезреть. Типа как можно без проблем наслаждаться песнью на иностранном языке, но зная этот язык тебе открывается еще один уровень ощущений. Ну типа ты смотришь на волны в океане и красиво тебе это, а для человека который понимает и осознаёт законы по которым эти волны живут даётся чуть более глубокое ощущение прекрасного.

Аноним 28/08/15 Птн 20:24:47 #108 №294397 
>>294392
>*выражение.
Аноним 28/08/15 Птн 20:31:19 #109 №294400 
>>294387
>Так же как степень будущей агрессивности в значительной мере устанавливается уже в период нахождения в матке, закладываются и многие другие особенности характера.
Делают. Нигеров в США давили и давят. Ставят диагнозы СДВГ интравертам и сажают на спиды. В СССР хачей держали в свинарниках - ЦРУ приказало впустить хачей в хату. Теперь хачи тут в рашке. Правда, недавно вот в москве ввели ограничение для нерезидентов.
>В первые два месяца беременности формируется пол будущего ребенка
Пол формируется примерно так до 22 лет. Хавая гормоны малчик может стать девочкой, а девочка - мальчиком.
>>294388
>физика не работает, вот врежется самолёт в башню и весь сопромат тебе ничего не скажет
Ты совсем тю-тю? Это не то же самое, и как раз когда самолет врежется в заданную точку на заданной скорости - можно будет определить, что случится со зданием. Примерно как произошло в 9/11.
>До кризиса 2008 года было 30 с лишним лет поразительно ровного роста развитых экономик, как раз за счёт того, что экономисты могли давить кризисы в зародыше
Как раз за счет того, что рашка за бесценок продавала нефть и руду на запад за долары. Таким же макаром я могу сказать, что "до кризиса 2008 года было 30 с лишним лет ровного роста, потому что мы молились господу". Каким образом по-твоему экономисты обеспечивали этот рост? Машины делали? Самолеты строили? Если у тебя нету дешевой нефти - ты хотя трижды экономист будешь, но конкурентоспособным ты не станешь.
>Модель безработицы, кстати, была ни разу не очевидной вещью - на ней крепко накололся Никсон в своё время, попытавшись через инфляцию снизить безработицу до нуля
То, что есть дауны, не понимающие псхологию людей - это не секрет. При чем тут экономика? Это политика и социология.
Аноним 28/08/15 Птн 20:33:01 #110 №294401 
14407831819850.jpg
Почему вот у этого лысого пидора есть секс, а у меня нет? Интересует именно научная точка зрения.
Аноним 28/08/15 Птн 21:25:50 #111 №294406 
>>294400
>как раз когда самолет врежется в заданную точку на заданной скорости - можно будет определить, что случится со зданием.
Правильно, так и экономика может определить (с какой-то точностью), что станет с рынком, если цена на нефть взлетит, условно. Или какой-то банк рухнет. Её смысл как бы именно в этом.
>Каким образом по-твоему экономисты обеспечивали этот рост?
Циклические колебания экономики практически никогда не обусловлены изменениями реального производства. Что великая депрессия, что кризис 2008, что азиатский кризис 1998 - это всё заморочки, завязанные на дисбалансы рынков. Соответственно, отсутствие значимых колебаний в период великого успокоения - следствие эффективного регулирования этих самых рынков.
>дауны, не понимающие псхологию людей
Очевидно, в 70е таких было большинство. Аккуратную границу между психологией и экономикой местами вообще провести сложно. В том конкретном случае сыграли сразу два ранее неизвестных фактора - во-первых, как раз неубираемая до нуля безработица (о которой тогда практически никто не догадывался), во-вторых, превентивная реакция рынка на заранее объявленную эмиссию (Никсон тогда попытался снизить безработицу через повышение инфляции, что предлагала кривая Филлипса). Я это всё к тому, что даже довольно старые модели вполне себе поведенческие вещи инкорпорируют, а поведенческая экономика как самостоятельная отрасль на месте не стоит и продолжает развиваться. Да, это смежная с психологией область, но тем не менее она таки считается экономической, и нобелевки уходят и этим людям в том числе.

>Пол формируется примерно так до 22 лет.
Ты пол сейчас как определяешь? Первичные признаки, очевидно, формируются до рождения вместе с основными железами, окончательный гормональный слом происходит в пубертате - а это максимум 15 лет. Потом можно до какой-то степени подействовать на вторичные признаки, но не более того.
sageАноним 28/08/15 Птн 21:40:52 #112 №294407 
>>294401
а тебе не надо
http://www.youtube.com/watch?v=NG7Nj-3hdPk
Аноним 28/08/15 Птн 21:52:42 #113 №294409 
>>294407
Как это не надо? Если я бугурчу по поводу его отсутствия, то логично предположить, что мне надо. И вообще я просил научную точку зрения на это, а ни эти непонятные кукарекания.
Аноним 28/08/15 Птн 22:00:16 #114 №294411 
>>294406
Можно сказать, что рукой управляют биохимические модели, что сокращения мышц вызваны ионными сигналами от нервов, и понимая нервы мы можем моделировать поведение мышц путем анализа сигналов от нервов.
То есть да, когда уже воля была дана - мы можем предсказать последствие этой воли, но какая эта воля будет - ты понятия не имеешь и иметь не можешь.
Я ни в коем случае не спорю с экономикой, как со средством систематизации реакций людей на ту или иную политическую волю в данном общественном укладе, но тут некоторые претендуют на то, что якобы нет никакой воли, есть только рынок, и он все диктует, и только рынок и бывает во вселенной, просто люди не понимали, что оказывается они жили по рыночным понятиям, а тут экономисты открыли глаза.
>Что великая депрессия, что кризис 2008, что азиатский кризис 1998 - это всё заморочки, завязанные на дисбалансы рынков
Кризис 2008 вызван отсутствием роста рынка долара, в результате чего возник избыток валюты и недостаток товаров - долар рухнул, создав кучу проблем тем, кто на него опирался.
Депрессия вызвана отсутствием регулирования рынка, возникают монополии, лебедь, рак и щука, рабочие нищают - капиталист получают всё; та же хуйня была в германии, из-за чего жиды дали деньги гитлеру, чтобы возникло наконец гос регулирование. При чем тут экономика? Суть денег оч проста, поведение людей в зависимости от ситуации на рынке определяется сугубо менталитетом людей, культурой, и работающая для одной нации модель не сработает в другой. И нахуй нам экономика?
Ах и
>Циклические колебания экономики практически никогда не обусловлены изменениями реального производства
действительно так, но я говорил об экономическом благополучии, а не суть скачков. Америке расти позволяла мировая гегемония, а не работа каких-то там экономистов. Америка захватывала рынки сбыта, скупала сырье за бесценок, обнищала целые страны, создала отток специалистов из обнищавших стран в америку - так и получилось благополучие америки.
Если у тебя пиздец по сырью и рынкам, то тебе никакое экономисты не помогут - тебе нужны военные завоевания, или китайцы, которые на собственном горбу вытащат страну. При чем тут экономика?
>поведенческая экономика как самостоятельная отрасль на месте не стоит и продолжает развиваться
Через сто лет на эту так называемую поведенческую экономику будут смотреть как на крепостное право.
Я бы все-таки хотел услышать, что можно было бы назвать экономикой. как наукой, потому что пока что мне это видится как некая слепленная из совершенно разных отраслей система идеологий и наблюдений за обществом, а также набор моделей, которые слабо отражают реально происходящие события, но для которых более свойственна верифицируемость (как для любой чисто умозрительной хуеты).
Собственно, я и говорил, что по сути настоящее управление экономикой заключается в политике и социологии, а не в чисто финансовых процессах. И я говорил, что вся судьба мировой экономики, и все закономерности поведения экономики - все они созданы и регулируются кем-то. Власть раздала жидам капиталы - жиды утащили каждый себе и каким-либо образом вкладывать в общак не желают - тогда власть имущие жиды находят гитлера, ставят его у руля - гитлер пиздит евреев, отбирает имущество и пускает в экономику. Можно сказать, что экономика - это искуство понимания поведения ошейника, на который тебя посадили/ты сел. Соответственно, ведущие экономисты заняты работой проповедника, поясняя людям, как в этом ошейнике нужно крутиться, как из него извлечь максимальную выгоду, и так далее.
sageАноним 28/08/15 Птн 22:05:07 #115 №294413 
>>294409
Для научной точки зрения очевидно недостаточно данных, но самый простой ответ: ты недостаточно прилагаешь усилий к тому чтоб поебаться, ну а то что ты бугуртишь - это лишь показатель, что ты этого хочешь, но вовсе не того, что тебе это надо.
https://www.youtube.com/watch?v=XIX0ZDqDljA
Аноним 28/08/15 Птн 23:48:43 #116 №294425 
>>294411
>нет никакой воли, есть только рынок, и он все диктует
Ну тут ни один вариант в абсолюте не работает. С одной стороны, регулятор (будь то ЦБ или правительство) очевидно имеет какие-то рычаги непосредственного влияния, с другой - против ветра не поссышь, и есть вещи, которые они контролировать напрямую не могут.

>работающая для одной нации модель не сработает в другой. И нахуй нам экономика?
В рамках работоспособности моделей отличия количественные. Ну то есть у разных наций может быть разная норма накопления или разное изменение этой нормы в ответ на какой-то стимул, но такая штука опять же просто включается в модель и всё.

>я говорил об экономическом благополучии, а не суть скачков
А я именно о них. Плюс, конечно, есть и макроэкономика долгосрочного роста, которая тоже моделируется - грубо и упрощённо говоря, если в государстве часть населения копает канавы и зарывает их обратно по какой-то причине - такое государство расти будет херово, и анализ того, что по сути является таким копанием канав - тоже экономический вопрос (что характерно, не зависящий от того, рыночная экономика или нет). Аналогично реакция экономики на вкладываемые ресурсы, которая много от чего зависит.

>Депрессия вызвана отсутствием регулирования рынка, возникают монополии, лебедь, рак и щука, рабочие нищают - капиталист получают всё
Вопрос не вполне тривиальный, там помимо монополизации имело место и принуждение государства к непонижению зарплат во время дефляции, что вынудило бизнесы увольнять сотрудников.

>все закономерности поведения экономики - все они созданы и регулируются кем-то.
Ну вот компьютеры тоже созданы кем-то, но это не мешает существовать computer science, изучающей алгоритмы, а суммарная реакция миллионов независимо действующих людей (ещё и не строго оптимально) - штука посложнее банального процессора. Опять же, если мы сейчас говорим о рыночных экономиках - примеров именно создания государством экономических инструментов единицы, в большинстве случаев фиксируются конструкции, возникшие по принципу "что не запрещено - то разрешено" (будь то векселя с хеджированием, будь то банки с кредитами, будь то акции). К тому же каждым таким инструментом пользуются тысячи и миллионы людей, которые действуют примерно так, как считают нужным, и вопрос, как даже искусственно созданный механизм будет работать во всей этой взаимно связанной системе - вполне себе нетривиальный. Собственно, задача экономики где-то в этом и есть - дать возможность правительству действовать так, что их решение, пройдя через мультиплицирующую цепочку экономических агентов, дало предсказуемый и ожидаемый результат.
Аноним 29/08/15 Суб 02:29:46 #117 №294456 
>>294425
Контроль за деньгами страны фактически означает контроль за вообще всем экономическим благополучием страны, а это значит и политический контроль, потому что ты можешь когда нужно поднять экономику, когда нужно - опустить.
Есть вещи, которые они не могут контролировать - это образ мышления каждого гражданина. Деньги же полностью подвластны им.
>В рамках работоспособности моделей... такая штука опять же просто включается в модель и всё
Да-да, "просто включаются в модель", и похуй что результаты могут быть противоположные из-за этой корректировки. Если по телевизору скажут, что "вот такая ситуация была, это единоразовый провал, такое больш не повторится, мы приняли все меры, у нас есть все ресурсы", то люди будут относится к ситуации спокойно и не ждать никаких кризисов, если скажут по телевизору, что правительство ваще дупля не дает что происходит, министры в отпусках - то сразу пойдет паника, и прочая хуета. В какую переменную ты предлагаешь эту хуйню вписывать, учитывая тот факт, что результаты будут противоположными, и какой толк с твоей экономики, если по сути это искуство уровня бабок-сплетниц.
И, я тебе скажу, именно так работает власть асашай и рашки: разведка собирает инфу о том, что любят, кому лояльны граждане, что за ситуация в стране, какие травмы были у людей, и так далее, потом головастые мужички садятся и смотрят "ага, вот у них лидера замочили, а по корану лидер новый должен прийти - значит мы сделаем нового лидера, который якобы аллахом им послан" - вот такие вот рычаги влияния на экономику. Ты можешь спросить "при чем тут экономика?". у так лидер же их карманный, он им и сделает все. что они прикажут.
>макроэкономика долгосрочного роста, которая тоже моделируется
Долгосрочная экономика может моделироваться только в прошлом. Завтра по твоей стране ебнут ядерными боеприпасами - и грош цена твоей модели.
Есть варианты, когда модели сделаны теми же людьми, которые делали политику - но ты сам понимаешь, что здесь модель работает, потому что ее выполнили как план, а не потому что она нихуя себе отражает суть мира.
Беда всей экономической теории в том, что там очень много если. Если власти продолжат дальше такой курс, если нефть продолжит дешеветь, если люди продолжат ебловать, но власть и некоторые продвинутые ребята имеют источники актуальной информации от людей, и они тебе могут с высокой точностью сказать, как отреагируют люди на то или иное событие, и они могут тебе замутить такой сценарий. который не уложится ни в одну модель, потому что человек настолько гибок, что различным вариациям сценариев нет числа.
>если в государстве часть населения копает канавы и зарывает их обратно по какой-то причине
Я тебе еще раз скажу, что я не спорю, что экономика - это наука уровня "дал 4 рубля - товар стоит 3,50 - сдачи должно быть 50 копеек", я против того, что здесь могут быть какие-то фундаментальные принципы на века. Отклонения есть, но вся модель экономическая - это сплошные поправки на поправках, и что-то может сработать через десять лет, а может и не сработать.
>суммарная реакция миллионов независимо действующих людей - штука посложнее банального процессора
Да, потому что она может быть любой. Любая твоя модель будет моделью прошлого, потому что завтра какую-то хуйню покажут по телевизору - и все миллионы повернутся в другу сторону. Итого ты изучаешь просто популярные нынче поводки и упряжь. Вот те самые "дал 4 рубля - сдача 50 коп". Сегодня это работает - завтра это развалят.
Аноним 29/08/15 Суб 03:54:18 #118 №294468 
Можно ли искуственно соединить водород и 3ислород и получить воду? Если да, то как? Если нет, то почему?
Аноним 29/08/15 Суб 04:48:50 #119 №294476 
>>294468
Что такое Зислород и как его достать?
Если это кислород, то в любом школьном учебнике химии тебе пояснят как и почему. В крайнем случае учебник физик, раздел про горение.
Аноним 29/08/15 Суб 04:54:10 #120 №294477 
>>294468
Горение водорода дает воду. Такие дела, малята.
Аноним 29/08/15 Суб 07:58:55 #121 №294493 
>>294400
>Пол формируется примерно так до 22 лет. Хавая гормоны малчик может стать девочкой, а девочка - мальчиком.
Ты про сисяндры и хуи говоришь, а не про ощущение себя мальчиком или девочкой?
Ты это сам придумал?

>Кажется, нет ничего легче, чем, взглянув на новорождённого, определить, мальчик это или девочка. Уже в момент оплодотворения ясно: две ХХ‑хромосомы – получается девочка, одна Х‑хромосома и одна Y‑хромосома – получается мальчик. Y‑хромосома мальчика запускает процесс, ведущий к выработке мужского гормона тестостерона. Между 6‑й и 12‑й неделями беременности развиваются половые органы плода как мужские или как женские – в зависимости от наличия или отсутствия тестостерона. Позже, во второй половине беременности, мозг дифференцируется в направлении мужского или женского, притом что мальчик производит максимальное количество тестостерона, а девочка – нет. В этот период формируется ощущение себя как мальчика или как девочки, наша гендерная идентичность, до конца нашей жизни закрепляющаяся в структурах нашего мозга.
Тот факт, что гендерная идентичность определяется уже в матке, стал известен не так давно. В 1960‑1980‑х гг. полагали, что ребенок рождается подобный чистой странице и что впоследствии общество ориентирует его в направлении мужского или женского поведения. Эти представления имели серьезные последствия для обращения с новорождённым, пол которого был неясен. Полагали, чтэ если делать операцию вскоре после рождения, то неважно, какой именно пол выбрали для ребенка. Последующее обращение с ребенком приведет к тому, что гендерная идентичность будет соответствовать половым органам. Лишь недавно объединения пациентов указали на то, скольким людям разбили жизнь, сделав им операцию по изменению пола, которая, как выяснилось впоследствии, не соответствовала гендерной идентичности, сложившейся в их мозге еще до рождения. Случай Джон‑Джоан‑Джон ясно показывает, к каким серьезным последствиям может приводить эта концепция. После того как мальчик (Джон) в возрасте восьми месяцев лишился пениса вследствие ужасной ошибки в ходе незначительного хирургического вмешательства (удаление крайней плоти из‑за слишком маленького отверстия мочеиспускательного канала), его решили превратить в девочку Джоан). В возрасте 22 месяцев ему удалили яички, чтобы облегчить превращение в девочку. Ребенка одевали как девочку, психологическое руководство осуществлял в Балтиморе профессор Мани, период полового созревания сопровождался приемом эстрогена. Мани описывал этот случай как огромный успех: ребенок развивался в нормальную девочку (см. эпиграф). Когда я на одном семинаре в США заметил, что это единственный известный мне случай, когда уже после рождения окружающая обстановка смогла изменить гендерную идентичность ребенка, слово взял профессор Милтон Дайемонд, который заявил, что утверждения Мани абсолютно необоснованны. Поскольку он был знаком с Джоан, ему было известно, что она, повзрослев, вновь изменила свой пол на мужской, сделавшись Джоном. Затем Джон женился и усыновил троих детей своей жены. Милтон Дайемонд опубликовал эти сведения. К несчастью, Джон в конце концов потерял свои деньги на бирже, расстался с женой и в 2004 году покончил жизнь самоубийством. Эта печальная история показывает, насколько сильным является программирующее влияние тестостерона на мозг во время пребывания в матке. Удаление пениса и яичек, психологическое воздействие и прием эстрогена в период полового созревания не смогли изменить гендерную идентичность.
Данные о том, что тестостерон действительно ответствен за дифференциацию половых органов и мозга в направлении маскулинности, подтверждаются синдромом андрогенной нечувствительности. Тестостерон, хотя и вырабатывается, но всё тело на него не реагирует. Поэтому как внешние половые органы, так и мозг дифференцируются в феминном направлении. И тогда люди, генетически будучи мужчинами (XY), становятся гетеросексуальными женщинами. В противоположность этому у женского плода, если в матке он подвергается воздействию большой дозы тестостерона из‑за расстройства коры надпочечников (конгенитальная гиперплазия коры надпочечников), клитор вырастает настолько, что в момент регистрации гражданского состояния иногда ребенка записывают как мальчика. Практически все эти девочки относятся к женскому полу. Однако у 2% из них, как впоследствии выясняется, в период нахождения в матке формируется мужская гендерная идентичность.
Аноним 29/08/15 Суб 09:25:57 #122 №294498 
>>294493
>Полагали, чтэ если делать операцию вскоре после рождения, то неважно, какой именно пол выбрали для ребенка. Последующее обращение с ребенком приведет к тому, что гендерная идентичность будет соответствовать половым органам.

Совершенно правильно полагали.

>После того как мальчик (Джон) в возрасте восьми месяцев
Восемь месяцев после рождения. Восемь, Карл! Какой клоун сочинил эту хероту?

>Эта печальная история показывает, насколько сильным является программирующее влияние тестостерона
Не было никакого программирования в матке, было программирование в течение восьми месяцев снаружи неё. Пиздец, просто пиздец.
Аноним 29/08/15 Суб 09:32:40 #123 №294499 
>>294191
Температура пропорциональна квадрату скорости молекул, то есть там дофига энергичных молекул, но так как плотность вещества очень низкая, то особой роли это не играет.
Аноним 29/08/15 Суб 09:43:49 #124 №294500 
>>294341
У реки есть "бассейн", то есть большая площадь, с которой она кормится дождевой водой. Это не только ручейки и речушки, но и грунтовые воды, к примеру (вода после дождя проходит сквозь землю до уровня глин, которые эту воду не пропускают, и там движется куда-то, образуя подземные реки и прочие родники). Площадь бассейна большая, площадь самой реки - не очень, отсюда и много воды.
Аноним 29/08/15 Суб 09:46:33 #125 №294501 
>>294260
Нобелевских премий по экономике не существует. Есть премия банка Швеции им. Нобеля. Пропаганда начинается с этого факта.
Аноним 29/08/15 Суб 09:49:48 #126 №294502 
>>294388
Физика работает - самолеты врезаются в башни очень редко. Кризис 2008 года не решен до сих пор.
Аноним 29/08/15 Суб 11:04:47 #127 №294503 
>>294500
Спасибо!

А откуда вообще начинаются реки?
Аноним 29/08/15 Суб 11:16:40 #128 №294504 
>>294503
Вода стекает сверху вниз, поэтому реки обычно начинаются откуда-то сверху. В этом "верху" будет или какой-нибудь горный ручеек, или родник (то есть выход подземных вод на поверхность)
Аноним 29/08/15 Суб 12:48:15 #129 №294509 
>>294504
или ледник
Аноним 29/08/15 Суб 17:12:06 #130 №294536 
>>294456
>Контроль за деньгами страны фактически означает контроль за вообще всем экономическим благополучием страны, а это значит и политический контроль, потому что ты можешь когда нужно поднять экономику, когда нужно - опустить.
Опустить могут, поднять (только через деньги) - не могут, дополнительная эмиссия (точнее, дополнительное понижение ключевой ставки) инфляцию даст, а роста экономики - не даст (если нет дефицита денег, конечно), что было неоднократно продемонстрировано натурными экспериментами по незнанию. Опять же, ниже ноля ставка не опускается, после этого начинаются всякие нетривиальные танцы с бубном в виде количественного смягчения, отрицательного процента по корсчетам и далее по списку, то есть даже увеличить количество денег в экономике правительство (а точнее - цб) может не всегда.

>В какую переменную ты предлагаешь эту хуйню вписывать
Ну как - в силу действия внешних факторов изменился уровень доверия к экономике, в следствие этого изменился уровень сбережений и что там ещё изменится, ну и дальше идёт цепочка изменений. А что там конкретно говорит телевизор - вне области действия, естественно, такую вещь экономика не может предсказать, она и не про это.

Ну вот ещё пример, наверное, более аккуратный - человек может переохладится, или принять лекарство, или три дня не есть, или что-то ещё (дофига вариантов), и с организмом от этого будет происходить много всего интересного. Физиолог не модет предсказать, когда и при каких обстоятельствах это произойдёт, но он может предсказывать, что с организмом будет твориться после вот этого вот. Вариантов - уйма, какие-то новые варианты действия старых препаратов находят и до сих пор, но это не мешает физиологии быть наукой. Ну вот тут вот то же самое. Ну то есть да, бывает что появляются какие-то новые модели поведения, изобретаются какие-то новые финансовые инструменты (то же самое количественное смягчение или отрицательный процент по корсчетам появились только недавно) - но и новые лекарства, и новые болезни таки возникают, и их изучают. Ну то есть да, экономика - это не квантовая физика (или геология), в которой есть какая-то надежда когда-нибудь её завершить, но таких наук хватает и помимо.
Аноним 29/08/15 Суб 18:54:16 #131 №294551 
14408636564880.jpg
Какой сейчас самый годный, базовый учебник по биологии?
Аноним 29/08/15 Суб 19:16:31 #132 №294554 
>>294368
Оно, спасибо.
Аноним 29/08/15 Суб 19:40:18 #133 №294561 
1) Как воздействуют гравитационные волны на вещество: изменяют вес и силу тяжести посредством изменением g, изменяют линейный размеры и течение времени, создают неоднородное гравитационное поле которое деформирует вещество гравитационным напряженностью или что-нибудь другое?
2) Тот же вопрос только вместо гравитационных электромагнитные, а в место вещества магниты и вещества имеющие заряд(ионы там, наэлектризованые ручки об волосы и т.д.).
3) Абсалютного нуля нельзя досчить так как при абсалютном нуле нет движения(энергии), а вещество без энергии лишено смысла?
4) К какому фундаментальному взамадействию относится сила Архимеда и приливные силы?
5) Гравитационная активная и пассивеая масса отличаются по значениям на любом знаке после запятой?
6) Какие еще меры и параметры, кроме давления, температуры и концентрации вещества если система многокомпонента, делают фазовые превращение первого рода? Ну т.е. изменение агрегатного состояния(из жидкости в газ, из твердого тела в плазму, из плазмы в сверхкритический флюид, из газа в вырожденную материю(охуенно плотное давление, так сказать сверхтвердое тело) и т.д.) и полиморфные преврашения, т.е. изменение кристаллической ячейки(из графита в алмаз, из жидкого кристалла в аморфную жидкость, из стекла в кварц, из кварца в тридимит и т.д.). И можно ли эти новые фазы получить просто изменяя давление, температуру и концентрацию вещества или хуй тебе, это уникальные фазы для которых нужны эти новые меры/параметры отличные от нуля?
Аноним 29/08/15 Суб 19:57:37 #134 №294566 
Почему вещества/материалы не делят по группам от воздействия на них гравитации, гравитомагнетизма, общего электромагнитного поля? По воздействию магнитного поля: парамагнетики, ферромагнетики диамагнетики и другие -магнетики. По воздействию электрического поля: диэлектрики, проводники, полупроводники. А по воздействию электромагнитного поля, гравитации и гравитомагнетизма не делят.
Аноним 29/08/15 Суб 20:10:28 #135 №294568 
>>294536
>роста экономики - не даст , что было неоднократно продемонстрировано натурными экспериментами по незнанию
Ты че, дурак? Те, кто делают деньги, прекрасно представляют, что это такое. Прежде всего это бумага, и ты должен убедить или заставить людей ее ценить.
Ясен хуй, что если эмиссия денег переходит в руки тех, кто тупо гребет себе, не вкладывая обратно в экономику - то получается пиздец экономики. Не в незнании дело - это ты не знаешь что они знают, ведь они-то прекрасно все знают, и нет тут никаких секретов и науки большой.
А есть реальные товары и ценности, но это уже совсем другая история.
То, что ты упускаешь, потому что хуево разбираешься в политике - это что можно держать экономику на том же уровне, при этом получая поддержку людей, при этом ставя нужных тебе людей у власти - и все только благодаря тому, что ты можешь в нужный момент эмиссию остановиться, а в другой нужный момент деньги напечатать. Или остановить регулирование курса валюты, и начнется пиздец рашкинский. Повысил ставку по процентам - развитие остановилось, понизил - развитие пошло. И так можно сидеть обворовывать людей оч долго.
Я тебе еще раз скажу, что здесь речь идет даже не столько про экономическое благополучие - а про власть и лояльность граждан. Вон, в украине пиздец по экономике, а люди на майдан не поднимаются почему-то. Потому что в экономике 70% явлений - это неформализуемая хуйня, которая сейчас произошла, а потом может и не произойти. Сегодня я обворовал, завтра я попытался, но мне наваляли пизды. Вот такая вот экономика.
>что там конкретно говорит телевизор - вне области действия, естественно, такую вещь экономика не может предсказать, она и не про это
Да нихуя она не может. Если ты понимаешь психологию людей, поведение социума, политическую ситуацию - ты можешь предсказывать экономику имея ученую степень бабули-сплетницы. Все дело в том, то 98% нихуя не смыслят в этих вещах, потому их понимание сводится к извлечению выгоды из поводка, на котором они сидят. Жиды и прочие власть имущие стремятся делать вид, что этот поводок на века, что он самый оптимальный и самый выгодный, но это просто наебка, и вся экономика - это наука о том. как правильно быть наебанным и обворованным, и соответственно всякие торговцы на бирже и спекулянты должны быть в курсе того, как народ любит быть обманутым, отталкиваясь от этого ты можешь строить свои планы по извлечению выгоды.
Вот если одно, и если второе, и если третье - тогда мы можем сказать, что экономика пойдет по такому-то пути. Но этих "если" настолько много, что эти прогнозы ценности не имеют. И именно жиды сделали так, что экономика стала предсказуема, но не потому, что они увидели механизм и прислушались к биению его сердца, а потому что они компенсируют любые отклонения и пытаются сделать вид, что законы экономики работают. И власти многих стран делают примерно то же, показательно устраивая контролируемые колебания валют.
Ты думаешь почему в рашке такой пиздец был с рублем? Потому что ЦБ действовал по старым методам, разработанным госдепом, и потому что не было специалистов, которые брали бы ответственность за свои действия, потому что страну много-много лет превращали в сырьевой придаток запада, а не в том, что кто-то что-то не знал. Как раз власть прекрасно понимала, что в любой момент нефть могут обрушить, отсюда наебнется рубль.
>Физиолог не может предсказать... это не мешает физиологии быть наукой
Это не мешает физиологии быть наукой, но это помогает физиологам быть мудаками. Эфект плацебо и прочие хуево формализируемые явления играет решающую роль в выздоровлении. Там же где лекарства четко действуют, и двойной слепой метод действие подтверждает - можно делать точные предсказания.
Именно в той области медицины, где пациент лежит без сознания, и только врачи разными действиями меняют его состояние - именно там медицина дает наиболее точные оценки, может сказать сколько человек проживет или через сколько выздоровеет. Но когда человек в сознании - тогда вступают в игру другие факторы, и многие вещи становятся малопредсказуемыми.
Именно это и происходит в экономике - участники экономики более-менее в сознании и активно влияют на ситуацию, отсюда возникает принципиальная непредсказуемость процессов.
Психология - это вообще не наука, например. Вот она наиболее близко расположена к экономике, в которой точно так же плохо верифицируемые и плохо воспроизводимые явления иногда ложатся в закономерность, а иногда - нет.
Аноним 29/08/15 Суб 20:29:16 #136 №294570 
Полностью формальный, технический, мертвый подход убил систему правосудия и убил вашу науку. Что собираетесь делать, господа мочёные? Когда поймете, что такой подход хорош только для написания компьютерных программ?
sageАноним 29/08/15 Суб 20:29:42 #137 №294571 
>>294561
Во-первых, научись писать слово абсолютный. Во-вторых, если бы абсолютный нуль был достижим, это было бы эквивалентно тому, что мы можем точно знать "где" находятся электроны(частицы), что противоречит неравенствам Гейзенберга.
Аноним 29/08/15 Суб 21:27:32 #138 №294580 
>>294568
>Прежде всего это бумага
>эмиссия денег переходит в руки тех, кто тупо гребет себе, не вкладывая обратно в экономику
Во-первых, "бумага" - это по отношению к деньгам тыщу лет как вторичная хуйня. Во-вторых, правительство контролирует эмиссию до какого-то предела, но не полностью (в большинстве стран, включая Россию) - понизить её (повысив процент) оно может, а повысить - только до какого-то предела, которого не всегда хватает. Опять же, когда ты говоришь про "регулирование курса" - оно делается продажей валютных резервов же. Это к вопросу о том, что ЦБ действовал неправильно - а как, по-твоему, он должен был действовать? Не, по поводу придатка я, естественно, согласен, но это не имеет никакого отношения к ЦБ - это при любом раскладе вне его компетенции.

В очередной раз - конечно, экономика не может однозначно говорить, где всё окажется через сто лет, чего ты, похоже, от неё хочешь, чтобы считать наукой. Естественно, её предсказания не дотягивают даже не биологических, не говоря уже о физических (а физику, ты, похоже, считаешь единственной наукой, у которой есть право на существование, раз физиологов называешь мудаками, что приводит меня в определённый ступор). Но у неё в рамках своей области действия вполне адекватная методология, и я не понимаю, схуя ли она неверифицируемая - если произошедшее не объясняется моделью - окей, вводим в модель новый фактор, всё как положено. Да, достаточно часто вылезают какие-то факторы, которые были непредвидимыми, но тут уж извините, такой предмет, а влияние этих факторов в большинстве своём в принципе описано. Плохая воспроизводимость - так ты физически не можешь поставить эксперимент в контролируемой среде, но такие среды тоже как-то нужно изучать по возможности формальными методами.

Даже если ты возьмёшь пациента в коме, ты не дашь стопроцентного прогноза действия лекарств просто в силу неполноты информации. Более того, даже на молекулярном уровне речь не идёт о стопроцентной предсказуемости, вот серьёзно.

Но вообще пошло ВРЁТИ!! в каких-то запредельных масштабах и теории заговора, конечно, а в их рамках серьёзно что-то обсуждать просто бессмысленно - как раз они и являются каким-то совершенно крайним признаком полнейшего пиздежа.
Аноним 29/08/15 Суб 21:28:43 #139 №294581 
14408729232970.jpg
Кто-то знает про Сколтех? Каков шанс поступить? И стоит ли?
Подумываю попробовать набижать, но в противовес есть возможность получить образование в Georgia Tech, обучение дистанционное но полноценное с норм дипломом и всем таким. И всё таки один из топвузов по CS, но в сколтехе есть общение и тусовка, а это очень важно для успехов и связей.
Аноним 29/08/15 Суб 21:42:29 #140 №294584 
>>294581
Сколтех - говно. Учится там один чорт из нашей группы. Полный обмудок, распрашивал его что и как - обычная программа ни о чем, только с пафосом.
Аноним 29/08/15 Суб 22:14:29 #141 №294592 
>>294581
В принципе неважно. Лишь бы ты не занимал место в нормальном вузе.
Аноним 29/08/15 Суб 22:30:08 #142 №294594 
>>294584
Короче, уровень иннополиса видать.

>294581
>В принципе неважно. Лишь бы ты не занимал место в нормальном вузе.

Ну да, поэтому ориентируюсь на топовые. Только даун аутист стал бы учится в мгу-вышке-мфти.
Аноним 29/08/15 Суб 22:41:35 #143 №294596 
>>294594
>Ну да, поэтому ориентируюсь на топовые. Только даун аутист стал бы учится в мгу-вышке-мфти.

Молодец.
Аноним 29/08/15 Суб 22:42:34 #144 №294597 
>>294594
Что не так с МГУ и МФТИ?
Аноним 29/08/15 Суб 23:52:25 #145 №294607 
>>294580
>экономика не может однозначно говорить, где всё окажется через сто лет
Экономика мне не может сказать, что будет завтра. Потому что завтра будет какая-то новая хуйня в политике, и все перевернется верх ногами. Это как русская рулетка - пять из шести раз теория срабатывает.
Именно на этом играют эмитенты долара - курс его держится строго стабильным. Не потому что он не может колебаться, а потому что его руками держат на конкретной цифре. Ровно как и цена на нефть сейчас понижена искуственно, как теневыми манипуляциями, так и контролем за крупнейшими экспортерами нефти.
Повышение курса валюты, в общем случае, делается скупкой валюты, то есть процесс обратный эмиссии.
То, что само правительство ограничено в своих действиях - это в том числе следствие многолетнего процесса по уничтожению рашки.
Правильный сценарий - это быстрый кризис с падением валюты, который сменяется периодом покоя. В рашке смена курса затянулась на месяцы, сделав проблем для рашкинского бизнеса.
Вся сложность работы какого-нибудь министра или подобного хуесоса заключается не в том, что экономика - это сложная штука, а в том, что у тебя нет практически никаких четких формальных критериев, ты должен быть всесторонне образован/информирован.
Я химик и программист.
>если произошедшее не объясняется моделью - окей, вводим в модель новый фактор
Вот. Количество новых факторов бесконечно. Но задача жидов и прочих создающих экономические системы заключается в том, чтобы устранить эти факторы, и сделать типа честную открытую систему, и убедить других людей, что она самая правильная и в нее нужно вкладывать деньги (ресурсы/имущество/похуйчто).
Аноним 30/08/15 Вск 02:57:56 #146 №294639 
>>294570
Два чая.
Аноним 30/08/15 Вск 03:43:21 #147 №294642 
14408954013780.png
>>294570
Кек, дрочеры оснований убили математику? Оттого что кто-то дрочит основания науке хуже не становится.
Аноним 30/08/15 Вск 10:16:18 #148 №294660 
14409189785100.png
Что можешь сказать про пик?
Аноним 30/08/15 Вск 12:05:07 #149 №294668 
>>294660
Могу сказать про физику и электронику. Книги в столбце эентри левел адекватны заголовку. Хоровиц и Ландавшиц это категория про. После Фейнмана браться за терфизику не стоит.
Аноним 30/08/15 Вск 13:10:55 #150 №294674 
>>294660
Толстота.
Где ты нарыл этот тролльский пик?
Аноним 30/08/15 Вск 13:17:22 #151 №294675 
Пидор
Аноним 30/08/15 Вск 15:17:18 #152 №294682 
>>294660
>Inter-universal Teichmüller theory - GOD LEVEL
Аноним 30/08/15 Вск 16:09:15 #153 №294695 
Ну что там, ген гомосексуализма нашли уже?
Аноним 30/08/15 Вск 16:13:03 #154 №294698 
Сап аноны. У меня всегда был вопрос. Возможно ли собрать подобие компьютера из мусора? Я не говорю о компьютере который по мощности будет равен современным, я говорю о таком который будет хотя бы как калькулятор.
Аноним 30/08/15 Вск 16:14:27 #155 №294699 
>>294668
>После Фейнмана браться за терфизику не стоит.
Там все объяснено? Намного не допёр.
Аноним 30/08/15 Вск 16:15:09 #156 №294700 
>>294698
Разве что счёты.
Аноним 30/08/15 Вск 16:40:16 #157 №294702 
>>294668
>Ландавшиц категория про
Пиздец...
Аноним 30/08/15 Вск 16:46:29 #158 №294703 
>>294698
Что значит "из мусора"? Нужна возможность собрать триггер (выключатель) который можно переключать с помощью других таких-же триггеров, соотвественно также нужна возможность передавать работу которая переключит тригер. С этого можно собрать что угодно. Там не знаю, может еще аналог диодов нужен, тяжело сразу сказать, возможно использование диодов связано только с тем что для передачи работы используется электричество, и если использовать, к примеру, пар то нахуй надо.
Аноним 30/08/15 Вск 17:00:09 #159 №294705 
>>294703
Спасибо за ответ. Умнее наверно было бы мне вместо мусора написать легкодоступные детали.
Также давно я слышал про создание примитивного компьютера из транзисторов. Возможно ли это?
Ну и если деталями для примитивного компьютера будет детали используемые в электроники тогда электричество понадобиться. Мне та все равно какой вид энергии будет использоваться.
Просто у меня появилось желание создать подобие компьютера самому. Если ты или другие аноны знаете какие нибудь сайты в которых есть описание по созданию примитивного компьютера, можете дать ссылки.
Аноним 30/08/15 Вск 17:15:03 #160 №294707 
>>294705
> Также давно я слышал про создание примитивного компьютера из транзисторов
Так компьютер из транзисторов и состоит. Транзисторы, диоды и генератор тактовой частоты. Ну и вспомогательные там резисторы, конденсаторы и т.п. Более сложная хуйня только в перефирийных устройствах жестких дисках, мониторе и т.д.
Почитай Ч.Петцольд - Код.
Аноним 30/08/15 Вск 17:26:49 #161 №294709 
>>294705
Легче raspberry pi купить.
Аноним 30/08/15 Вск 17:31:23 #162 №294710 
>>294705
На хабре поцык делал микросхемы самостоятельно, можешь поинтересоваться.
Как ты наверное помнишь, транзисторные компутеры были размером со шкаф и стоили в изготовлении овер дохуя. Сейчас, уже есть технологии печати любой схемы чуть ли не на принтере, и тем более если тебе нужна простая схема - сделать ручками ее под силу даже простому смертному.
Хотя я не понимаю в чем проблема, если простые микроконтроллеры были легкодоступны простым людям уже в совке.
Аноним 30/08/15 Вск 17:39:07 #163 №294714 
>>294709
Ему наверное хочется самому поконструировать, тут больше PLD или FPGA какой-то подойдет.
Аноним 30/08/15 Вск 17:44:27 #164 №294717 
14409458673830.png
Какие есть годные книги по эволюции?
Аноним 30/08/15 Вск 18:17:52 #165 №294721 
>>294710
> Имплаинг сейчас процессоры не на транзисторах
Аноним 30/08/15 Вск 20:17:44 #166 №294746 
Не занимался физикой со школы, хочу ботать.
Три тома Ландсберга подойдут?
Аноним 30/08/15 Вск 22:41:07 #167 №294763 
14409636677950.jpg
Слушай, вот биосинтез белка в клетке, как блять это всё происходит? как какие то молекулы оказываются в нужное время в нужном месте, что бы начать его и произвести, они же блять просто молекулы, что влияет на их "поведение"?? как они вообще могут
регулярно и достаточно точно что бы клетка не ломалась всё это делать? Потом аминокислоты доставляются к рибосоме которая на основания информации с рнк(которую сука как специально выкинуло из ядра дальше в цитоплазму, почему её выкинуло, кто это сделал, блять, там что типа она всплывает что ли как говно в проруби?) их лепит в строгой последовательности, как ебать, зачем, это происходит они же сука куски материи. Они же там просто плавают, как они могут со средней скоростью в 1 секунду всё это делать на протяжении десятков лет? Что это за хуйня, двач?
Нахуй так жить?
Аноним 30/08/15 Вск 22:44:45 #168 №294764 
>>294763
>Они же там просто плавают
На микроуровне ничего просто не плавает. Всё плавает сложно, как шестерёнка в будильнике.
Аноним 30/08/15 Вск 22:50:13 #169 №294767 
>>294702
Лол, маня, это вполне профессиональная литература независимо от того насколько лёгким тебе кажется материал.
Аноним 30/08/15 Вск 23:53:40 #170 №294778 
Реквестирую литературу где подробно разбирается вопрос взаимосвязи восприятия времени со скоростью процессов внутри мозга человека.
Аноним 31/08/15 Пнд 00:01:56 #171 №294781 
Анон!
Если мкс ОСТАНОВИТЬ, то космонавты смогут ходить нормально по полу, верно?
Аноним 31/08/15 Пнд 00:05:38 #172 №294782 
>>294764
Что-то плавает, что-то нет. РНК из ядка активно выкидывается (погугли nuclear pore и охуей, как она сложно устроена), а до рибосомы свободно плывёт. Аминокислоты в ринципе свободно плавают и связываются с тРНК, синтез белка в самой рибосоме завязан на взаимодействие тРНК и рибосомы (в первую очередь - рибосомной РНК). Размеры клетки же мизерные, скоростей диффузии вполне хватает, регуляция происходит либо на границе компартментов (пустить-не пустить, как с выходом РНК из ядра или попатанием белка в ЭР, или дать свящаться/не дать связаться), либо на уровне везикулярного транспорта, но это уже нужно чтоб первым способом создался пузырёк, заполненный продуктом, который уже можно маршрутизировать как вагон по рельсам.
Аноним 31/08/15 Пнд 00:07:01 #173 №294783 
>>294781
Если МКС остановить, она начнёт падать и никто ходить не сможет. Если предотвратить и это (сжигая ёбанные тонны топлива в секунду) - то да. Только нахуя?
Аноним 31/08/15 Пнд 02:31:44 #174 №294801 
>>294778
>литературу где подробно разбирается вопрос взаимосвязи восприятия времени
Предлагаю на эту тему прочитать книгу тимоти лири и уебать марочку. Что я сам и сделал, собсна.
Аноним 31/08/15 Пнд 03:21:02 #175 №294804 
>>294801
Уебать бы тебе по морде.
Аноним 31/08/15 Пнд 12:38:56 #176 №294844 
>>294030 (OP)
Нарисуйте мне схему с npn и pnp транзистором, с направлением тока, когда транзистор проводит ток. Что бы было показано куда "текут" электроны в базу или из базы, в эмиттер или из эмиттера и т. д.
Аноним 31/08/15 Пнд 15:17:18 #177 №294861 
>>294844
Фуррь-электронщик в треде.

Электроны текут из минуса в плюс. У npn плюс на коллекторе, у pnp - на эмиттере.

Основной ток течёт по цепи эмиттер-коллектор, через базу - только управляющий ток.

Рисовать сейчас неохота, припрёт тебе - нарисую.
Аноним 31/08/15 Пнд 16:05:41 #178 №294872 
Ананасы, будьте добры. Помогите разобраться в восприятии или ощущении скорости. Вот на примере карусели. Наблюдая со стороны кажется что она медленно вращается. Катаясь же на ней, кажется, что она вращается быстрее, чем когда наблюдал.
Аноним 31/08/15 Пнд 16:11:32 #179 №294874 
Работает ли третий закон ньютона для электромагнитных полей? Например с помощью определеной формы внешнего магнитного поля можно задать форму плазме, а если изменить форму плазмы в этом внешнем магнитном поле с помощью других сил(гравитации там) измениться ли форма внешнего магнитного поля?
Магнитные потоки это реальность(виртвальные фотоны или другие частицы) или абстракция(как линии магнитных полей)?
При повышение напряженности электрического поля даже диэлектри начинают проводить ток(молния бьет через воздух, который диэлектрик), т.е. становиться проводник, а к чему будут стремиться вещества при повыщннии индуктивности магнитного поля, к парамагнетикам, диамагнетикам или ферромагнетикам?
Аноним 31/08/15 Пнд 16:13:54 #180 №294875 
>>294874
>>а если изменить форму плазмы в этом внешнем магнитном поле с помощью других сил(гравитации там) измениться ли форма внешнего магнитного поля?

Да. Собственно причина солнечной активности - плазма в разных слоях движется с разной скоростью и магнитное поле "наматывается" на ось Солнца.

За остальное - не отвечу.

Фуррь-электронщик-кун
Аноним 31/08/15 Пнд 16:21:04 #181 №294877 
>>294874
http://femto.com.ua/articles/part_1/0516.html - собственно вот.

Фуррь-электронщик-кун
Аноним 31/08/15 Пнд 17:05:15 #182 №294888 
>>294872
Когда ты смотришь на карусель, за то время, пока она делает полоборота точка на карусели смещается в поле зрения на угол, который занимает в нём карусель (довольно небольшой). Когда ты на карусели сидишь, за время полуоборота неподвижные точки пролетают весь угол зрения. Эффект тот же, что с самолётом - ты не видишь его скорость, потому что его угловая скорость очень маленькая (ну, или если ты в самолёте, ты точно так же не чувствуешь скорость, смотря на землю).
Аноним 31/08/15 Пнд 17:25:54 #183 №294891 
>>294888
Спасибо.
Аноним 31/08/15 Пнд 21:57:31 #184 №294945 
Сможет ли жизнь победить энтропию или всему в конечном итоге пиздец?
Аноним 01/09/15 Втр 00:12:59 #185 №294958 
>>294945
Жизнь локально понижает энтропию за счет энергии из другого места, где энтропия растет сильно. В частности, за счет термоядерного котла под боком. Поэтому в плане термодинамики - пиздец, да. Хотя с точки зрения современной космологии пиздец будет выглядеть по-другому, типа большого разрыва.
Аноним 01/09/15 Втр 08:45:55 #186 №294989 
>>294958
Это всего лишь одна (маргинальная) теория. Не пудри людям мозги.
Аноним 01/09/15 Втр 09:03:24 #187 №294990 
>>294945
Существует ли энтропия на самом деле или это всего лишь коэффициент для подгонки кривой теории под ответ?
Аноним 01/09/15 Втр 09:06:57 #188 №294992 
>>294989
Ты о чем именно?
Аноним 01/09/15 Втр 09:34:29 #189 №294998 
>>294990
Энтропия - это мера состояния системы, обобщающая элементарные события сисиемы (события изменения состояний элементов системы) до макросостояний системы. Причём, мера эта относительная, определены способы измерения изменения энтропии, но не её абсолютной величины. (Как и абсолютная величина энергии системы на самом деле не абсолютна, определена относительно допустимых текущей физической теорией способов взаимодействия с системой, относительно наших знаний и умений эту энергию извлекать.)
Аноним 01/09/15 Втр 09:38:47 #190 №294999 
>>294998
Мы вообще не можем говорить об абсолютных величинах чего бы то ни было. Любой внешний сигнал для мозга так или иначе измеряет дельту (в пределе - дельту изменения ионизации нейромедиаторных каналов). Для мышления базовым физическим принципом является именно относительность всего и вся, виртуальность абсолюта.
Аноним 01/09/15 Втр 09:55:38 #191 №295000 
>>294999
>виртуальность абсолюта.
Вот и выросло поколение посмотревших матрицу.
Аноним 01/09/15 Втр 10:02:45 #192 №295002 
>>295000
Поколение, читавшее Лема, Матрицей уже не впечатлялось, мой юный любитель кинематографа.
Аноним 01/09/15 Втр 10:04:33 #193 №295004 
>>295000
Вот и выросло поколение, не читавшее Платона и не понимающего фундаментальных вопросов научного метода.
Аноним 01/09/15 Втр 10:05:01 #194 №295005 
>>295004
*не понимающее
Аноним 01/09/15 Втр 10:09:16 #195 №295006 
>>295002
Но ведь худ фильм > худ книги. Еле дочитал саный солярис, последняя моя попытка привить себе любовь к худ литературе была, дочитал только по тому что начал, ну и потому что всё таки надо расширять кругозор и даже через ни хочу касаться культовых произведений культуры чисто что бы знать, но это было последней каплей.
Аноним 01/09/15 Втр 10:18:08 #196 №295007 
>>295006
Книга может дать только информацию нагрузить чисто думательный аппарат, а кино приносит эстетическое удовольствие музыка, картинка. Книга как чисто развлекательный контент устарела.
А вместо того что бы 100 книг прочитать про одни и те же человеческие переживания и отношения, лучше прочитать пару тройку хотя бы науч поп книг по нейробиологии, работе мозга, и прочих попыток научным методом объяснить поведение человека .
Аноним 01/09/15 Втр 10:30:17 #197 №295010 
>>295007
Хотя у меня может вооброжаловка не работает, но я просто не могу представить себе то что пытается описать автор.

>Густой аромат роз наполнял мастерскую художника, а когда в саду
поднимался летний ветерок, он, влетая в открытую дверь, приносил с собой то
пьянящий запах сирени, то нежное благоухание алых цветов боярышника.
С покрытого персидскими чепраками дивана, на котором лежал лорд Генри
Уоттон, куря, как всегда, одну за другой бесчисленные папиросы, был виден
только куст ракитника -- его золотые и душистые, как мед, цветы жарко пылали
на солнце, а трепещущие ветви, казалось, едва выдерживали тяжесть этого
сверкающего великолепия; по временам на длинных шелковых занавесях
громадного окна мелькали причудливые тени пролетавших мимо птиц, создавая на
миг подобие японских рисунков, -- и тогда лорд Генри думал о желтолицых
художниках далекого Токио, стремившихся передать движение и порыв средствами
искусства, по природе своей статичного. Сердитое жужжание пчел,
пробиравшихся в нескошенной высокой траве или однообразно и настойчиво
круживших над осыпанной золотой пылью кудрявой жимолостью, казалось, делало
тишину еще более гнетущей. Глухой шум Лондона доносился сюда, как гудение
далекого органа.
Портрет Дориана Грея.

Вот как это всё с ходу смоделировать, художники неделями работают над костюмами героев, над декорациями, над светом, над всем, как я должен всё это с ходу представить? И в фантастике я не могу с ходу представить интерьер корабля по описанию автора или любого другого окружения, у меня в голове туман
из образов что то более ясно что то мение, как во сне и не более. Хоть бы концепт арты прикладывали что ли. Хуй знает, люблю что бы чётко было, без всяких благоуханий цветков цвета морской волны на закате индийского океана, возьми да сука нарисуй, что сказать хотел, не еби мозги.
Аноним 01/09/15 Втр 10:42:36 #198 №295012 
>>295010
Пиздец, нахуй ты меня в эту деревню завел. Сибири упори, и все норм будет.
>Книга может дать только информацию нагрузить чисто думательный аппарат, а кино приносит эстетическое удовольствие музыка, картинка. Книга как чисто развлекательный контент устарела.
Ты прав и не прав. Кино приносит "ржаку". Пожрать и посрать, на отягощая котелок - вот что кино позволяет. Не в последнюю очередь сейчас ты отвечаешь эстетствующим кастратам, которые читают статусные книги, но нихуя в них не понимают, зато "читал книгу, я блять интелектуал".
>>295006
>Еле дочитал саный солярис
Бг-г-г, я бы не осилил. Не могу читать чистый худ лит. Войну и мир почитай, я помню взял кусок откуда-то из середины и залип часа на два.
Наука или образование?  rainautumn 01/09/15 Втр 11:10:24 #199 №295021 
что общего в этих словах? Считаю что ничего, а вы?
Аноним 01/09/15 Втр 11:39:46 #200 №295026 
>>295021
>Сладкое или яблоко
С чего им быть общими? Только лишь предвзятая ассоциация.
Аноним 01/09/15 Втр 11:50:14 #201 №295027 
>>294998
>определены способы измерения изменения энтропии
экспериментально или через формулу? ту самую для исправления кривизны которой энтропию и ввели например
Аноним 01/09/15 Втр 11:51:57 #202 №295030 
>>293994
>>293966
Пиздец, какая нахуй наука, какие нахуй исследования, если на всей сраной борде не нашлось человек, который понимает, что такое цикл карно, и при чем тут холодильник. Какие нахуй черные дыры, какая блять тепловая смерть, если тут термодинамику уровня девятого класса осилить не могут?
Аноним 01/09/15 Втр 11:54:02 #203 №295031 
>>295021
Ну, а так в школах, конечно надо учить не математике и литературе с физикой, а в первую очередь теории науки и логическому, рациональному мышлению. В противном случае текущие школьное образование поощряет слепо зубрит догматы и наказывает если ты их не усваиваешь, тоже богословие только в 21м веке. Мы в блядской школе основы философии проходили и нахуя это надо? Зачем меня заставляли заучивать чужую жизненную философию? У нас её директриса вела, пацан один чёт бугуртнул и намекнул ей что думает что фрейд хуйню нёс и психоанализ плохо работает, она не спросила почему и сразу выгнала его, лол.
Аноним 01/09/15 Втр 12:28:52 #204 №295037 
>>294992
Он о большом разрыве. Хотя мне нравится эта теория. По-моему довольно и красивый конец. Каждая элементарная частица уходит за горизонт событий относительно всех остальных. Этакое абсолютное хиккование, вечное одиночество. А насчет маргинальности, так все теории конца вселенной в равной степени маргинальны, пока нету теории квантовой гравитации и неизвестна суть темной энергии, да и много чего еще.
Аноним 01/09/15 Втр 13:15:31 #205 №295042 
>>295031
Логическое, рациональное мышление в чистом виде - это прямой путь в стаю тупых мудаков. Я писал об этом в треде про старых пердунов. "А смысл жизни нашей в чем? А смысла в жизни нет".
Ты говоришь о том, какой должна быть правильная школа, но ты сука забываешь задать вопрос "а нахуй я вообще в нее ходил и было ли это моим желанием". Все, дальше сам думай.
Аноним 01/09/15 Втр 13:31:09 #206 №295047 
>>295042
>"А смысл жизни нашей в чем? А смысла в жизни нет"
Но так и есть же.


>а нахуй я вообще в нее ходил и было ли это моим желанием
Хуй знает, но я очень рад был ходить в школу(отличник даже по поведению и физкультуре), я реал думал что меня там научат атомы расщеплять и ваще всю подноготную мироздания вскроют, и что если я буду хорошо учится то преуспею в жизни и получу статус, и всё у меня будет хорошо и очень серьёзно отнёсся к достижению успехов в школе. Насколько это желание моё я хз. Голодный уверен что это его желание поесть, наркоман что это его желание еще принять наркотик, а самец уверен что полностью его желанье выебать самку своего вида и так вообще можно дойти до того что свобода воли иллюзия, возможно, что только суицид это победа чистого разума над программой выживай-плодись.
Аноним 01/09/15 Втр 15:14:48 #207 №295070 
Анон, а что если скорость света бесконечна? Точнее, свет распространяется мгновенно на любые расстояния? И мы видим звезды не такими, какими они были дохуя лет назад, а в реал-тайме?
Аноним 01/09/15 Втр 15:19:02 #208 №295072 
>>295070
http://www.voprosy-kak-i-pochemu.ru/mnenie-priroda-energii-i-oshibki-izmereniya-skorosti-sveta/
Аноним 01/09/15 Втр 15:45:14 #209 №295078 
>>295070
нет
Аноним 01/09/15 Втр 17:46:11 #210 №295088 
>>295078
Почему?
Аноним 01/09/15 Втр 17:46:57 #211 №295089 
Что нужно, чтобы достичь скорости, на которой до марса лететь недклю? Просто допиздища топлива, чтоб разгоняться?
Аноним 01/09/15 Втр 17:51:24 #212 №295091 
>>295089
Нужно поставить тебе пять двоечек в четверти. Тогда улетишь на баттхёртной тяге.
Аноним 01/09/15 Втр 17:57:29 #213 №295094 
>>295088
Потому что тысячи экспериментов.
Аноним 01/09/15 Втр 18:15:26 #214 №295096 
>>295070
Тогда не работал-бы GPS (он юзает кучу поправок, вытекающих из теории относительности, эрго конечности скорости света).
Аноним 01/09/15 Втр 18:18:09 #215 №295097 
>>295089
Да.
Аноним 01/09/15 Втр 19:20:31 #216 №295116 
>Множество трансцендентных чисел континуально.
Как так? Ведь между двумя любыми числами всегда будет рациональное.
Аноним 01/09/15 Втр 19:31:53 #217 №295117 
>>295116
> Ведь между двумя любыми числами всегда будет рациональное.
И причём тут это?
Аноним 01/09/15 Втр 19:52:20 #218 №295122 
>>295116
Множество алгебраических чисел (более мощное, чем множество рациональных) всего лишь счётно, множество всех чисел - континуум, следовательно, их разность - континуум, в чём проблема?
Аноним 01/09/15 Втр 20:19:00 #219 №295126 
>>295097
т.е. теоретически мы бы могли разгонаться так, чтоб за три дня долетать до марса, и проблема только в топливе?

Как долго разгоняться до такой скорости? Как это посчитать?
>>295096
А если там дело в чем-то другом?
Аноним 01/09/15 Втр 20:32:52 #220 №295127 
>>295126
Совсем школьник? Расстояние известно (можешь взять среднее), приемлимое ускорение можешь принять g и считаешь по уравнению S = gt^2 + gt(734*3600-2t)
Аноним 01/09/15 Втр 20:34:09 #221 №295128 
>>295096
Даже без поправок на искажение времени GPS работает через разницу во времени дохождения сигнала от разных спутников же.
Аноним 01/09/15 Втр 20:37:04 #222 №295130 
>>295127
Блядь, звёзды проебались. Короче, расстояние равно расстояние разгона плюс расстояние торможения (gt^2 в сумме) плюс расстояние полёта (неделя минус 2t умножить на gt).
Аноним 01/09/15 Втр 20:52:15 #223 №295132 
>>295127
>Совсем школьник?

Ты не поверишь, я не знаю школьной программы ни физики, ни химии. Преподавание было столь мерзким, что отбило у меня желание что-то учить до сих пор, амне уже 24. Буквально у меня судороги наступают при мысли об чтении каких-то учебных материалов. Кроме шуток.
Аноним 01/09/15 Втр 21:03:47 #224 №295134 
>>295122
У какого множества больше мощность, у множества всех непрерывных функций или у множества всех подмножеств счётного множества?
Аноним 01/09/15 Втр 21:03:49 #225 №295135 
>>295122
>>295117
Ну я так понял, континиум значит непрерывность, а если на числовой прямой между любыми трансцендентными числами всегда стоит алгебраическое, то их нельзя соединить отрезком, все точки которого трансцендентные числа, т.е. это такие же отдельные точки, как все отсальные.
> Множество алгебраических чисел (более мощное, чем множество рациональных) всего лишь счётно, множество всех чисел - континуум, следовательно, их разность - континуум, в чём проблема?
Ну для меня это выглядит противоречием с геометрическим представлением пока что.
Ну т.е. берём два трансцендентных числа, они различаются в каком-то знаке, значит всё, можем между ними поставить рациональное.
Аноним 01/09/15 Втр 21:08:04 #226 №295136 
>>295134
>ества больше мощность, у множества всех непрерывных функций или у множества всех под
>>295135


Пошло в математический тред, убожество. не тащи сюда это фентези-дерьмо без задач.
Аноним 01/09/15 Втр 21:19:36 #227 №295137 
>>295136
Множество всех непрерывных функций включает тебя самого. Всю твою никчемную жизнь можно описать непрерывной функцией. Ты должен полюбить это множество.
Аноним 01/09/15 Втр 21:20:23 #228 №295138 
14411316230660.jpg
>>295136
> Пошло в математический тред, убожество. не тащи сюда это фентези-дерьмо без задач.
Вся суть мамкиных недофизиков.
Дискретность времени и пространства доказал уже?
Аноним 01/09/15 Втр 21:29:57 #229 №295140 
>>295135
>континиум значит непрерывность
Нет. Это значит только, что непрерывность является континуумом. Какой бы малый отрезок ты ни взял, в нём будет континуум точек, в том числе счётное количество рациональных.
>их нельзя соединить отрезком, все точки которого трансцендентные
Да. Но из этого ничего не следует.

>>295134
Равномощны же.
Аноним 02/09/15 Срд 00:52:51 #230 №295176 
>>295138
А ты яблоко разрезаниями и складываниями кусочков вместе сделал яблоко больше в два раза или атомы помешали?
Аноним 02/09/15 Срд 00:56:13 #231 №295178 
>>295176
>пространство
>атомы
Аноним 02/09/15 Срд 15:59:33 #232 №295335 
14411987738740.jpg
Производит ли космический летательный аппарат ЭМИ?
Как дела сейчас обстоят с ЭМИ у всякой военной техники? Возможна ли 100%-ая изоляция техники от ЭМИ? Какого спектра?
И вообще, поясните за это
http://star-citizen.pro/index.php/component/k2/item/566-dizajn-radioelektronnaya-borba
пожалуйста.
Аноним 02/09/15 Срд 16:08:35 #233 №295336 
>>295335
>Производит ли космический летательный аппарат ЭМИ?
>Как дела сейчас обстоят с ЭМИ у всякой военной техники?
Ты хоть сам понял, что спросил?
Шел бы лучше домашку делать
Аноним 02/09/15 Срд 16:41:31 #234 №295340 
>>295336
Я домашку уже лет 15 не делаю. А вот плановую операцию на сегодня выполнил и пришел домой пораньше
Аноним 02/09/15 Срд 17:05:50 #235 №295344 
>>295340
Термин ЭМИ употребляется, когда речь идет о поражающих факторах электромагнитного излучения. Тебя же, вероятно, интересует ЭМС (электромагнитная совместимость).
>способность технических средств одновременно функционировать в реальных условиях эксплуатации с требуемым качеством при воздействии на них непреднамеренных электромагнитных помех и не создавать недопустимых электромагнитных помех другим техническим средствам.
https://ru.wikipedia.org/wiki/Электромагнитная_совместимость
>Как дела сейчас обстоят с ЭМИ у всякой военной техники?
По-разному, в зависимости от типа техники и используемого в ней оборудования. Самая серьезная степень защиты от ЭМИ в самолетах и космических аппаратах. Методы защиты тоже разные, но как правило это "экранирование". Пикрилейтед - кабель с экранирующей оплеткой.
>Возможна ли 100%-ая изоляция техники от ЭМИ?
Нет, невозможна.
Аноним 02/09/15 Срд 17:06:53 #236 №295345 
14412028130940.jpg
>>295344
Пик отклеился
Аноним 02/09/15 Срд 18:11:41 #237 №295350 
>>295344
>ЭМИ употребляется, когда речь идет о поражающих факторах
Ой не пизди тут, а, ыксперт.
>электромагнитного излучения
- это и есть ЭМИ.

>>295335
>Возможна ли 100%-ая
Незачем. Достаточно снизить до приемлемых параметров.
Аноним 02/09/15 Срд 20:04:45 #238 №295378 
Как посчитать последовательность?
по типу: n=(n-1)x1.2 и округлить до целого числа в меньшую сторону, где n1=10 . n2=12 n3=14 n4=16 n5=19 Как посчитать n100 не считая всё по отдельности?
Спасибо.
Аноним 02/09/15 Срд 20:46:26 #239 №295381 
14412159866010.png
Откуда у негров из центральной Африки толерантность к лактозе?
Аноним 02/09/15 Срд 21:32:55 #240 №295392 
>>295378
Для вывода общяя формула нам потребуется математическое индукциЁ. Для начала нам надо предсказать саму формулу. попробуем поподставлять её саму в себя и понаблюдаем за результатами. Ах да, ты забыл указать базу - первый элемент последовательности. Ну что ж, примем её за n0(нумеруем всё с нуля ибо всё в мире есмь пустота и суета). n1=n01.2; n2=n11.2=n01.21.2, n3=n21.2=2n01.21.21.2 и так далиЁ.
Даже недалёкий штудент заметит закономерность - индексы под n увеличиваются, а количество множителей 1.2 растёт. Растёт в нехитрой зависимости от индекса. Так что, не без помощи Всевышнего мы можем положить формулу ni=n01.2^i. Но как же математика? На Бога надейся, а сам не плошай. Нам следует доказать верность этой боговдухновенной формулы.
Для этого проверим гипотезу при i=0: ni = n0
1.2^0 - работает, но этого недостаточно.
Докажем, что если для i формула верна, то она также верная для i+1(в этом и состоит основная магиЁ).
ni=n01.2^i - верно по условию индукции. n_(i+1)=n_i1.2(просто перефразируя твоё n=(n-1)x1.2 в удобоваримый для меня вид - мыслим как смещение индексов на единичку назад, либо подстановочку n`=n-1, далее используем n`). вычисляем n_(i+1) = n_i1.2=n01.2^i1.2=n0q^(i+1) - формула остается верной.
Ну а дальше следует просто поверить в мощь математической индукциЁ(обычно она постулируется, как метод доказательства - но не об этом речь) - формула будет верна для любого i.
УРА! А теперь время принести молитву Всевышнему, без него бы мы никогда не смогли предвидеть общий вид формулы. Но доказали-то её верность мы своими руками - во истину - не Боги горшки обжигают!
Аноним 02/09/15 Срд 21:52:19 #241 №295398 
>>295381
http://www.youtube.com/watch?v=h8Uzo8G9L1g
был где то более нормальный видос где было показано как смешивают, но не нашел. покопайся в релейтед.
Аноним 02/09/15 Срд 22:04:27 #242 №295401 
>>295392
Нихуя не понял.
Как всё же выглядит формула, округления до целого числа в меньшую сторону?
Если что я уже указал n1=10,а не n1=0
Если умножать 0 на 1.2 то все числа последовательность будут равны нулю.
Я же прошу 10х1.2=12(это n2); 12х1.2=14.4 с округлением это 14(это n3); 14х1.2=16.8 с округлением это 16(это n4).
Как посчитать n100?
Я кстати не студент.
Аноним 02/09/15 Срд 22:26:48 #243 №295405 
Наукач, объясни гуманитарию логарифмы. Есть разница между N log N и logN(N)? Это обозначает одно и то же? И почему гугл выдаёт такие результаты - https://gyazo.com/4dcbcaf3ee6ef03b484e3e6789a14a0d, если там по идее должно быть 0,5?
Аноним 02/09/15 Срд 22:42:33 #244 №295407 
>>295405
Есть разница, конечно. log без указания показателя - это десятичный логарифм. В твоём примере log (9) будет тем, в какую степень нужно возвести 10, чтобы получить 9, естественно, это не 2/81, а что-то чуть меньше 1. Если хочешь посчитать логарифм произвольного основания на калькуляторе без соответствующей кнопки, это делается как log (81) / log (9).
Аноним 02/09/15 Срд 22:44:20 #245 №295409 
>>295405
Нет, не одно и то же. Логарифм равен степени в которую нужно возвести основание логарифма(N подстрочное), чтобы получить N.
Для логарифмов по некоторым основаниям введены специальные обозначения. Такие как lg - логарифм по основанию 10, ln - логарифм по основанию e. В других случаях надо обязательно указывать основание. Пример, lg(1000) = 3. Основание логарифма - 10. в какую степень надо возвести 10, чтобы получилась 1000? В 3 степень.
Аноним 02/09/15 Срд 22:52:39 #246 №295411 
>>295409
>>295407
Спасибо за исчерпывающие объяснения, котятки, теперь всё понятно.
Аноним 03/09/15 Чтв 00:59:23 #247 №295434 
Мне тред создать?
>>295401
Аноним 03/09/15 Чтв 02:13:39 #248 №295441 
14412356196070.jpg
>>295027
бамп вопросу
Аноним 03/09/15 Чтв 03:08:03 #249 №295445 
>>295401
Насколько я понимаю, эта штука без вычисления промежуточных n не решается никак, но а в чём проблема их вычислить? Слава Б-гу, у нас помимо калькуляторов те же таблицы есть.
Аноним 03/09/15 Чтв 05:23:36 #250 №295449 
>>295445
Спасибо.
Ну так это долго. И не применимо к большим числам.
Аноним 03/09/15 Чтв 05:55:57 #251 №295451 
>>295401
>>295449
Ну так и быть, дорогой не-студент - тебе нужна формула геометрической прогрессии для n-го члена. Доказать её строго я тебе попытался (попутно развлекая себя шутками и пребаутками) - но видимо пока рановато.
https://ru.wikipedia.org/wiki/%D0%93%D0%B5%D0%BE%D0%BC%D0%B5%D1%82%D1%80%D0%B8%D1%87%D0%B5%D1%81%D0%BA%D0%B0%D1%8F_%D0%BF%D1%80%D0%BE%D0%B3%D1%80%D0%B5%D1%81%D1%81%D0%B8%D1%8F
А про математическую индукцию тут - это на вырост
https://ru.wikipedia.org/wiki/%D0%9C%D0%B0%D1%82%D0%B5%D0%BC%D0%B0%D1%82%D0%B8%D1%87%D0%B5%D1%81%D0%BA%D0%B0%D1%8F_%D0%B8%D0%BD%D0%B4%D1%83%D0%BA%D1%86%D0%B8%D1%8F
Аноним 03/09/15 Чтв 06:35:13 #252 №295457 
>>295451
Ты условия не дочитал, похоже. Тут не геометрическая прогрессия, на каждом шаге от числа берётся целая часть. Формула геометрической прогрессии даёт неправильный результат уже на третьем шаге, и далее ошибка нарастает.
Аноним 03/09/15 Чтв 06:50:25 #253 №295459 
>>295449
Смторя насколько большим. Тот же эксель влёт (секунды за две) считает эту штуку до n4000, там 250 знаков, это сильно больше, чем практически может понадобиться. Если нужно ещё больше - минут за 20 пишется программа на том же С, которая хуй знает докуда тебе посчитает, но я не думаю, что нужно больше.
Аноним 03/09/15 Чтв 07:01:53 #254 №295461 
>>295457
да, верно - твоя шняга это не геом. прогрессия, но очень похожа на неё. Всё дело портит округление. Попробую поразмыслить вслух -
A_n=[A_n-1 q] -> можно представить, что каждый шаг мы вычитаем дробную часть. Чему она равна? Она зависит только от последней цифры элемента последовательности, по закону mod(a0.2, 1) - где mod это взятие остатка. На примере - 10.2=0.2, 20.2=0.4, ... 50.2=0, 60.2=0.2(произошел перенос разряда, отсюда), 70.2=0.4, далее 0.6, 0.8, 0. Как видим - тут цикличность на лицо, поэтому надо записать зависимость дробной части от индекса последовательности(обозначу за Rn) и добавить её в формулу
A_n=An-1
q-Rn. Т.е. каждый шаг мы вычитаем дробную часть, для которой зависимость от номера мы можем вывести.
Эта последовательность уже не геометрическая прогрессия, но общая формула члена для неё есть - где-то её видел. Можно попробовать вывести самому, либо заюзать вольфрам - сейчас мне в лом этим заниматься.
В общем, я тебе обрисовал ход вывода формулы - спрашивай, что непонятно.
Аноним 03/09/15 Чтв 07:04:56 #255 №295462 
>>295459
Как я понял, стоит именно задача аналатически вывести общую формулу - а не посчитать для какого-то N. Тем более могут быть проблемы с переполнением, может ОПу вопроса вообще нужно в длинных числах всё посчитать.
Но пусть всё-таки >>295457 уточнит что ему надо - посчитать точно или аналитически вывести формулу.
Аноним 03/09/15 Чтв 07:07:23 #256 №295463 
>>295461
кстати, парсер сжевал все мои звездочки. Сорян, привык ими обозначать умножение, а не иксами.
Аноним 03/09/15 Чтв 07:14:34 #257 №295464 
>>295461
А может для вывода применить комп и не ебать мозги?
Онлайн-вольфрам http://www.wolframalpha.com/ видимо не катит, там нету символа/функции "взятия целой части" - а может и есть, но скрыто. Также ты бы мог задать последовательность реккурентной формулой и вежливо попросить посчитать аналитически в мат. пакете(хорошо могущего в символьные вычисления - например wolfram mathematica).
Аноним 03/09/15 Чтв 07:31:55 #258 №295466 
>>295463
Это печально, потому что (несмотря на то, что ексель, как выяснилось, начинает лажать где-то в районе n200) я не вижу цикла в последней цифре - а значит и в отбрасываемой дробной части.
Аноним 03/09/15 Чтв 07:39:58 #259 №295468 
>>295466
Очень странно. Может, я поторопился с цикличностью - это была интуитивная гипотеза. До какого N она наблюдается и что с ней происходит потом? Все-таки сюда надо привлекать бумажку с ручкой и формулки - либо математический пакет.
Аноним 03/09/15 Чтв 07:42:56 #260 №295469 
>>295466
Хотя если целая часть достаточно большая - там уже погрешность представления чисел в памяти будет(разряды мантиссы конечные). Но там количество цифр должно зашкаливать за 6 как минимум(грубо это погрешность 32-разрядной плавающей точки). Скинь то место из последовательности, где цикличность нарушается.
Аноним 03/09/15 Чтв 08:14:15 #261 №295473 
>>295468
Она просто нигде не начинается. Вот первые 30 чисел
10
12
14
16
19
22
26
31
37
44
52
62
74
88
105
126
151
181
217
260
312
374
448
537
644
772
926
1111
1333
1599
Последняя цифра, конечно, однозначно определяет дробную часть, но вот сама она определяется последними двумя цифрами, вторая - второй и третьей с конца и так далее.
Аноним 03/09/15 Чтв 08:39:47 #262 №295474 
>>295473
Ладно, ты заинтересовал меня своей задачей. Ща посмотрю в calc(это электронные таблицы, я под линупсом).
Говоря про цикличность я имел в виду то, что для любого числа кол-во вариантов дробной части ограничено(перебирая возможные первые цифры чисел An и смотря на дробные части An+1 мы наблюдаем цикличность) и скорее всего там есть закономерность.
Аноним 03/09/15 Чтв 08:56:01 #263 №295476 
14412597611150.jpg
Уже спрашивал, но проигнорировали. У земли есть два поля, электромагнитное и электрическое. Первое из-за возможных токов в ядре, второй из-за наличия у земли заряда. Вопрос, почему меня не бьет током, если подпрыгну почему не ударит током? Есть же разность потенциалов.
Аноним 03/09/15 Чтв 09:00:54 #264 №295478 
>>295473
а вот и последовательность дробных частей до N=100
0 0.4 0.8 0.2 0.8 0.4 0.2 0.2 0.4 0.8 0.4 0.4 0.8 0.6 0 0.2 0.2 0.2 0.4 0 0.4 0.8 0.6 0.4 0.8 0.4 0.2 0.2 0.6 0.8 0.6 0.2 0.2 0.6 0 0 0.8 0.6 0.2 0.8 0.2 0.8 0.2 0.6 0.6 0.6 0.4 0.4 0.8 0.2 0.8 0 0 0.2 0.6 0.4 0.8 0.4 0.8 0.8 0.6 0.6 0.2 0.2 0.8 0.8 0.8 0.7999999998 0.6000000001 0.1999999997 0.1999999997 0.5999999996 0.2000000002 0 0 0.5999999996 0.7999999989 0 0.4000000004 0.1999999993 0.8000000007 0.3999999985 0.3999999985 0.799999997 0.3999999985 0 0.1999999955 0.3999999911 0.3999999911 0.3999999911 0.799999997 0.400000006 0.400000006 0.400000006 0.8000000119 0.3999999762 0.3999999762 0
Как видишь - погрешность(скорее всего представления, а не вычислительная) начинает себя проявлять. На вскидку - простой закономерности я в этой последовательности не вижу - так что общую формулу подсказать не могу. Способ улучшить "читабельность" и удобство работы - это представить последовательность дробных частей как посл-ть натуральных чисел(просто поделив на 0.2 (и округлим к целому т.к. у нас есть копеечные погрешности) - от этого ничего не теряем) - на выходе
0,2,4,1,4,2,1,1,2,4,2,2,4,3,0,1,1,1,2,0,2,4,3,2,4,2,1,1,3,4,3,1,1,3,0,0,4,3,1,4,1,4,1,3,3,3,2,2,4,1,4,0,0,1,3,2,4,2,4,4,3,3,1,1,4,4,4,4,3,1,1,3,1,0,0,3,4,0,2,1,4,2,2,4,2,0,1,2,2,2,4,2,2,2,4,2,2,0,4,
использование базы числовых последовательностей
oeis_org ничего внятного не дало. Но отсюда не следует, что закономерности нет - следует обратиться к специалистам "более высокого уровня", обитающих на математических форумах вроде dxdy, mathoverflow(на ангельском). Может подскажут хитрую магию сверхвысокого уровня, которая решит исходную задачу на раз.
Аноним 03/09/15 Чтв 09:03:05 #265 №295479 
>>295476
Во-1 ты имеешь сопротивление и выступаешь как резистор.
Во-2 в организме и так есть микротоки в жидкостях(в крови например) - они текут видимо уже с поправкой на электрическое поле земли.
В-3 видимо разность потенциалов "естественного" поля небольшая.
Аноним 03/09/15 Чтв 09:09:33 #266 №295481 
>>295473
И еще - ты так и не сказал - тебе нужно просто посчитать сколько-то элементов последовательности или тебе нужна общая формула. Просто посчитать - быстро и с хорошей точностью ты можешь, написав программу на C за 20 минут(за час-два-три если не знаешь C и погромирование). Если нужны довольно большие числа - используй unsigned long(диапазон представимых чисел можешь в инете найти) и/или double, если еще больше - придется прибегнуть к длинной арифметике.
Вот вроде и всё, что я могу сказать по этой задаче.
Аноним 03/09/15 Чтв 10:07:51 #267 №295498 
Анон, такой вопрос: сколько нужно лампочек чтобы создать освещение равнозначное, например, свету в комнате в полдень? Как вообще измеряется этот свет?
Аноним 03/09/15 Чтв 11:13:12 #268 №295520 
>>295498
Твой вопрос но-нубски сформулирован для физики. Можно его переформулировать так - как получить равномерное освещение в параллелепипеде с помощью точечных источников света. Так можно составить поисковый запрос/вопрос на форум. Свет много в чём измеряется - в зависимости от контекста. Есть например https://ru.wikipedia.org/wiki/%D0%9E%D1%81%D0%B2%D0%B5%D1%89%D1%91%D0%BD%D0%BD%D0%BE%D1%81%D1%82%D1%8C

https://ru.wikipedia.org/wiki/%D0%A1%D0%B2%D0%B5%D1%82%D0%B8%D0%BC%D0%BE%D1%81%D1%82%D1%8C

https://ru.wikipedia.org/wiki/%D0%A1%D0%B8%D0%BB%D0%B0_%D1%81%D0%B2%D0%B5%D1%82%D0%B0
Сомневаюсь, что ты всё это поймешь без минимальной мат. подготовки. Пережевывать это "на пальцах" мне в лом, попроси других анонов/создай тред/погугли.
Аноним 03/09/15 Чтв 14:44:44 #269 №295574 
>>295498
Солнечная постоянная - это примерно киловатт мощности на квадратный метр (это не киловатт лампочек, это киловатт прилетающей энергии).

Другое дело, если тебе ещё и такой-же спектр нужен, тогда нужны лампочки с температурой спирали в 6500К.
Аноним 03/09/15 Чтв 15:09:03 #270 №295582 
Мимо погроммист, решил учить матан, подскажите учебник для совсем краба.
Один наугад скачал и ...
" вот мы наблюдаем теорему #### и её выражение ^&&&/%^^% , это очень просто и очевидно,поэтому мы выразим из него &€£€^%^/^&+×1.. "
Аноним 03/09/15 Чтв 15:12:48 #271 №295583 
Не то, чтобы погроммист, а первый курс погроммистов.
Добавил
Аноним 03/09/15 Чтв 15:53:42 #272 №295590 
>>295582
Ну это называется разрыв шаблона при переходе от элементарной математике к высшей. Почти у всех наблюдается(за исключением мат. школьников дрочащих матан уже в школе). Постарайся расслабить ментальные булки для безболезненного проникновения матана в мозг, читать и вникать, задавать вопросы(преподу или гуглу, спрашивать на спец. форумах вроде dxdy), осмыслять. А не заучивать и повторять непонятную белиберду как попугай. Но, к сожалению - это довольно распространенный способ усвоения и сдачи математических дисциплин.
Из учебников - фихтенгольца по старинке вроде все советуют(гугли - рекомендаций по инету полно). Он старается всё разъяснять - особенно в тонких местах, давать прикольные "лирические отступления". Еще - многие советуют учить по университетским лекциям - там преподы стараются разжевать и облегчить всё по максимуму(поищи в электронных и видео-версиях). В книжках часто бывает крайне жесткий "заход" для начинающего, предполагающий что ты - мат. школьник и достаточно надрочен.
Аноним 03/09/15 Чтв 16:35:38 #273 №295602 
Вот у всех по по две одинаковые хромосомы(зачем 2 одинаковых?), предаётся только одна, как определяется какая?
Аноним 03/09/15 Чтв 16:45:45 #274 №295607 
http://ru-an.info/новости/гравитация-это-совсем-не-закон-всемирного-тяготения/

Вот тут человек утверждает, что гравитацией обладают только отдельные тела. Хотелось бы выслушать мнение экспертов.
Аноним 03/09/15 Чтв 18:16:30 #275 №295631 
>>295607
>Гравитация – это совсем не «Закон всемирного тяготения»

В сегодняшней земной науке сфальсифицировано всё: все основные законы естествознания оказались извращены на 100% Все наши представления о структуре Мироздания не имеют никакого отношения к реальности! Мы почти ничего не знаем..

>Этот простой пример показывает, насколько сегодняшняя наука далека от реальности и от желания честно заниматься полноценными исследованиями. Конечно, в этом вина не только псевдо-учёных, но и в большей степени кукловодов, которые изо всех сил стремятся сделать из людей стадо овнов (баранов), которое бы занималось добыванием корма и больше ничем. Но «учёных» это не оправдывает, потому что слабоволие и безчестность сделало из них самых настоящих паразитов, веками сладко кушающих за счёт Общества, и не дающих взамен ничего, кроме довольно убогих фантазий, тормозящих эволюционное развитие Человечества.

>Недолгий поиск в Ютюбе привёл к обнаружению большого числа видеороликов, на которых демонстрируются вполне реальные примеры антигравитации. Некоторые из них я перечислю здесь, чтобы вы смогли убедиться, что антигравитация (левитация) действительно существует, но… до сих пор никем из «учёных» не объяснена, видимо, гордость не позволяет…


Понятно.
Аноним 03/09/15 Чтв 18:20:52 #276 №295634 
>>295631
>В сегодняшней земной науке сфальсифицировано всё: все основные законы естествознания оказались извращены на 100%

В принципе и этого достаточно, что бы понятно всё стало.
Аноним 03/09/15 Чтв 18:22:34 #277 №295635 
>>295631
/sci/ запалился в каком-то фриковнике? В SETI-тред на днях запостили оглавление "труда", в котором в т.ч. про влияние мата на ДНК.
Аноним 03/09/15 Чтв 20:16:05 #278 №295672 
>>295590
Тааак, спасибо, вроде начал по-немногу понимать нёх.
Навернул комплексные числа и матрицы.
Теперь вопрос, ну не заучивать же кучу формул, как стихи ?
Половину можно выразить, вторую понять по определению и тд. Но..
Аноним 03/09/15 Чтв 21:08:01 #279 №295676 
>>295635
+
Аноним 03/09/15 Чтв 21:21:27 #280 №295678 
>>295635
>>295631
>>295634

Окей, петухоиды малолетние, как вы объясните это?

https://youtu.be/Mw79upkOCXo

Давайте, я жду.
Аноним 03/09/15 Чтв 22:48:59 #281 №295687 
>>295672
Для решения задач/доказательств - реально нужно помнить основные формулы и свойства объектов - векторов, пределов, матриц(ты ими постоянно оперируешь). Хотя поначалу можно обойтись шпорой - после осознания этих формул. Формулы сами запомнятся по ходу решения большого количества задач. Заучивать специально наверное не надо - просто смотри в шпору во время решения задач - тут как с переводом с иностранного люди запоминают слова, когда переводят текст(мне кажется так проще запомнить). Научишься жонглировать ими за нехуй - если не лениться прорешивать домашки с семинаров. Но самое основное имхо - определения и формулировки "полезных в хозяйстве" теорем, надо хорошо их представлять, потому что из них можно тривиально вывести чуть ли половину всего остального. А вот нетривиальные доказательства в голове держать не стоит, лучше уж вариант "один раз понять и забыть". Теория(доказательства, всякие теоретические тонкости) сама по себе в задачах не нужна - но чтоб хорошо сдать экзамен в конце семестра нужно её знать и понимать(в идеале - от препода зависит). Вообще в теорию лучше пытаться вникать постепенно - а то под конец семестра навалится адским грузом, возникнет полный ахуй, куча вопросов и непонимание - всё в условиях дефицита времени(сессия).
Аноним 04/09/15 Птн 08:07:00 #282 №295724 
>>295678
Это левитация.
Аноним 04/09/15 Птн 09:02:20 #283 №295729 
Я придумал вопрос, лел.
Как работает полупрозрачное зеркало? У меня три школо-"модели" на выбор (на большее фантазии не хватат):
1. На атомном уровне оно что-то вроде шахматной доски, т.е. есть "клеточки", которые пропускают свет, а есть отражающие. Соот-но половина фотонов отражается, половина пролетает.
2. Из-за каких-то неведомых колебательных процессов у материала есть "короткая память": грубо говоря, каждый чётный фотон пролетает, каждый нечётный отражается.
3. Какие-то квантовые ебеня, которые приводят к вероятностному характеру: каждый следующий фотон как симметричная монетка, 50% вероятность, что пролетит, 50%, что отразится.

Если вкратце ответа нет, подскажите, пожалуйста, годную литературу, итмоптики.
Аноним 04/09/15 Птн 09:31:35 #284 №295736 
>>295729
На таком уровне лучше рассматривать свет как волну, а не как частицу. Потому что фотоны - это не шарики, это "квантовые ебеня", которые проявляют себя как волны в таких процессах. Уравнения Максвелла (которые описывают поведение волн), все дела.

Чтобы получить интуицию, представь себе ванну, у которой посередине натянута пленка которая позволяет разделить 2 жидкости. В одной половины ванны налита вязкая жидкость, в другой - вода. Если ты бросишь камень, то "на границе раздела сред" (у этой пленки), будут образовываться две волны, прошедшая и отраженная. Вот на википедии нашел гифку:
https://en.wikipedia.org/wiki/Fresnel_equations#/media/File:Partial_transmittance.gif

В плане формул, тут есть все, что тебе нужно
http://aco.ifmo.ru/upload/publications/book_basic_optics_2000.pdf
Параграф 3.2, но чтобы его понять, нужно прочитать весь параграф 3.
Аноним 04/09/15 Птн 09:39:58 #285 №295740 
>>295736
Спасибо, гифка хорошая, аналогия с ванной интересная. Док посмотрел -- ок, это чисто волновой подход.
Но всё-таки не покидает голову вопрос, что будет с отдельным фотоном, падающим на такое зеркало. Он же не разделится на два с той же длиной волны.
Аноним 04/09/15 Птн 10:04:52 #286 №295744 
>>295736
Ок, я, наверное, должен корректнее сформулировать вопрос, раз речь идёт о корпускулярно-волновом дуализме.
Вот ставлю я лазер мощностью 1 мВт, 532 нм. Т.е. он испускает порядка 3е16 фотонов в секунду. Далее я ставлю 10 полупрозрачных зеркал (последовательно) и ожидаю на дву выходах из последнего зеркала получить по ~3e6 фотонов. Ставлю там два ФЭУ с временным разрешением 10 нс в режиме счета фотонов, зацепляю выходы на осциллографы и смотрю. Что я увижу? Их показания будут скоррелированы (типа импульс на ФЭУ1, потом импульс на ФЭУ2, по очереди) или нет? Это же, по идее, два пуассоновских процесса должны быть. Но взаимосвязаны ли они как-то?
Аноним 04/09/15 Птн 10:09:03 #287 №295747 
14413505431450.png
>>295740
Вообще, если что, я оптик, а не физик. Поэтому у нас курсы этой хуйни были, но в работе достаточно некоторой интуиции. Так что говорю по памяти и могу сказать что-то не то.

На уровне фотонов можешь считать, что в среде происходит их постоянное переизлучение атомами. Т.е. фотон летел, поглотился атомом и тут же переизлучился - с той же частотой. Отсюда возникает, собственно, разная скорость света в средах. Переизлучает атом в случайную сторону (естественно, он не знает ничего о границе раздела сред и окружающем мире), но в стекле из-за интерференции фотонов (опять корпускулы-волны, совсем без них никуда) только 2 направления не будут взаимогаситься, вперед, так сказать, и назад.
Т.е. на пикрелейтеде на 2 соседних атома падают фотоны и отражаются в случайную сторону. Но взаимногасящие направления погасят ситуации 1,3,4,6 и оставят только 2 и 5. То же и с отраженной волной.
Это что происходит с потоком фотонов. Один фотон, по идее, ведет себя так же, как поток. Мы же не можем проследить за ним и посмотреть, что он делает.

В итоге, если ты пошлешь фотон в полупрозрачное зеркало, то детектором ты его поймаешь с 50% вероятностью или с одной, или с другой стороны, вот и все, что можно сказать.
Аноним 04/09/15 Птн 10:22:39 #288 №295753 
>>295744
Ну, одновременно импульсов на обоих ФЭУ явно быть не должно. По очереди конечно тоже не будет - это же вероятностный процесс.
Аноним 04/09/15 Птн 12:47:05 #289 №295786 
>>295747>>295753
Ладно, думаю, я в целом на нужном мне уровне уяснил, спасибо.
Переизлучающие атомы -- это прям гюйгенс-френель почти чистый, материализовавшийся.
Аноним 04/09/15 Птн 13:33:37 #290 №295796 
Чём там с нанатрупками, совсем тухло дело? Даже в суперкарах не применяют, сто лет ещё ждать сверхпрочных тканей, тросов, кирпичей для 10километровых зданий?
Аноним 04/09/15 Птн 14:34:59 #291 №295807 
14413664995890.jpg
>>295796
Нанатрубы стали нанатрупами по факту распила грантов - деньги кончились.
Аноним 04/09/15 Птн 15:04:51 #292 №295816 
14413682911970.jpg
Дед химик. При СССР еще в школе преподавал химию. Из списанных реактивов (соли какие-то вроде), он выпарил грамм 200 серебра. Выглядит все это как порошок. Решил мне отдать. Говорит, отнеси к ювелиру, он там тебе переплавит. Но чета очково нести это незнакому мужику. Как и что он там плавить будет и что в итоге замешает вместе с серебром не ясно.

Созрел план. Взять тигель керамический, купить газовую турболампу (пик). Температура плавления серебра около 900 градусов. А вот насчет "зажигалки" это я хз, везде разная инфа о температуре. Поплавит? Там вроде температура высокая. Да и серебро в порошке чистейшее, будет ли оно плавиться без примеси и после затвердевать в нормальный слиток.
Аноним 04/09/15 Птн 16:23:42 #293 №295846 
Два вопроса по электромагнитному излучению:
1. Почему нижняя граница для ЭМ волн это гамма-излучение с длинами 10^-11 м? Почему длина волны не может быть 10^-15 или 10^-20 метра?
2. Верхней границы для длины волны нету? Можно ли сгенерировать волну длиной в видимую часть вселенной?
Аноним 04/09/15 Птн 16:30:10 #294 №295850 
>>295816
Если осадок темный (серый, бывает почти черный - значит примеси оксида/сульфида серебра), то без флюса - тяжело сплавить.
С другой стороны сам флюс (бура - чаще всего), мешает плавить такой слабой горелкой.

Да, используй Гугл, руководств по аффинажу много.
http://www.youtube.com/watch?v=cYfmuK28pmo
Аноним 04/09/15 Птн 16:49:46 #295 №295858 
>>295846
Это всё хуйня и научное фэнтези. Ты же всерьёз не думаешь, что такие микродлины можно как-то реально измерить?
Забей, короче. Физика на 95% - это фэнтези.
Аноним 04/09/15 Птн 17:13:13 #296 №295864 
>>295846
Чем короче длина волны, тем больше энергия у фотона. При 10-11м энергия будет в районе МэВ, и гамма-квант начнет рожать электрон - позитронную пару (масса электрона 0.5МэВ). То есть, гамма с более короткой длиной волны может получиться лишь при взаимодействиях очень энергичных (тяжелых) частиц, и такой гамма-квант в момент пидарасит на производство других частиц.

Длина волны со вселенную теоретически может быть, но тебе нужно время порядка пробега света от одного края вселенной до другого, чтобы ее создать.
Аноним 04/09/15 Птн 17:21:45 #297 №295866 
>>295864
Спасибо, бро. Это познавательно.

>>295858
Что ты делаешь в этом разделе?
Съеби быдло, в \ь\
Аноним 04/09/15 Птн 17:59:15 #298 №295874 
>>295864
А можешь пояснить - такие пары будут даже в вакууме образовываться, без всякого взаимодействия исходного фотона с другими частицами?

Другой анон.
Аноним 04/09/15 Птн 18:06:31 #299 №295877 
14413791918790.jpg
Пожалуйста помогите решить кубическое уравнение, желательно распишите. Препод сказал первым двум кто решит поставит зачет. Я пытался его решить, и там вообщем понятные формулы, но у меня плохо с арифметикой =(

2x^3-x^2-10x+1=0

Нужно что бы были алгебраические корни и уравнение должно быть без i
Аноним 04/09/15 Птн 18:47:58 #300 №295883 
>>295874
В вакууме вроде как сохранение энергии-импульса запрещает распад одиночного фотона на пары - нужен посредник (ядро), уносящее часть импульса.
На самом деле гамма короче 10-11м наблюдается в космических лучах, но при входе в атмосферу ее разносит. Похоже, может разнести и при взаимодействии нескольких фотонов (гамма плюс магнитное поле).
Аноним 04/09/15 Птн 21:01:48 #301 №295906 
>>295883
>нужен посредник (ядро), уносящее часть импульса.
Нихуя. Это происходит исключительно в сильных ЭМ полях. Сам посчитай, какая напреженность поля у ядра атома.
Аноним 04/09/15 Птн 21:57:40 #302 №295912 
В математик-треде меня игнорят, видимо вопросы слишком ньюфанские или еще что, но может тут добрый человек найдется и подскажет как по численному решению Рунге-Кутта определить является ли траектория замкнутой или нет?
Аноним 04/09/15 Птн 23:15:12 #303 №295927 
>>295912
ну вот ты решаешь численно ДУ - и проверяешь, не попала ли твоя точка("вектор состояния") в малую окрестность(погрешность же) исходной точки. Проблема в том, что ты можешь "перелететь" через исходную точку кривой - делаешь проверку по 3 точкам - исходной, текущей и предыдущей - если ты приближаешься к исходной точке - то нужно уменьшать шаг итерации, пока не убедишься что ты попадаешь примерно в исходную точку - либо пролетаешь мимо.
Примерно так я себе представляю.
Аноним 04/09/15 Птн 23:24:34 #304 №295931 
>>295927
дополнение - возможно тебе придется "откатываться" на предыдущую итерацию чтобы уменьшить шаг и пересчитать снова.
Аноним 04/09/15 Птн 23:33:24 #305 №295933 
>>295912
Тебе в 2D траектория, или больше?
Если в 2D, там можно многое сказать про по характеру особых точек.
В 2D можешь провести отрезок, перпендикулярный к траектории, и посмотреть на последовательные пересечения через период. С обоих сторон от предельного цикла будет приближаться к циклу (если он устойчивый).
Аноним 05/09/15 Суб 03:39:00 #306 №295954 
>>295912
Тебе в каком виде нужно, для машины, или глазами посмотреть? Думаю, тебе нужен алгоритм поиска цикла в последовательности. Он устроен так - ты проходишь по последовательности одновременно со скоростью n и 2n. Тогда, если у тебя есть цикл, то рано или поздно точки совпадут.

a_n = initial_point;
a_2n = initial_point;

do {
a_n = next_runge_kutta(a_n);

a_2n = next_runge_kutta(a_2n);
a_2n = next_runge_kutta(a_2n);

} while (dist(a_n, = a_2n) < epsilon)

Где dist - вовзращает расстояние между точками a_n и a_2n.
Аноним 05/09/15 Суб 11:47:14 #307 №296012 
14414428341050.jpg
Говорят пора писать научные статьи, как это делать? Как выбрать тему? Что если знаний не хватит?
II-курс III-семестр.
Аноним 05/09/15 Суб 11:52:59 #308 №296013 
Хочу познать устройство мира (физику). Посоветуйте литературы.
Аноним 05/09/15 Суб 17:50:05 #309 №296024 
>>296013
Советую пройти на хуй, долбаёб.
Аноним 05/09/15 Суб 20:24:05 #310 №296030 
>>296012
Иди в пизду, дебил.

Серьёзно, давайте послыать этих малолетних дебилов. Из таких только апездолов будет полна наука.
Аноним 08/09/15 Втр 18:41:29 #311 №296034 
1. По фазоавой диаграмме воды, при нормальных условиях вода жидкость. Вопрос: какого хуя тогда в воздухе при нормальных условиях есть водяной пар - газообразное состояние воды?
2. Допустим есть максимально огромная твердая планета, но без атмосферы. Вопрос: будет ли на поверхности планеты давление? С одной стороны нету атмосферы ничего не давит, но с другой давление это отношение силы на площадь, а у такой планеты есть сильная гравитация, т.е. хорошая такая сила тяжести на площадь поверхности.
3. Вопрос: Будут ли происходить структурные полиморфные превращения кристаллов(не парамагнетик-ферромагнетик, сверхпроводник-несверхпроводник и т.д. а именно структурные и полиморфные превращения: кварц-кристалоболит, кварц-стекло, жидкость-жидкий кристалл, альфа-железо в гамма-железо и т.д.); аллотропия; смена агрегатного состояния при изменение напряжености электрического, индуктивности магнитного, "хз какая характеристика отвечает за силу воздействия" электромагнитного внешнего поля? Вообще как именно влияют эти поля на вещество? Электрическое создает электричкий ток, т.е. увеличение температуры? Магнитное силовое воздействие, т.е. увеличивает давление? А электромагнитное что тогда? Однавремено увеличение темпераиуры и давления?
Аноним 08/09/15 Втр 19:05:37 #312 №296036 
>>296034
1 это та вода, что еще не выпала на поверхность
2 нет атмосферы - нечего притягивать гравитации
3 не будут
Аноним 08/09/15 Втр 22:35:36 #313 №296063 
14417409367540.jpg
Возникли следующие вопросы:

1. Под цифрами 1 и 2 на пике будет все тот же анандамид с теми же хим. и физ. свойствами?
2. Этот скелет можно выкручивать как только вздумается и будет то же самое вещество?
3. Если да, то это применимо ко всем подобным молекулам с длинным скелетом?

Заранее спасибо
Аноним 09/09/15 Срд 03:26:57 #314 №296083 
14417584177830.jpg
Вот у нас есть ядро атома летящее в пространстве на релятивистских скоростях. Скорость достигла значений при которых масса ядра достигла массы макрообъекта. Встретив скажем звезду, ядро вышла на её орбиту. Насколько это возможно и как бы влиял на планетарную систему такой объект?
Аноним 09/09/15 Срд 05:48:26 #315 №296084 
14417669069190.jpg
>>296034
>какого хуя тогда в воздухе при нормальных условиях есть водяной пар
Водяному пару похуй где быть, хоть в вакууме, хоть в воздухе. Даже с поверхности льда молекулы воды улетат, превращаясь сразу в газ, и этот процесс не зависит от давления. Но от давления зависит другой процесс - превращения газа в жидкость. Именно в этом и должен был бы состоять твой вопрос - как вообще ебаные молекулы могут быть подвижными, и при этом не разлетаться? Честно говоря я и сам не знаю. Если ты посмотришь диаграммы кристализации смесей веществ, то ты увидишь насколько эта хуйня может быть сложной.
Так или иначе, скорость конденсции газа в жидкость/кристалы зависит от давления газа. В том числе жидкая вода может поглощать воздух, но растворимость воздуха пиздец какая низкая, потому вода предпочитает поглощать воду. А еще больше она предпочитает поглощать хлороводород, который с бешенной скоростью в нее впитывается даже при очень низком давлении хлороводорода.
Когда процесс конденсации происходит медленнее, чем испарение - начинается кипение жидкости по всему объему. Но там нихуя не все так просто, потому что даже перегретая жидкость может оставаться жидкой выше температуры кипения, ровно кк и газ может оставаться газом, несмотря на то, что его температура и давление уже давно должны были сделать жидкость.
>Допустим есть максимально огромная твердая планета, но без атмосферы. Вопрос: будет ли на поверхности планеты давление?
Ответ зависит от определений и других условий.
Третий вопрос какой-то ебаный пиздец, ты лезешь в неподъемную для тебя хуйню.
>>296063
>анандамид
Здесь имеет место цис-транс изомерия по двойной связи. 4 группы - 16 возможных конфигураций.
Аноним 09/09/15 Срд 09:02:07 #316 №296085 
>>296034
>Будут ли происходить структурные полиморфные превращения кристаллов(не парамагнетик-ферромагнетик, сверхпроводник-несверхпроводник и т.д. а именно структурные и полиморфные превращения: кварц-кристалоболит, кварц-стекло, жидкость-жидкий кристалл, альфа-железо в гамма-железо и т.д.); аллотропия; смена агрегатного состояния при изменение напряжености электрического, индуктивности магнитного, "хз какая характеристика отвечает за силу воздействия" электромагнитного внешнего поля?

При воздействии постоянными значениями полей - нет. При переменных все зависит от вещества и его природы.

>Вообще как именно влияют эти поля на вещество?
Очень не просто.

>Электрическое создает электричкий ток, т.е. увеличение температуры?
Да температура меняется.
Постоянное только ионизирует.
То есть при каком-то значении за счет "неидеальности" вещества в нем будут образовываться токи, когда начнется процесс изменения структуры доменов в металле, например.
>Магнитное силовое воздействие, т.е. увеличивает давление?
Нет. Если постоянное, то никакого давления не создается. Можешь погуглить "лягушка в магнитном поле."
>А электромагнитное что тогда? Одновремено увеличение температуры и давления?
Все электромагнитные действия на вещество напрямую связаны с его, веществом, способностью поглощать-излучать.
Которую очень непросто исследовать.
Аноним 09/09/15 Срд 09:52:05 #317 №296088 
>>296084
>как вообще ебаные молекулы могут быть подвижными, и при этом не разлетаться?
А в чем вопрос, если они находятся в замкнутом пространстве?
Аноним 09/09/15 Срд 11:43:23 #318 №296093 
>>296088
Я не он, но поясню популярно, для дебилов. При нормальных условиях вода жидкость, НО из за максвеловского распределения скоростей дальний хвост кривой скорости показывает что есть некая часть молекул энергии которых ждостаточно для покидания жидкости.
Если пространство замкнутое, то установится временное равновесие, так как газ может проникать сквозь любую толщину стенок, правда очень и очень долго.
Аноним 09/09/15 Срд 11:54:42 #319 №296095 
Правда, что Гауссовская система единиц в физике уже лет 40 как не используется?
Аноним 09/09/15 Срд 12:09:30 #320 №296096 
>>296088
Какое нахуй замкнутое пространство? Ты читал вообще на что я отвечал?
Аноним 09/09/15 Срд 12:27:34 #321 №296098 
>>296095
В обучени не используется.
В физике очень даже используется, так как более простые законы и нету ебли с магнитными и электрополями.
Аноним 09/09/15 Срд 13:01:46 #322 №296108 
>>296085
Спасибо
>>296084
Спасибо, про замкнутное пространство спрашивал другой анон
>>296034-кун
Аноним 09/09/15 Срд 13:41:07 #323 №296116 
14417952676690.jpg
Объясните про ледники в ледниковый период.
Не очень понимаю что это...
Из школы помню что-то про толщину в километры, ледник надвигается - динозаврам пиздос, какие-то ледниковые озёра, мелкопитающие выжили...

Это просто гигантский, скользящий кусок льда?
Каким хуем он движется?
Стоят динозавры, пасутся, тут хуяк на горизонте лёд?
Из чего он:- это, бля, кусок моря-айсберг, кусок вершины горы, самоконденсируется, что это, бля????
Аноним 09/09/15 Срд 13:42:14 #324 №296117 
>>296093
> есть некая часть молекул энергии которых достаточно для покидания жидкости.
Вопрос был в том - откуда они берутся?
Аноним 09/09/15 Срд 13:46:26 #325 №296120 
>>296117
На все воля хаоса. Это обычная статистика. Половина молекул имеют такую скорость, четверть меньше, четверть больше.
А энергию получают от столкновений друг с другом. Например, стотыщмильенов альфа молекул ВНЕЗАПНО решили хуйнуть все вместе по одной омега молекуле и выбили ее из жидкости аж на орбиту альфа центавра.
Аноним 09/09/15 Срд 13:49:17 #326 №296122 
>>296116
Температура падает из за недостатка парниковых газов в атмосфере.
Из за меньшей температуры испаряется меньше воды, которая парниковый газ.
Так и запускается похолодание.
Почему мньше оды стало испаряться? Причин много, вплоть до того, что слишком много воды испарилось и облака поднялис в стратосферу, из которой не хотели опускаться, и тупо закрыли солнце.
Аноним 09/09/15 Срд 17:40:33 #327 №296181 
>>296098
мы в универе проходили и использовали СГС и слышал, что на практике система СГС юзабельна.
Аноним 09/09/15 Срд 19:14:01 #328 №296218 
>>296083
Чтобы выйти на орбиту ему нужно сбросить скорость (а значит он потеряет массу). Будет влиять также как и любая другая масса - например как комета.
Аноним 09/09/15 Срд 20:31:39 #329 №296228 
>>296218
А если он к примеру сбросит скорость равную половине массы за счёт слияния с идентичным ядром? Или просто, начало его пути началась уже в системе?
Как комета он не будет себя вести т.к. кометы е летают на около-световой скорости. Не распидорасит ли систему при таком спутнике?
Аноним 09/09/15 Срд 20:45:36 #330 №296233 
>>296228
Под "как комета" имелось ввиду влияние массы на систему. Понятно что будет вести по другому (пролетит мимо). Что будет происходить при сбрасывании скорости зависит от способа, скорее всего будет ЕМ волна (если в вакууме произойдет). Наверное что-то вроде ядерного взрыва (если таки получится моментально сбросить скорость). А так он скорее всего будет ебашить сквозь звезды как нейтрино. Время от времени расхуяривая отдельные атомы вызывая небольшую вторичную радиацию на своем пути.
Аноним 10/09/15 Чтв 06:22:33 #331 №296336 
Няши, объясните принцип работы всяких там крутых горелок, турбозажигалок и сварочных аппаратов - таких штуковин, у которых очень горячее пламя локализованное в небольшом пространстве. Мне кажется что если тупо усилить поток газа, то такого эффекта не будет, газ не будет успевать загораться, будет загораться дальше, но так же, без напора, обычным пламенем как от костра.
Аноним 10/09/15 Чтв 14:56:53 #332 №296451 
>>296030
>Иди в пизду, дебил.
>
>Серьёзно, давайте послыать этих малолетних дебилов. Из таких только апездолов будет полна наук
Ну написал бы сразу, что не знаешь, зачем быдлом себя выставлять? Будто это непонятно.
Аноним 10/09/15 Чтв 15:58:48 #333 №296462 
>>296012
Перестать сидеть на дваче, поинтересоваться в универе какие преподы пишут статьи, в какой области и куда - походить к ним, поспрашивать не надо ли им хуй пососать ассистента(кодить/моделировать/считать что-то, верстать статьи, на подсосе чем-то заниматься - в общем рутину разгребать) - скажешь, что заинтересован в публикациях. Желательно еще проявить какие-нибудь скиллы, чтобы показать, что ты не совсем даун с двачом головного мозга.
Аноним 10/09/15 Чтв 16:02:32 #334 №296465 
>>296117
Там всё через высокоуровневую магию Хаоса магию термодинамики и статистической механики получается. Выводятся распределения микро-параметров(к примеру скоростей частиц), по ним моделируются макро-параметры(температура, давление) - из каких-то базовых постулатов(почитай про идеальный газ, к примеру - в рамках школьной программы проходится).
Аноним 11/09/15 Птн 08:23:24 #335 №296673 
14419490049150.jpg
Хай наукач!
Тут такое дело, мне нравятся дирижабли. Я думал о дирижаблях и естественно пришел к мысли, что лучший наполнитель для дирижаблей - ничто (глубокий вакуум).
Ясен пень, что в этом случае нужна прочная оболчка которая возьмет на себя нагрузку.

Так вот вопрос, из каких мемориалов (хотя бы теоретически), на нынешний момент, можно создать сферу в которой поддерживался бы вакуум (10^−5 мм.рт.ст) и она имела бы положительную плавучесть воздухе (20 С у уровня моря.)
И самый главный вопрос, какого тогда она будет размера ? (хотя бы 1 мм в диаметре набереться??)
Аноним 11/09/15 Птн 08:24:09 #336 №296674 
>>296673
материалов.

быстрофикс
Аноним 11/09/15 Птн 08:38:48 #337 №296677 
>>294030 (OP)
Ребят, так уж вышло, готовился к аспирантуре, ессесно учил философию, вспоминал немцев, греков, любимых мною англичан Беркли и Юм в первую очередь и пр. Так вот, отцы эт конечно хорошо, но все же мне интересно узнать про более современную философскую мысль, все таки философия рефлексивна, а значит все знания "отцов" раз сорок перерабатывались и переосмысливались с учетом хода истории. В принципе начал с Мамардашвили, Тоффлера достал с полки. Но у первого все-таки много от марксизма советский философ же, а у второго американская культура мозга, за ее пределами знания плавают.
Может кто посоветует еще каких современных философов в разных направлениях? Лучше конечно из общей философии, но можно и более специальные историософия, социальная философия и пр..
Аноним 11/09/15 Птн 08:45:24 #338 №296679 
>>296673
Используя вакуум вместо (к примеру) гелия твой выигрышь в подьемной силе составит вес этого гелия. Тоесть где-то 180 грам на кубический метр объема баллона. В случае Гинденбурга используя вакуум ты смог бы подымать на 36 тон больше (чем в случае с гелием) - треть от того что он мог подымать (правда он был с водородом, в случае гелия выигрыш был бы незначительно выше). В общем, хотя вакуум даст большую подъемную силу, ебаться с ним мало смысла. Также совсем мало смысла использовать именно глубокий вакуум, если использовать гелий с давлением в несколько раз меньшим атмосферного у глубого вакуума будет выигрыш всего в несколько тонн (в размерах Гинденбурга).
Аноним 11/09/15 Птн 08:56:29 #339 №296682 
>>296679
Бля треть веса это не плохо.
Да и гелий вечно куда то сбывается, а он не дешевый.
Аноним 11/09/15 Птн 10:56:38 #340 №296713 
>>296673
Считай, одна атмосфера давит на один квадратный сантиметр с силой один килограмм. Это максимум который надо выдержать оболочке на уровне моря.
Дальше все просто, чем больше объем тем больше оболочка и тем большее давление на нее оказывается.
Если не умеешь ситать,для гигантов уровня гиденбурга потребуются такие охуительные материалы, что придется изобретать нейтронные материалы чтобы они выдерживалитакую нагрузку.
Аноним 11/09/15 Птн 11:03:21 #341 №296715 
>>295178
Разрыв математика.
Аноним 11/09/15 Птн 13:02:50 #342 №296754 
>>296713
гиденбург умер да и хуй с ним. Время гигантов прошло.

Я думаю насечет микро зондов свободно болтающихся в воздухе и снимающих всю хуйню на свои камеры.
С учетом нынешней миниатюризацией, дирижабль в таком случае нужен не очень то и большой. сантиметров 5-10 в диаметре.
Аноним 11/09/15 Птн 13:08:16 #343 №296757 
>>296715
а теперь, школьник - поразмахивай руками и докажи, что материя фундаментально неделима - начинать можешь с кварков.
Аноним 11/09/15 Птн 13:08:51 #344 №296758 
>>296713
ебать, для 10 см шарика тонна двести.
ебать беать.
Нужен срочно материаловед.
Аноним 11/09/15 Птн 13:09:57 #345 №296759 
>>296758
что видимо и объясняет, почему вакуумом не накачивают дирижабли. Проще потерпеть накладные расходы на газе, чем на материалах.
Аноним 11/09/15 Птн 13:10:27 #346 №296761 
>>296759
вернее на газе и грузоподъемности
Аноним 11/09/15 Птн 13:18:14 #347 №296764 
>>296761
а для 1 сантиметрового всего 12,5 кг.
и при этом он вытеснит 4.18 куб см воздуха.
что будет гдето 1,2 грамма и из этого надо вычесть вес оболочки.
Аноним 11/09/15 Птн 13:42:22 #348 №296770 
>>296764
ты лучше давление считай(а оно будет постоянным = атмосферному-внутреннее) - а не силу на всю площадь. Я вообще не шарю в сопромате - но вроде там нужны удельные величины вроде давления. Дальше смотри характеристики материалов.
Аноним 11/09/15 Птн 15:25:32 #349 №296788 
>>296673
Г.Альтов "Пионерская правда", 20.03.1979. - С.4.
КАК СТАТЬ ИЗОБРЕТАТЕЛЕМ?

Иногда этот вопрос формулируют иначе: "Посмотрите, пожалуйста, мой проект и скажите, получится ли из меня изобретатель?" Проекты обычно бывают очень слабые, но ведь это ни о чём не говорит. В третьем классе меня однажды осенило: а что, если сделать "пустотный" дирижабль? Чем легче газ, которым наполнен дирижабль, тем больше подъёмная сила. Отсюда "гениальная" идея: если внутри дирижабля будет пустота, подъёмная сила получится максимальной. О том, что атмосферное давление раздавит такой дирижабль, я просто не подумал...
Аноним 11/09/15 Птн 15:43:38 #350 №296798 
>>296788
Спасибо анон, порадовал.
Аноним 11/09/15 Птн 18:59:35 #351 №296856 
>>296770
Но ведь давление то и производит всю силу.
Аноним 13/09/15 Вск 21:28:32 #352 №297470 
Привет, ребята. Вопрос вот какой, допустим, есть уравнение поверхности F(x, b)=0. b - вектор параметров. Нет ли какой теоремы или еще чего, чтобы указать, что на некотором множестве x in X, поверхность была выпуклой при этих параметрах?
Аноним 13/09/15 Вск 22:11:29 #353 №297499 
>>297470
Вторые производные посмотреть на постояноство знака?
Аноним 13/09/15 Вск 22:36:55 #354 №297508 
Подскажите где автор, вот этого http://n-t.ru/tp/ng/sf.htm, обосрался?
Аноним 13/09/15 Вск 22:54:44 #355 №297510 
>>297508
А он высказывал там какие-то аргументы? Прочитал пол-статьи, всё какие-то цитаты и тезисы типа:
> должна существовать среда или вещество, в которой находится энергия, после того как она покинула одно тело, но еще не достигла другого...
А-а-а-а-а! Ну раз ДОЛЖНА, то и правда ведь, в пизду эти эксперименты и опыты.
Аноним 13/09/15 Вск 23:20:05 #356 №297518 
Почему если закрыть один глаз и пальцем подвигать другой, то создаётся ощущение, что движется мир вокруг. Но если подвигать тот же глаз "мышцами глаза" то всё норм?
Аноним 13/09/15 Вск 23:23:29 #357 №297519 
>>297518
Потому что существует обратная связь.
Аноним 13/09/15 Вск 23:28:39 #358 №297520 
>>297519
А мозг не в курсах, что я своим же пальцем свой же глаз намеренно двигаю?
Аноним 13/09/15 Вск 23:35:18 #359 №297521 
>>297520
В курсах, ты же не бегаешь в панике от "мир же рушится", а понимаешь, что происходит. Просто обратная связь не такая, гхм, "прямая", а идет через сознание.
Аноним 13/09/15 Вск 23:35:19 #360 №297522 
>>297520
Мозг или сознание?
Аноним 14/09/15 Пнд 15:51:50 #361 №297601 
>>297499
Хм, тоже думал. И это вроде как достаточное условие. Причем, для многомерного случая, скорее, не производные, а проверить положительную определенность матрицы Гессе. Для заданной точки x0 и набора параметров b нужно определить наименьшее собственное число гессиана. А вот как выяснить, будет ли на заданном множестве (т.е. при всех x из этого множества) и заданных b выпукла, не понятно. А вообще, идейка-то у меня была указать критический набор параметров, при которых поверхность перестает быть выпуклой. В общем, ХЗ, может, непонятно написал...
Аноним 14/09/15 Пнд 15:56:51 #362 №297602 
>>296682
Удержать гелий, чтобы он помедленнее съёбывался гораздо дешевле, чем удерживать вакуум от съёба.
Аноним 15/09/15 Втр 08:30:03 #363 №297734 
>>297601
Почитай определение производной и её геометрический/физический смысл. И перестань нести свой псевдонаучный бред.
Аноним 15/09/15 Втр 08:35:49 #364 №297735 
14422953496840.jpg
Аноны, где найти последовательное описание эффекта Мессбауэра с точки зрения квантовой электродинамики? Везде лепят какую-то кашу в виде полуклассического описания, просто пиздец бесит.
Аноним 15/09/15 Втр 11:42:31 #365 №297774 
Почему и в США в Кембридже крутые университеты, и в Англии? Совпадение или нет?
Аноним 15/09/15 Втр 12:07:53 #366 №297779 
Если изменить направление движения тела, безз измения скорости движения, тратится ли на это энергия? Можно ли превратить импульс в энергию и наоборот?
Аноним 15/09/15 Втр 12:09:29 #367 №297782 
>>297774
Они были англосаксами ... Они... ЗАПОЛОНИЛИ... ВСЮ ПЛАНЕТУ!
Аноним 15/09/15 Втр 12:14:15 #368 №297783 
>>297601
очень просто же - матрица Гессе в общем виде будет зависеть от x и b - подставляй b, проверяй положительную определенность. Либо запиши условие положительной опр-ти для любого x и решай уравнение для b.
Аноним 15/09/15 Втр 12:33:21 #369 №297786 
Эндорифин по химической структуре близок к опиатам или опиаты близки по химической структуре к эндорфину?
Аноним 15/09/15 Втр 12:58:39 #370 №297790 
>>297786
Таки симетг'ично. Опиаты хорошо "эмулируют" т.н. перетяжку(важная функциональная группа, на которую завязан рецептор, если не изменяет память). Есть одно но эндорфины - пептиды, а опиаты(натуральные которые) - сложные соединения с циклическим скелетом.
Аноним 15/09/15 Втр 13:30:56 #371 №297800 
>>297786
Эндорфины - высокомолекулярные органические соединения. Опиаты - маленькие молекулы, химическая активность которых совпадает с активностью эндорфинов, т.е., с активностью их фрагментов, которые и "бьют" в рецептор.
Аноним 15/09/15 Втр 14:43:24 #372 №297826 
Что такое гомогенная катастрофа? Объясните подробнее, пожалуйста.
Аноним 15/09/15 Втр 15:58:26 #373 №297859 
У меня вопрос такой колом стоит. Если фотон не имеет массы, то каким образом он может придавать импульс объекту на который падает? Имей хоть какую скорость при нолевой массе, в итоге получаем ноль. Или я пиздоглазое мудило и не правильно читал?
Аноним 15/09/15 Втр 16:15:17 #374 №297866 
>>297859
Просто ты думаешь, что импульс определяется как mv, а это не так, это упрощение.
Аноним 15/09/15 Втр 16:15:32 #375 №297867 
>>297859
У фотона нет массы покоя (что значит, что его как покоящейся частицы просто не существует). А в движении масса есть, хоть и близкая к нулю.
Аноним 15/09/15 Втр 16:16:08 #376 №297868 
>>297866
Альтернативщик-торсионщик, плиз.
Аноним 15/09/15 Втр 16:18:56 #377 №297871 
>>297734
В каком месте я написал что-то, что ты считаешь бредом?
>>297783
Для отдельных точек x и заданного набора b это как раз про то, что я говорил. А вот с записью для любого икс как раз проблема. Чтоб понятнее было, приведу свой урезанный пример:
x = {xi}, i=1..4
yi=xi, i=1..3
y4=x3+x4.
X_i = const
f(y)=sumibii (yi/Xi)2 + sumisumj 2bij (yi/Xi)2 (yj/Xj)2
Аноним 15/09/15 Втр 16:20:00 #378 №297872 
>>297871
f(y)= ... -1
selffix
Аноним 15/09/15 Втр 16:21:09 #379 №297873 
>>297868
https://ru.wikipedia.org/wiki/Интегралы_движения
Аноним 15/09/15 Втр 16:48:27 #380 №297879 
>>297873
Матанщик-хардкорщик, плиз.
Аноним 15/09/15 Втр 17:08:32 #381 №297880 
Микроскопа на 1280 крат хватит для работы с чашкой петри и взращивания колоний разнообразных бактерий и грибков?
Такой подойдёт? http://www.levenhuk.ru/katalog/mikroskopy/levenhuk-50l/
P.S. школьник, хочу поиграть в биолога.
Аноним 15/09/15 Втр 17:11:24 #382 №297881 
Реквестирую годной литературы по аэродинамике и практике авиастроения.
Аноним 15/09/15 Втр 17:15:36 #383 №297883 
>>297880
Вполне подойдет.
Увеличение свыше 1000 на оптических микроскопах бессмысленно из-за дифракционного предела. Критично на самом деле качество оптики.
Аноним 15/09/15 Втр 17:42:21 #384 №297889 
>>297883
А какой нужен микроскоп, чтобы увидеть квантовую неопределенность?
Аноним 15/09/15 Втр 17:43:14 #385 №297890 
>>297873 >>297866 >>297867

Это анон с фотоном, учу физику по википедии, и вот что я не могу понять:
Фотоны. Единственная вполне достоверно существующая безмассовая частица....что даёт верхнюю оценку массы фотона 3,5*10^-60 грамм.
https://ru.wikipedia.org/wiki/%D0%91%D0%B5%D0%B7%D0%BC%D0%B0%D1%81%D1%81%D0%BE%D0%B2%D1%8B%D0%B5_%D1%87%D0%B0%D1%81%D1%82%D0%B8%D1%86%D1%8B
И следом
Постоянно повышается точность проводимых экспериментов по определению массы и заряда фотонов. Обнаружение хоть сколько-нибудь малой величины заряда или массы у фотонов нанесло бы серьёзный удар по Стандартной модели. Все эксперименты, проведённые до сих пор, показывают, что у фотонов нет ни заряда, ни массы покоя.
https://ru.wikipedia.org/wiki/%D0%A4%D0%BE%D1%82%D0%BE%D0%BD
Либо лыжи не едут, либо это такой хитрый финт ушами, что давайте думать, что масса это просто накопленная энергия и таким образом когда фотон несёт энергию, он переносит и массу. Еще из этого следует, что система излучающая теряет массу, а система поглощающая массу накапливает, но вот я сижу весь окруженный светом, у меня черные волосы на голове, то есть они поглощают много света, а жирнее они от этого за все мои годы не стали.
Вопрос знатокам, что за хурма?
Аноним 15/09/15 Втр 17:44:54 #386 №297893 
>>297890
UPD Просто я понимаю, что я что-то не понимаю, но не понимаю, что именно.
Аноним 15/09/15 Втр 17:52:20 #387 №297895 
>>297735
https://vk.com/doc59661443_418816070?hash=9ae954f25f8d2c0d24&dl=bc1cacfc592d103e3b только презу могу дать не моя думаю врядли поможет
>>297779
Законы сохранения энергии и импульса независимы.
Аноним 15/09/15 Втр 17:54:11 #388 №297896 
>>297890
В чем вопрос то? Почему твои волосы жирнее не стали?
Аноним 15/09/15 Втр 18:23:17 #389 №297900 
>>297896
Тут вообще один большой вопрос всеобщего непонимания ситуации, который описывается выражением "Что за херня тут происходит?" Но постараюсь в силу возможностей разбить на подвопросы.
Фотон состоит из чего нибудь или это просто фотон сам по себе и ни на что больше не делится? Далее, как в нём запасается энергия? И наконец, самое главное как же он её передаёт то? Он при соприкосновении с объектом пропадает, как я понял, то есть совсем, исчезает, был и нет его, прав я в этом или нет, так вот как он механически свою энергию передаёт, а тем более передаёт массу, если как только он о что-то тормозит, то исчезает?
Надеюсь на твоё терпение анон.
Аноним 15/09/15 Втр 19:21:41 #390 №297913 
>>297826
Бамп.
Аноним 15/09/15 Втр 21:47:05 #391 №297934 
>Законы сохранения энергии и импульса независимы.
>>297895
А какая тогда разница между импульсом, моментом импульса и энергией? Ведь все три являются мерой количества движения.
Аноним 15/09/15 Втр 22:01:06 #392 №297936 
>>297934
Гугли теорему Нётер.
Алсо, энергия - скалярная величина; импульс, момент импульса - векторные.
Импульс - мера механического движения.
Момент импульса - мера вращательного движения и завист от импульса.
Аноним 15/09/15 Втр 22:18:09 #393 №297939 
>>297900
> механически
Забудь это слово. Все твои проблемы с пониманием проистекают из того, что ты пытаешься у себя в голове нарисовать какие-то шестерёнки, сталкивающиеся мячики, волчки и хуй знает что ещё. Физика элементарных частиц так не работает. Она имеет дело не с механическими моделями, а с математическими. Вообще, первичным физическим объектом является поле, а не частица, и полевая картина мира куда более адекватна реальности. Есть конечное число фундаментальных полей, пронизывающих всё пространство и взаимодействующих друг с другом. А частицы - это просто порции этих полей, которые не делятся
>>297895
>Законы сохранения энергии и импульса независимы.
В классике разве что. В релятивизме временные и пространственные трансляции взаимосвязаны, и нельзя иметь однородное время и неоднородное пространство, равно как и наоборот. Всё это частные случаи лоренц-ковариантности.
Аноним 15/09/15 Втр 22:21:55 #394 №297941 
>>297939
Релятивист в треде, скорее в калаби-яу.

Доказал уже свою теорию относительности и квантовой механики?

>Вообще, первичным физическим объектом является поле, а не частица, и полевая картина мира куда более адекватна реальности. сть конечное число фундаментальных полей, пронизывающих всё пространство и взаимодействующих друг с другом. А частицы - это просто порции этих полей, которые не делятся
Просто пушка. Еще и поехавший феласаф.
Аноним 15/09/15 Втр 22:39:14 #395 №297945 
>>297941
Что сказать-то хотел? Если ты ищешь, кого бы потралить кефиром, можешь проходить мимо.
Аноним 15/09/15 Втр 22:47:43 #396 №297946 
>>297945
С началом учебного года даже качество тралинга заметно упало. Рак снова заполнил интернеты.
Аноним 15/09/15 Втр 23:07:12 #397 №297947 
>>297946
То есть из этого вытекает, что ты составляешь лимит от школьника до идеального тролля?
Аноним 15/09/15 Втр 23:23:39 #398 №297950 
А с IOPscience скайхаб статьи не выдирает?
Аноним 15/09/15 Втр 23:30:14 #399 №297951 
>>297939
То есть в физике элементарных частиц мне не то, что не нужно, а нельзя ничего воображать из взаимодействия макромира, а нужно только представлять отношение одного к другому?
Аноним 15/09/15 Втр 23:30:47 #400 №297953 
14423490475610.jpg
Мало ли, вдруг найдутся геологи/географы.

Перевожу статью по изменению климата и таянию льдов, не могу найти однозначного аналога термину Meltwater Pulse (глобальное повышение уровня моря вследствие таяния ледников после окончания последнего ледникового периода). Никакого аналога подобного термина в русской литературе по гляциологии я не нашел.
Есть ли он вообще?
https://en.wikipedia.org/wiki/Meltwater_pulse_1A
Аноним 15/09/15 Втр 23:41:28 #401 №297954 
>>297953
Таяние (растаяние), точка оттаяния.
Аноним 15/09/15 Втр 23:42:08 #402 №297955 
>>297953
Так и переведи - "Таяние ледников".
Мимополовойорган
Аноним 15/09/15 Втр 23:48:14 #403 №297958 
>>297954
>>297955
Не, хуйня. Не сообщающий ничего о сути явления и не входящий в номенклатуру научных терминов, такой перевод ущербен. Просто переведу как повышение уровня моря, вызваного талыми водами.

Спасибо за отлик в любом случае.
Аноним 15/09/15 Втр 23:48:16 #404 №297959 
>>297955
Точнее даже "Период таяния ледников"
самофикс
Аноним 15/09/15 Втр 23:53:22 #405 №297961 
>>297958
>Просто переведу как повышение уровня моря, вызваного талыми водами.
Имплаинг талая вода может взяться ещё откуда-то кроме ледников.
Хозяин-барин в общем.
>Спасибо за отлик в любом случае.
Всегда пожалуйста.
Аноним 15/09/15 Втр 23:54:39 #406 №297962 
>>297958

Ну скажи, что Таяние ледников в такой-то геологический период.
Аноним 16/09/15 Срд 13:26:47 #407 №298018 
Двач помогай.

1.
Самка медоносной пчелы Apis mellifera диплоидна, а трутни гаплоидны. Существует, однако локус со множественными аллелями, влияющий на опредедение пола у диплоидных особей. Все диплоидные
особи, гетерозиготные по этому локусу,-самки, а гомозиготные по любому из аллелей - самцы. Гомозиготные сацы не достигают половозрелости, поскольку обычно поедаются рабочими пчелами на личиночной стадии в первые трое суток после вылупления из яичек. Таким образом, гомозиготный генотип в естественных условиях летален. Однако изъятых из улья личинок таких самцов можно довести до стадии половозрелости. В этом локусе насчитывается около двадцати аллелей, обозначаемых символами A1, A2, ... ,A20. Пусть в потомстве некоторой оплодотворенной гаплоидным трутнем матки лишь половина яичек способна к развитию (это хорошо заметно, поскольку половина ячеек в сотах остается пустой, и пустые ячейки распределены по сотам случайным образом). Что можно сказать о генетической конституции матки и трутня?
1.
>лишь половина яичек способна к развитию
Если по условию, гомозиготные особи сжираются то, получается 1 половина будет гомозиготной и сжираться, а вторая состоять из гомозиготных самок(их других быть не может) и гаплойдных трутней? Самка не может быть гомозеготной, я так понял. У самки Aa будет всегда. А или а или AA/aa(если он выжил) у самца равносильна для соотношения полов и значения не имеет, вроде так.

2.
Диплоидный трутень оплодотворил матку, у которой в локусе, определяющем пол, один из аллелей идентичен аллелю трутня. Какой процент потомства будет жизнеспособен в улье?
> один из аллелей самки идентичен аллелю трутня
Не бывает гомозиготных самок(все самки Aa) и гетерозиготных самцов(все самцы либо A,a или если они диплойдны AA или aa) , получается что один из аллелей самки всегда будет равен ДВУМ аллелям диплойдного самца, они же у него одинаковые. Или я что то не так понял? Зачем это неполное уточнение, запутать?
>Какой процент потомства будет жизнеспособен в улье?
Те кого не сожрут, то есть гетерозигогтные особи(самки) и гаплойдные самцы.
Aa(самка) x AA/aa( диплойдный самец)

Получается 1:1, 50%. Это типа реверс задачи? А как определить сколько будет гаплойдных трутней или на них по условию похуй?

Чёт ваще пиздец, читал вроде всё ясно понятно было, а как к задачам перешёл, залип нахуй, еще и ответов нет.
Аноним 16/09/15 Срд 13:29:19 #408 №298019 
>>298018
>а вторая состоять из ГЕТЕРОзиготных самок*
Аноним 16/09/15 Срд 15:13:07 #409 №298029 
http://lenta.ru/news/2012/07/20/exoticdwarf/
Этот гелий чем-нибудь по свойствам отличается от обычного гелия? И можно ли получить гелий из статьи повышая давления(до миллиона или миллиарда паскаль) и изменением температуры?
Аноним 16/09/15 Срд 16:01:37 #410 №298038 
>>297945
Скзать хотел что квантовая мехника говное ебаное, которое как сдело в разряде теорий так и сидит. Выродилась вся физика в ебаную математику.
А всякие коты шредингера, неопределенности гейзенберга исходят исключительно из за конечности заряда как электрона так и ЭМИ поля.
Аноним 16/09/15 Срд 16:03:23 #411 №298039 
>>297951
В нынешней теори, заметь теории а не аксиоме, все описывается исключительно математикой. Физеки такую хуйню нагородили что сами представить не могут что там и как взаимодействует, забывая про то что природе на числа глубоко похуй и она работает без них.
Аноним 16/09/15 Срд 16:04:40 #412 №298040 
>>297958
Расплавоводная точка
Аноним 16/09/15 Срд 16:06:12 #413 №298041 
>>298029
Не отличается, при одинаковой температуре, объеме и давлении.
Можно. Но невозможно из за отсутствия сейчас тких охуительных материаловспособных выдержать такие температуры и давления.
Аноним 16/09/15 Срд 16:10:20 #414 №298043 
Канторович Л.В., Акилов Г.П. Функциональный анализ - норм, или не стоит его заказывать, есть книги по-лучше?
Залил на рыгхост на всякий http://rghost.ru/7FD4lVZFm
Аноним 16/09/15 Срд 16:10:54 #415 №298044 
>>298043
Тредом ошибся сорри
Аноним 16/09/15 Срд 17:36:09 #416 №298055 
>>298039
На твоё воображение природе ещё более похуй. Математика отражает реальные объекты и связи между ними, а интуиция постоянно фейлит.
Аноним 16/09/15 Срд 17:38:55 #417 №298056 
14424143354840.png
>>298038
Авторитетное мнение школьника ИТТ. Конечно же, нам очень важно было его услышать.
Аноним 16/09/15 Срд 20:01:23 #418 №298085 
>>298039
Снова ты, мудила со школьным образованием?
Аноним 16/09/15 Срд 20:02:31 #419 №298086 
>>298083 (OP)
О да, настолько реальные, что волновая функция это пик квантовой мысли.

>>298056
А вот и студенты подъехали. Поспоришь с этим, хочешь сказать?
>А всякие коты шредингера, неопределенности гейзенберга исходят исключительно из за конечности заряда как электрона так и ЭМИ поля.
Аноним 16/09/15 Срд 20:10:17 #420 №298090 
>>298085
Пальцем в небо.
Бывший студент электротехник.
Аноним 16/09/15 Срд 20:22:10 #421 №298091 
14424241304930.jpg
Не хочу создавать отдельный тред. Лучше напишу здесь.
По поводу пикрелейтед. Кто может дополнить список.
Давайте запилим еще образовательной инфографики для повышения математической культуры.
Аноним 16/09/15 Срд 20:24:28 #422 №298092 
>>298086
Чем по жизни занимаешься?
Аноним 16/09/15 Срд 20:26:12 #423 №298093 
>>298091
Зачем? Сомневаюсь что кто то с двачей в доскональности знает хотя бы базу всего того что ты надрочил в списочек. Я так точно не знаю ничего с пикчи.
Аноним 16/09/15 Срд 20:26:57 #424 №298094 
>>298092
Пики точены или хуи дрочены?
Аноним 16/09/15 Срд 20:42:38 #425 №298099 
>>298094
Интересно, откуда вы, долбоебы, лезете.
Так чем ты занят по жизни, что заставляет тебя лезть на саентач в конце рабочего дня и демонстрировать своё невежество?
Аноним 16/09/15 Срд 20:45:22 #426 №298102 
>>298091
Гомотопическая теория типов
Аноним 16/09/15 Срд 20:48:00 #427 №298105 
>>298099
Отсутствие девченки и понимае того, что квантовая механика теоритическое говно ебаное.
Аноним 16/09/15 Срд 20:54:57 #428 №298108 
>>298093
>Зачем?
Для повышения престижа профессии, для популяризации науки среди населения, для несения знания в массы.
Аноним 16/09/15 Срд 20:55:34 #429 №298109 
>>298105
и не знание русской орфографии
Аноним 16/09/15 Срд 20:59:47 #430 №298111 
14424263879320.jpg
>>298108
Ну, математика, особенно высшая оторвна от реальности практически поностью. С этим спорить себе дороже. Да и знание гомотопической алгебры склеить вместе два проводка не сильно поможет.
Тобишь, она не нужна практически, сейчас имеются более серьезные проблемы чем отсутствие доказательства аррациональности числа п в степени е.
ИМХО.
Аноним 16/09/15 Срд 21:01:14 #431 №298114 
>>298109
Может это потому что я хохол, не, мелкобуквенный?
Аноним 16/09/15 Срд 21:01:16 #432 №298115 
>>298105
Так и чем ты занимаешься? Ты учишься? На кого или тебя выпиздили? Работаешь, кем? Ты себя не сдеанонишь, а мне сэкономишь время.
Аноним 16/09/15 Срд 21:06:22 #433 №298118 
14424267828130.jpg
>>298086
Тут не с чем спорить, это просто безграмотный голословный высер уровня гастронома на улице Герцена. Ты не знаешь, что такое заряд и откуда он берётся, и ты не знаешь, что такое принцип неопределённости и как он работает. Ты вообще нихуя не знаешь, но везде лезешь. Типичный школьник. Впрочем, если ты хотя бы попытаешься обосновать свою хуету, я, так и быть, снизойду до указания тебе, где конкретно ты обосрался.
Аноним 16/09/15 Срд 21:07:14 #434 №298119 
>>298115
Безвольный сыч, которого выгнали из универа, который ничем не занимается и теряет время сидя на дваче. Ты это хочешь услшать?
Аноним 16/09/15 Срд 21:08:30 #435 №298121 
>>298118
Ясно.
По существу сказать больше нечего?
Аноним 16/09/15 Срд 21:12:10 #436 №298122 
>>298119
Нет, я хотел узнать, имеешь ли ты какое-то отношение к квантам. Видимо нет, значит терять время на спор с тобой бессмысленно. Вместо того чтобы выебываться попытайся понять и задать осмысленные вопросы, ну или хоть как то аргументировать позицию. На такую толстоту нет никакого желания отвечать, а лично я кванты нежно люблю.
Аноним 16/09/15 Срд 21:13:30 #437 №298123 
>>298121
Дай-ка я тебе объясню суть дискуссий в приличном обществе, мань. Вот представь, сидят благородные джентльмены в смокингах, обсуждают квантовую физику. Тут в комнату влетает перемазанный говном дурак и начинает вопить: "ВЫ ВСЕ УЁБКИ ТУПЫЕ НА САМОМ ДЕЛЕ ЛЮДИ ХОДЯТ ПО ПОТОЛКУ Я ПРАВ А ВЫ НЕТ ДОКАЖИТЕ ЧТО ЭТО НЕ ТАК". Предположим на секунду, что эти джентльмены не имеют опыта интеракции с дураками, и начинают ему на полном серьёзе объяснять, почему он заблуждается. Дурак же, даже не дослушав объяснения, продолжает нести хуйню: "А ЕЩЁ БУМАГА - ЭТО СОПЛИ БРОНТОЗАВРА, БАТАРЕИ - ЗАГОВОР БАКАЛЕЙЩИКОВ, МОЙ АНУС - ВЕЧНЫЙ ДВИГАТЕЛЬ, А МЕТРО - ХРАМ САТАНЫ. АРГУМЕНТИРУЙТЕ ИЛИ СОСНУЛИ". А следом за ним в дверь вламывается ещё сотня таких же юродивых, которые так же начинают носиться по залу, пачкая говном интерьеры и генерируя словесный понос. Работа клуба оказывается намертво парализованной, потому что любой умственно неполноценный может выдавать бредовые утверждения с огромной скоростью, а опровержение по всем правилам - вещь трудоёмкая. Да он и не слушает опровержений, ему главное - привлечь к себе внимание. Поэтому, сынок, бремя доказательства лежит на утверждающем. Если ты соизволил высрать что-то, ты должен это аргументировать, иначе тебя просто пошлют нахуй и будут правы.
Аноним 16/09/15 Срд 21:14:19 #438 №298124 
>>298122
Еще один теоретик, кандидат на Нобеля. Как мило, двач все ещё остается бордой успешных людей.
Аноним 16/09/15 Срд 21:31:28 #439 №298127 
>>298124
Я экспериментатор.
Аноним 16/09/15 Срд 21:38:54 #440 №298129 
>>298123
Зайду издалека.

Сила действия магнитного поля на ток, тобишь на движущиеся электроны, в пределе электрон, равна
F=B(I_1L_1)(I_2L_2)/d^2
где
I сила тока в проводниках
L их длинна
d расстояние между ними
B магнитная постоянна равная 10^-7 округленно, чтобы даже ты смог посчитть в дальнейшем то из нее вылезет без особых напряжений для мозга

По закону Кулона сила действующая между двумя зарядами равна
F=DQ_1Q_2/d^2
где
Q заряды
d расстояние между ними
D электрическая постоянная, которая тоже округленно равна 9 умножить 10^9

Величины D и B разные. Связи между ними не существует, пока мы не замечаем что изменяющееся магнитное поле порождает изменяющееся электрическое.
Максвел, которого я сейчас практически дословно пересказываю, предположил что изменяющееся электрическое поле порождает изменяющееся магнитное.
При переменном токе получается что изменяющееся электрическое поле изменяется в точности так же как и магнитное. Эти изменения должны распространятся в виде волны, при чем изменение одного поля должны вызывать изменения другого поля.
Из этого следует, что значения D и B связанны.
Исходя из дроби
D/B
мы получаем число 9 умножить10^16 с размерностью (ньютон на метр квадратный деленный на кулон квадратный)/(ньютон/ампер квадратный) или метр квадратный/секунду квадратную. Это размерность квадрата СКОРОСТИ. Тобишь корень из этого результата должен отражать скорость распространения волны полученной в результате изменения электрического и магнитного полей.
Извлекая корень из этого числа получаем 3 умножить 10^8 метров деленных на секунду.
Для самых маленьких, это скорость света. Максвел на двух постоянных и одном логическом выводе открыл ЭМИ и с высокой точностью вычислил ихнее значение скорости.
Аноним 16/09/15 Срд 21:39:24 #441 №298130 
>>298129
Тут я надеюсь выродкам-экспериментаторам с двача все понятно?
Аноним 16/09/15 Срд 22:00:01 #442 №298146 
>>298129
> поля на ток, тобишь на движущиеся электроны, в пределе электрон
Бгг

> Максвел, которого я сейчас практически дословно пересказываю
Не, ты не видел работы Максвелла, я сомневаюсь, что ты уравнения его можешь записать.

> предположил что изменяющееся электрическое поле порождает изменяющееся магнитное
Лол. В уравнения Максвелла входит закон Фарадея. И как же иначе мы обнаружили бы магнетизм, если он порождается исключительно движением заряженных частиц и магнитное поле нивелируется движением системы отсчета?

> Эти изменения должны распространятся в виде волны, при чем изменение одного поля должны вызывать изменения другого поля.
Ох нихуя ж себе, откуда же это выходит? Может из уравнений Максвелла? И каким же образом?

> Тобишь корень из этого результата должен отражать
Корень что-то там должен только в голове ебанашки, примерно такого как ты.

> открыл ЭМИ
Мне интересно, как расшифровывается аббревиатура ЭМИ, по-твоему?

> Тут я надеюсь выродкам-экспериментаторам с двача все понятно?
Чего сказать то хотел, дурачина?
Аноним 16/09/15 Срд 22:01:11 #443 №298147 
>>298111
>математика, особенно высшая оторвна от реальности практически полностью
Что по моему мнению есть хорошо, ибо высокий порог вхождения способствует отсеиванию быдла.
Но если мы сможем заинтересовать хотя бы нескольких пытливых анонов, то это будет большим достижением, очень хорошим.
Частенько вижу тут ракотреды типа: я тупой гуманитарий, но хачу угоротеть по математеке, памагите, с чиво начать.
Аноним 16/09/15 Срд 22:03:49 #444 №298150 
>>298146
Понятно, что экспериментатору нихуя не понятно.
Зоконы Максвела сюда приклеил, Фарадея, предположения у него не предположения, скорость свет теоритически не открыть, охуеть.
Но я продолжу.
Аноним 16/09/15 Срд 22:09:11 #445 №298151 
>>298150
Нет, это ты не понимаешь, и даже не представляешь как на основе уравнений Максвелла можно получить какие-то волны, лол.

Про закон Фарадея слыхал хоть? Продолжай, а мы посмеемся над ебанашкой.
Аноним 16/09/15 Срд 22:42:37 #446 №298162 
>>298151
В дальнейшем развитии исследований электромагнитных волн ученые, в говне моченые, пришли к выводу что весь спектр исзлучения можно выразить в виде уравнения о температуре абсолютно черного тела. Догадка проста, что хорошо поглощает ЭМИ, то хорошо излучает ЭМИ.
На основе этого моченые пришли к двум следующим выводам
1. Закон Стефана, который говорит что полная энергия излучения примерно равна значению температуры в четвертой степени
2. Закон Вина, длинна волны λ на которую приходится максимальная энергия излучения, пропорциональна абсолютной температуре тела.
Эти два вывода, теперь уже закона, вполне справедливы, но при рассчетах замкнутых систем из них полезла ошибка.
А именно, если взять спичку/свечку/печку/что либо еще, то получается что мощность излучения тела должна быстрым юзом уходить в коротковолновую часть спектра, или даже дальше, в идеале в жесткое гамма излучение. Естественно в опытах ничего такого не было и все подобные тела излучали на уровне оранжевых волн, или около того . Можно было сказать что эти два закона ошибочн и не нужны.
Однако, на длинноволновом конце спектра рассчеты этих двух законов вполне себе кореллровали с опытом.
Тогда на сцену вышел герр Планк. Он задался вопросм, что надо минимально изменить в теории, чтобы помирить длинные и короткие волны.
В процессе думы он заметил что подобные результаты зависят исклюительно от длинны волны. Исходя из этого он предположил, выделю для особо одаренных ПРЕДПОЛОЖИЛ, не вывел из каких то уравнений, не списал у какого то другого физика, а сидя в лаборатории за столом, с карандашиком и бумажкой, ПРЕДПОЛОЖИЛ, что энергия излучения должна передаватся упакованной в строгих количествах, разных для разных длинн волн, маленькие для длинных волн и большие для коротких.
Из такого предположения получается, что энергия квантов должна равнятся некому значению энергии умноженному на длинну волны, тобишь E=hv, где h равно 6.6умножить10^-34 джоулей в секунду.
Так и открыли квантование электромагнитного излучения.
Аноним 16/09/15 Срд 22:56:51 #447 №298167 
14424334116850.jpg
>>294030 (OP)
Ребята, вопрос по философии.
Всё имеющееся, или Универсум, как абсолют/абсолютная категория действительно ли предполагает вбирание в себя всего мыслимого и немыслимого, или же ограничено областью нашего понимания и включает в себя всё, что мы способны охватить своим разумом?
Спасибо.
Аноним 16/09/15 Срд 23:11:52 #448 №298169 
>>298167
Вопрос уровня сможет ли бох создать неподъемный камень.
Человек, например, смог охватить разумом чило гремма, но в наблюдаемую вселенную оно не влезет, если каждый знак числа впихивать в планковский объем.
И да, плюнь в глаза тому кто скажет что может объять необъятное.
Аноним 17/09/15 Чтв 00:06:16 #449 №298173 
>>298169
Не влезет в наблюдаемую вселенную не число Грэма, а его ДЕСЯТИЧНОЕ ПРЕДСТАВЛЕНИЕ. Это разные вещи. Ни одно представление абстрактного понятия числа априори не лучше другого. Конкретный выбор осуществляется из соображений удобства в данном конкретном случае. Грэм как раз нашёл очень компактное представление этого числа.
Аноним 17/09/15 Чтв 05:05:20 #450 №298196 
>>298167
Еще деда Парменид утверждал, что всё есть бытие(даже мысли и фантазии), небытия нет.
Так что Универсум - это просто всё. Впрочем, понятие Универсума может зависеть от контекста(взять теорию множеств, к примеру ;).
Аноним 17/09/15 Чтв 05:21:51 #451 №298199 
вот есть лошади, есть пони, а есть мини-пони размером с кота. вопрос: почему никто до сих пор не вывел мини-мышей размером с таракана? млекопитающие размером с насекомое, это же ОХУИТЕЛЬНО.
Аноним 17/09/15 Чтв 08:32:32 #452 №298207 
>>298162
> ученые, в говне моченые
Пока я вижу говно только в одном месте - в твоей голове.

> можно выразить в виде уравнения о температуре абсолютно черного тела
Как же выглядит это уравнение?

> мощность излучения тела должна быстрым юзом уходить в коротковолновую часть спектра, или даже дальше, в идеале в жесткое гамма излучение
Мощность излучения должна уходить... что ты несешь, ебанутый? Это изложение проблемы ультрафиолетовой катастрофы от дауна? Ну и головка горящей спички не является абсолютно черным телом, лол.

> Так и открыли квантование электромагнитного излучения.
Чего сказать то хотел, ебанашка?
Аноним 17/09/15 Чтв 09:16:59 #453 №298213 
>>297735

Дай бог памяти, в Гольданском хорошо описано. Книга называется "Химическое применение Мессбауэровской спектроскопии", но я не уверен, что она есть в оцифрованном виде.
Аноним 17/09/15 Чтв 09:19:01 #454 №298214 
>>298086

Ты даже физический вуз не закончил, это видно из того, что ты, судя по высказыванию о волновой функции, даже не знаком с матрицей плотности. О чем с тобой вообще можно дискутировать?
Аноним 17/09/15 Чтв 09:23:55 #455 №298215 
>>298199
https://ru.m.wikipedia.org/wiki/%D0%9A%D0%B0%D1%80%D0%BB%D0%B8%D0%BA%D0%BE%D0%B2%D0%B0%D1%8F_%D0%B1%D0%B5%D0%BB%D0%BE%D0%B7%D1%83%D0%B1%D0%BA%D0%B0
Аноним 17/09/15 Чтв 09:24:57 #456 №298216 
>>298214
О матрице плотности.
Аноним 17/09/15 Чтв 09:44:12 #457 №298223 
>>298214
Ну да ну да, электрона в атоме водорода нигде нету, есть только вероятность его нахождения.
Аноним 17/09/15 Чтв 12:26:31 #458 №298294 
>>298223
С чего ты взял, что электрон обязан "быть где-то"? К квантовым системам вообще неприменимо понятие положения.
Аноним 17/09/15 Чтв 12:45:53 #459 №298317 
>>298294
Применимо, но не "механически", да, а как абстрактное (математическое) свойство локализованности наблюдения изменения состояния квантовой системы, включающей сенсор прибора регистрации с информацией о его положении.
Аноним 17/09/15 Чтв 13:05:53 #460 №298332 
14424843535260.jpg
>>294030 (OP)
Приветствую научных господ. Есть одна идея как заработать денег. Чтобы понять взлетит она или нет, нужно решить задачу по физике уровня начальной школы, а я настолько даун в этом деле, что без анона не справлюсь.
Итак, нужно посчитать количество джоулей необходимое для того чтобы нагреть 20кг меди от 20 до 40 градусов цельсия и поддерживать эту температуру 1 час.
Аноним 17/09/15 Чтв 13:20:31 #461 №298342 
>>298332
И откуда деньги появятся?
Аноним 17/09/15 Чтв 13:28:41 #462 №298346 
>>298342
Их принесут сотни тысяч клиентов конечно же.
Аноним 17/09/15 Чтв 15:07:55 #463 №298389 
14424916752790.jpg
>>294030 (OP)
Абсолютно ли все объекты, существующие и гипотетические, можно отнести к категории вещей? Спасибо.
Аноним 17/09/15 Чтв 17:00:16 #464 №298454 
Помогите вспомнить название народности.
Знаю, что они с каких островов, не маори.
Вроде слышал, что среди этой народности много парней играет в лиге американского футбола, потому, что они от природы здоровые.
Аноним 17/09/15 Чтв 17:19:30 #465 №298458 
>>298389
лат. objectum — предмет
Аноним 17/09/15 Чтв 17:39:42 #466 №298465 
>>298389
Это вопрос по языковедению?
Аноним 17/09/15 Чтв 19:01:56 #467 №298502 
Анончики, я мимо хуй в науке, но хуй любопытный. В общем, через что нужно пройти, чтобы понимать о чём современная физика? Всякие теории струн, квантовые механики и т.п. Т.е. чтобы разобраться в уравнениях и сложить некую целостную картину о чём идёт речь (да-да, тупой школьник).
Аноним 17/09/15 Чтв 19:12:26 #468 №298503 
>>298502
"Привет чуваки. Я дебил с ветром в голове. Нука объясните мне по-бырому, над чем тысячи умнейших людей планеты работают последние сто лет, лучше в паре фраз"
Аноним 17/09/15 Чтв 19:17:01 #469 №298505 
>>298503
Да! Примерно это мне и нужно!
Вообще, хотя бы отправную точку. :(
Вот по математике есть хорошая книга "Что такое математика?" Рихарда Курранта.
Аноним 17/09/15 Чтв 19:20:30 #470 №298507 
>>298505
По физике тоже есть, если ты можешь в английский.
Lawrie - A unified grand tour of theoretical physics.
Аноним 17/09/15 Чтв 21:06:01 #471 №298554 
Поясните пожалуйста, что значит деление по модулю 2?
Аноним 17/09/15 Чтв 21:08:06 #472 №298556 
>>298505
Науч поп тебе нужен?
"Кратчайшая история времени" Хоккинга
Аноним 18/09/15 Птн 05:04:55 #473 №298596 
>>298554
Деление с остатком.
15=7x2+1
Где 1 - очевидный остаток.
Макаба одну звездочку не пропускает, ну и пиздец.
Аноним 18/09/15 Птн 13:28:37 #474 №298626 
Почему пластик на морозе и металлы, политые жидким азотом, становятся очень хрупкими?
Аноним 18/09/15 Птн 13:35:37 #475 №298628 
>>298626
Фальсифицируемого объяснения нет. Нефальсифицируемое - что-то с атомами происходит.
Аноним 18/09/15 Птн 13:43:45 #476 №298629 
>>298626
У металлов ударная прочность и пластичность равномерно снижается со снижением температуры.
Пластики являются сложными веществами с хуй пойми какой массой молекулы, поэтому у них нет такого понятия, как температура плавления - для формования изделия нагревают и под давлением придают форму. При этом, при комнатной температуре форму пластика можно менять, но в значительно меньшей степени. При определенной температуре пластик затвердевает полностью, для полипропилена это что-то вроде -10 градусов, для более легкого полиэтилена эта температура ниже. При затвердевании он сохраняет прочностные характеристики, но становится совершенно негибким, таким образом в нем можно создать локальное большой перенапряжение изгибом через рычаг из самого этого пластика.
На эту тему еще можно еще вспомнить стекло, которое тоже при температурах жидкой фазы воды представляет собой полимер, очень твердый, но и очень хрупкий. Вот такими и становятся металлы и пластики при низкой температуре.
Аноним 18/09/15 Птн 19:04:23 #477 №298708 
>>298628
;)
Аноним 18/09/15 Птн 19:32:04 #478 №298722 
>>298629
Аморфные. Как шоколад.
Аноним 19/09/15 Суб 03:18:46 #479 №298821 
>закон сохранения энергии следствие однородности времени
>в ОТО масса создает неоднородность времени
>красная область спектра фотона имеет энергию меньше чем фиолетовая область спектра
>гравитационное красное смещение, смещает фотоны в красную область спектра за счет растяжения пространства.
Ну и какого хуя закон сохранения энергии до сих пор такая абсалютная истина, что если что-то его нарушает, то это сразу шлют нахрен?
Аноним 19/09/15 Суб 07:09:03 #480 №298832 
>>298821
А теперь подумай, куда девается энергия "краснеющего" фотона. Хинт - движение заряженной частицы в электрическом поле.
Аноним 19/09/15 Суб 11:49:00 #481 №298857 
>>298821
Энергию просто размазывает по пространству. Она была сперва плотно упакована в частоту х а потом растянулась в частоту 1/100000 х
Аноним 19/09/15 Суб 12:25:53 #482 №298858 
>>298857
Как её размазывает, если фотон дискретен и его энергия = hv? Размазать может волну, но не частицу(иначе должно измениться количество фотонов). Я думаю тут дело в том, что гравитация работает в обе стороны - "закрасняя" фотон чёрная дыра/галактика испытывает со стороны фотона равное усилие. Я конечно пытаюсь это интерпретировать классически, пусть релятивисты разъяснят чё к чему.
Аноним 19/09/15 Суб 14:35:11 #483 №298883 
>>298858
Энергия равна постоянной планка умноженной на частоту. Частота это количество волн в секунду или количество волн на неком отрезке пространства.
Вот пространство расширяется, и все что в нем тоже расширяется. Частицы удерживает от расширения сила гравитации, а волны такого свойтва лишены. Плюс, энергия фотона не сосредоточена в какой то одной точке, а может находится в любой точке волны, в промежутке от начала до конца.
Аноним 20/09/15 Вск 19:24:14 #484 №299142 
14427662542240.jpg
>>294030 (OP)
Сап всем.
Народ, вопрос таков. У нас есть пустое множество без элементов - нуль, конечные множества, состоящие из ограниченного числа элементов, и бесконечные множества - счётные и несчётные. Вопрос таков - существует ли что-то такое, что имеет совершенно другую математическую природу? Ведь нулём можно описать отсутствие чего-либо, например, что значение функции в данной точке принимает нуль, конечным множеством можно оперировать в той же физике, где стараются избегать расчётов, связанных с бесконечностью, а бесконечность активно используется при расчётах в теории множеств. Так вот существует ли что-то ещё, что-то такое, что способно повергнуть в трепет своим могуществом, рядом с которым даже бесконечность будет казаться не столь невообразимой? Или уже само понятие бесконечности предполагает, что она не имеет никаких ограничений и вследствие этого больше её уже быть не может, только бесконечность с большей мощностью множества?
Аноним 20/09/15 Вск 19:33:51 #485 №299148 
>>294030 (OP)
Котаны, в универсум входит область непознанного и он бесконечен, или же он представляет собой лишь ограниченную область, которую мы можем охватить своим разумом?
knuebok 20/09/15 Вск 19:51:46 #486 №299157 
>>299142
Ты как-то сильно в астрал ушёл. Есть всякие высшие категории, в которых разбираются 1.5 человека, если тебе интересно.
Аноним 20/09/15 Вск 19:55:54 #487 №299159 
>>299142
Ещё выше множеств лежат классы - совокупности настолько большие, что их нельзя считать множествами без того, чтобы вылезали противоречия. К примеру, класс всех множеств.
Аноним 20/09/15 Вск 20:09:22 #488 №299173 
>>294030 (OP)
Подскажите учебник по основам (или высший уровень) генной инженерии читать онлайн или скачать
Аноним 20/09/15 Вск 21:45:33 #489 №299228 
>>299159
Спасибо большое.
Вы имели в виду универсальное множество, или я немного не так поняла? И есть ли что-нибудь большее, чем классы?
knuebok 20/09/15 Вск 21:57:47 #490 №299232 
>>299228
>или я немного не так поняла?
Немного не так поняла.
>И есть ли что-нибудь большее, чем классы?
Очевидно, есть.
Аноним 20/09/15 Вск 21:58:10 #491 №299233 
>>298454
Здравствуй, добрый человек.
Это самоанцы, и, помимо американского футбола, они сильны в регби. Они, как правило, приземисты и чрезвычайно коренасты. Можешь поискать Гэри Гудриджа или Марка Ханта, у второго вообще прозвище Super Samoan из-за его чудовищных нокаутирующих ударов.
Ах да, ещё забыл про Дэвида Туа, который проигрывал лишь судейским решением - у чувака просто каменная башка какая-то.
Аноним 21/09/15 Пнд 01:21:55 #492 №299284 
>>294030 (OP)
Есть какая то литература на тему ускорителей?
Аноним 21/09/15 Пнд 08:03:33 #493 №299321 
Прям вообще стыдный вопрос.

А есть системы исчисления в которых Pi целое число?
Аноним 21/09/15 Пнд 08:25:09 #494 №299322 
>>294030 (OP)
Йо, ребятки. Такой вопрос: нет ни у кого годных книжек по ночным прицелам в электронном формате? Че-то никак не выбраться до библиотеки, а в гугле меня скоро забанят уже за такие запросы.
Аноним 21/09/15 Пнд 08:34:08 #495 №299324 
>>299321
Ну сделай сам, ёпты, клоун. Считай в базе pi. Тогда у тебя трансцендентными будут традиционные рациональные числа.
Аноним 21/09/15 Пнд 10:17:58 #496 №299334 
>>299321
Есть, но бездоказательно скажу, что тогда весь ряд натуральных для нас чисел станет дробным иррациональным. Заебись с такой хуйней жить.
Твое дело математически доказать, почему в системах где пи целое число натуральный ряд иррациональный.

Тащемта, для строительства особых пирамид с порталами в другие миры как раз нужны инструменты проектирования и счисление, где число пи равно 1.
Аноним 21/09/15 Пнд 10:42:28 #497 №299343 
>>299334
Ты только что неэвклидовую геометрию.
Аноним 21/09/15 Пнд 12:30:45 #498 №299373 
14428278452680.gif
Ребят, посоветуйте какую-нибудь толковую монографию по акустике. Чтобы прям в одной книге всё самое необходимое было описано. Гугл выдаёт в основном либо явно узконаправленные книги - какая-нибудь нелинейная акустика, гидроакустика, итд. А мне прям про звук в целом надо, а не изучение отдельных прикладных моделей.
Аноним 21/09/15 Пнд 12:41:56 #499 №299376 
>>299373
Звук это возмущения давления воздуха.
Аноним 21/09/15 Пнд 12:46:47 #500 №299378 
>>299376
Угу, а где-то в голове у нас есть барабанная перепонка, входящая в состав слухового аппарата.
Аноним 21/09/15 Пнд 13:38:44 #501 №299387 
14428319249540.png
>>294030 (OP)
Кто то может дать зельдовича высшая математика для начинающих физиков и техников не в такой ущербной колонковой форме? Чтобы все было как у белых людей, одна строчка на всю ширину странички.
Аноним 21/09/15 Пнд 15:07:06 #502 №299414 
Анон, смотри какой вопрос.
На днях Пахом пришел на Битву Экстрасенсов и два раза угадал машину, в которой лежит человек.
Допустим, машин было 20.
Сколько раз нужно угадать Пахому, чтобы его результат составил более пяти стандартных отклонений и был признан наукой?(допустим, эксперимент будет проведен корректно).
http://elementy.ru/LHC/HEP/study/errors/sigma читал, но как сюда подставить нужные значения не понимаю.
Аноним 21/09/15 Пнд 16:47:57 #503 №299445 
Посоны, что мешает электрону встретиться с протоном?
Аноним 21/09/15 Пнд 17:05:05 #504 №299451 
>>299445
Нейтрон.
Аноним 21/09/15 Пнд 17:16:18 #505 №299452 
>>299451
Но ведь нейтрон нейтральный. И у водорода вроде бы нет нейтрона.
Аноним 21/09/15 Пнд 18:49:45 #506 №299468 
>>299445
ничего лол, разве что малые размеры. Тем более они притягиваются. Если ты про атом водорода - то мешает электрон входящий в "электронное облако" атома.
Аноним 21/09/15 Пнд 19:37:30 #507 №299479 
>>299468
Значит с некоторой вероятностью электрон находится внутри ядра?
sageАноним 21/09/15 Пнд 19:38:57 #508 №299480 
14428535372890.jpg
Потцоны, где скачать аудиолекции какие-нибудь по предметам естественно-научного цикла? Или там аудиокниги какие, но не уровня докинза-хокинга, а посерьезнее. философия is fine too
На край сгодятся подкасты типа стр42 или видосиков с ютуба, но очень не хочется ебли с конвертированием и вырезанием аудио.
Аноним 21/09/15 Пнд 19:39:37 #509 №299482 
>>299480
ой, сажица приклеилась
Аноним 21/09/15 Пнд 22:13:48 #510 №299532 
У меня есть три молекулы вещества. Можно ли узнать, какой цвет у вещества, запах, агрегатное состояние при нуле градусов, хорошо ли растворяется в воде?
Аноним 21/09/15 Пнд 22:38:45 #511 №299536 
>>299532
Можно.
Аноним 21/09/15 Пнд 22:42:30 #512 №299538 
>>299480
В интернете есть специальные сайты с курсами по разным дисциплинам. Курсера, универсариум, edx и так далее. Аудиоподкасты можно найти в магазине айтюнса.
Аноним 22/09/15 Втр 03:28:41 #513 №299597 
>>299536
>Можно.
Каким образом это определяется?
Аноним 22/09/15 Втр 08:38:32 #514 №299619 
>>299597
Гугл в помощ.
Аноним 22/09/15 Втр 08:42:08 #515 №299620 
>>299597
Могу сказать по спектроскопии.
Поглощение света, которое цвет вещества определяет, зависит от конфигурации энергетических состояний электронов внешних оболочек атомов молекулы, по крайней мере для видимого спектра. Соответственно построение диаграммы состояний для каждого атома с учётом окружения и агрегатного состояния даст тебе знание о том, какой диапазон поглощения и испускания у этих молекул возможен.

Но так никто не делает. Я сейчас на вскидку не скажу где конкретно ты воткнешься, но поиск волновой функции электронов не водородоподобного атома уже гемор.
Аноним 23/09/15 Срд 22:13:19 #516 №300127 
Насколько трудно написать аи для 7 роботов, чтобы смоделировать поведение стаи(птичьей) или роя?

Тема проекта: Mit den vorhandenen 7 ct`Bots (Roboter-Hardware) soll ein Schwarmverhalten demonstriert werden.
comments powered by Disqus

Отзывы и предложения